0% found this document useful (0 votes)
87 views162 pages

Math 54 Module

This document contains terms of use for course material for MATH 54: Elementary Analysis II taught in the second semester of the 2023-2024 academic year. The material was prepared by Ramises G. Manzano Jr., PhD and is strictly for use by students officially enrolled in the course. Copyright protection of the material is governed by Philippine law and permission is required to print, reproduce, share, or sell the content. The document consists of seven chapters that provide techniques and concepts for integration, improper integrals, infinite sequences and series.

Uploaded by

djhuliamarie
Copyright
© © All Rights Reserved
We take content rights seriously. If you suspect this is your content, claim it here.
Available Formats
Download as PDF, TXT or read online on Scribd
0% found this document useful (0 votes)
87 views162 pages

Math 54 Module

This document contains terms of use for course material for MATH 54: Elementary Analysis II taught in the second semester of the 2023-2024 academic year. The material was prepared by Ramises G. Manzano Jr., PhD and is strictly for use by students officially enrolled in the course. Copyright protection of the material is governed by Philippine law and permission is required to print, reproduce, share, or sell the content. The document consists of seven chapters that provide techniques and concepts for integration, improper integrals, infinite sequences and series.

Uploaded by

djhuliamarie
Copyright
© © All Rights Reserved
We take content rights seriously. If you suspect this is your content, claim it here.
Available Formats
Download as PDF, TXT or read online on Scribd
You are on page 1/ 162

Terms of use: This course material is strictly for class use only to students officially

enrolled in MATH 54, ELEMENTARY ANALYSIS II, SECOND SEMESTER, A.Y. 2023 - 2024

M A T H
5 4

Learner’s Module
ELEMENTARY ANALYSIS II

Copyright protection is governed by R.A. 10372. This work cannot be


printed, reproduced, shared, or sold without permission from the author.
Terms of use: This course material is strictly for class use only to students officially
enrolled in MATH 54, ELEMENTARY ANALYSIS II, SECOND SEMESTER, A.Y. 2023 - 2024

T HIS MODULE IS PREPARED BY R AMISES G. M ANZANO J R , P H D


Copyright protection is governed by R.A. 10372. This work cannot be
printed, reproduced, shared, or sold without permission from the author.
Terms of use: This course material is strictly for class use only to students officially
enrolled in MATH 54, ELEMENTARY ANALYSIS II, SECOND SEMESTER, A.Y. 2023 - 2024

printed, reproduced, shared, or sold without permission from the author.


Copyright protection is governed by R.A. 10372. This work cannot be
Contents

I Week One

1 Techniques of Integration . . . . . . . . . . . . . . . . . . . . . . . . . . . . . . . . . . . . . . . . . 9
1.1 Integration by Parts 10

1.2 Trigonometric Integrals 14

1.3 Exercises 19

II Week Two

2 Techniques of Integration . . . . . . . . . . . . . . . . . . . . . . . . . . . . . . . . . . . . . . . . 21
2.1 Trigonometric Substitution 21

2.2 Exercises 28
III Week Three

3 Techniques of Integration . . . . . . . . . . . . . . . . . . . . . . . . . . . . . . . . . . . . . . . . 30
3.1 Integration of Rational Functions by Partial Fractions 30
3.1.1 Case I: The denominator Q(x) is a product of distinct linear factors. . . . . . . . . . . . . . . . . . 32
3.1.2 Case II: Q(x) is a product of linear factors, some of which are repeated. . . . . . . . . . . . . . 34
Terms of use: This course material is strictly for class use only to students officially

3.1.3 Case III: Q(x) contains irreducible quadratic factors, none of which is repeated . . . . . . . . 35
enrolled in MATH 54, ELEMENTARY ANALYSIS II, SECOND SEMESTER, A.Y. 2023 - 2024

3.1.4 Case IV: Q(x) contains a repeated irreducible quadratic factor . . . . . . . . . . . . . . . . . . . . 36

printed, reproduced, shared, or sold without permission from the author.


3.2 Exercises 37

Copyright protection is governed by R.A. 10372. This work cannot be


IV Week Four

4 Improper Integrals . . . . . . . . . . . . . . . . . . . . . . . . . . . . . . . . . . . . . . . . . . . . . . . 39
4.1 Improper Integrals with Infinite Limits of Integration 39

4.2 Improper Integrals with Infinite Discontinuities 43

4.3 Exercises 45

V Week Five

5 Infinite Sequence and Series . . . . . . . . . . . . . . . . . . . . . . . . . . . . . . . . . . . . . 48


5.1 Sequences 48

5.2 Series 56

5.3 Exercises 61

VI Week Six

6 Infinite Sequence and Series . . . . . . . . . . . . . . . . . . . . . . . . . . . . . . . . . . . . . 64


6.1 The Integral Test 64

6.2 The Comparison Tests 67

6.3 Alternating Series 70

6.4 Exercises 71
VII Week Seven

7 Infinite Sequence and Series . . . . . . . . . . . . . . . . . . . . . . . . . . . . . . . . . . . . . 73


7.1 Absolute Convergence 73
7.1.1 The Ratio Test . . . . . . . . . . . . . . . . . . . . . . . . . . . . . . . . . . . . . . . . . . . . . . . . . . . . . . . 75
7.1.2 The Root Test . . . . . . . . . . . . . . . . . . . . . . . . . . . . . . . . . . . . . . . . . . . . . . . . . . . . . . . 76
Terms of use: This course material is strictly for class use only to students officially

7.2 Rearrangements 77
enrolled in MATH 54, ELEMENTARY ANALYSIS II, SECOND SEMESTER, A.Y. 2023 - 2024

7.3 Strategy for Testing Series 78

printed, reproduced, shared, or sold without permission from the author.


Copyright protection is governed by R.A. 10372. This work cannot be
7.4 Exercises 80

VIII Week Eight

8 Infinite Sequences and Series . . . . . . . . . . . . . . . . . . . . . . . . . . . . . . . . . . . . 82


8.1 Power Series 82

8.2 Exercises 86

IX Week Nine

9 Infinite Sequences and Series . . . . . . . . . . . . . . . . . . . . . . . . . . . . . . . . . . . . 88


9.1 Representations of Functions as Power Series 88

9.2 Differentiation and Integration of Power Series 89

9.3 Taylor and Maclaurin Series 91

9.4 Exercises 95

X Week Ten

10 Partial Derivatives . . . . . . . . . . . . . . . . . . . . . . . . . . . . . . . . . . . . . . . . . . . . . . . . 98
10.1 Three-Dimensional Coordinate Systems 98
10.1.1 3D Space . . . . . . . . . . . . . . . . . . . . . . . . . . . . . . . . . . . . . . . . . . . . . . . . . . . . . . . . . . 98

10.2 Functions of Several Variables 103

10.3 Limits and Continuity 104


10.4 Exercises 108

XI Week Eleven

11 Partial Derivatives . . . . . . . . . . . . . . . . . . . . . . . . . . . . . . . . . . . . . . . . . . . . . . . 111


11.1 Partial Derivatives 111
Terms of use: This course material is strictly for class use only to students officially

11.1.1 Geometric Interpretations of Partial Derivatives . . . . . . . . . . . . . . . . . . . . . . . . . . . . . . . 113


enrolled in MATH 54, ELEMENTARY ANALYSIS II, SECOND SEMESTER, A.Y. 2023 - 2024

11.1.2 Higher-Order Partial Derivatives . . . . . . . . . . . . . . . . . . . . . . . . . . . . . . . . . . . . . . . . . . 115

printed, reproduced, shared, or sold without permission from the author.


Copyright protection is governed by R.A. 10372. This work cannot be
11.2 The Chain Rule 116
11.2.1 Implicit Differentiation . . . . . . . . . . . . . . . . . . . . . . . . . . . . . . . . . . . . . . . . . . . . . . . . . 118

11.3 Exercises 119

XII Week Twelve

12 Partial Derivatives . . . . . . . . . . . . . . . . . . . . . . . . . . . . . . . . . . . . . . . . . . . . . . . 122


12.1 Maximum and Minimum Values of Functions 122

12.2 Exercises 127

XIII Week Thirteen

13 Multiple Integrals . . . . . . . . . . . . . . . . . . . . . . . . . . . . . . . . . . . . . . . . . . . . . . . 130


13.1 Double Integrals Over Rectangles 130
13.1.1 Volumes and Double Integrals . . . . . . . . . . . . . . . . . . . . . . . . . . . . . . . . . . . . . . . . . . . 131
13.1.2 The Midpoint Rule . . . . . . . . . . . . . . . . . . . . . . . . . . . . . . . . . . . . . . . . . . . . . . . . . . . . 135

13.2 Exercises 137

XIV Week Fourteen

14 Multiple Integrals . . . . . . . . . . . . . . . . . . . . . . . . . . . . . . . . . . . . . . . . . . . . . . . 139


14.1 Iterated Integrals 139
14.1.1 Area of a Plane Region . . . . . . . . . . . . . . . . . . . . . . . . . . . . . . . . . . . . . . . . . . . . . . . . 144

14.2 Exercises 146


Terms of use: This course material is strictly for class use only to students officially
enrolled in MATH 54, ELEMENTARY ANALYSIS II, SECOND SEMESTER, A.Y. 2023 - 2024

15

15.3
15.2
15.1
XV

Exercises
Double Integrals in Polar Coordinates
Double Integrals Over General Regions
Week Fifteen

156
7

148
Multiple Integrals . . . . . . . . . . . . . . . . . . . . . . . . . . . . . . . . . . . . . . . . . . . . . . . 148

161

Copyright protection is governed by R.A. 10372. This work cannot be


printed, reproduced, shared, or sold without permission from the author.
Terms of use: This course material is strictly for class use only to students officially
enrolled in MATH 54, ELEMENTARY ANALYSIS II, SECOND SEMESTER, A.Y. 2023 - 2024

1.3
1.2
1.1

Exercises
Integration by Parts
Trigonometric Integrals
Week One

Techniques of Integration . . . . . . . . . . . . . 9
Copyright protection is governed by R.A. 10372. This work cannot be
printed, reproduced, shared, or sold without permission from the author.
Terms of use: This course material is strictly for class use only to students officially
enrolled in MATH 54, ELEMENTARY ANALYSIS II, SECOND SEMESTER, A.Y. 2023 - 2024

printed, reproduced, shared, or sold without permission from the author.


Copyright protection is governed by R.A. 10372. This work cannot be
1. Techniques of Integration

At the end of this topic you are expected to:


• find integrals using integration by parts
• solve trigonometric integrals

Below is a list of some important integrals that we have studied so far in Math 53.

xn+1 1
Z Z
n
x dx = +C (n ̸= −1) dx = ln |x| +C
n+1 x

bx
Z Z
x x x
e dx = e +C b dx = +C
ln b
Z Z
sin x dx = − cos x +C cos x dx = sin x +C

Z Z
sec2 x dx = tan x +C csc2 x dx = − cot x +C

Z Z
sec x tan x dx = sec x +C csc x cot x dx = − csc x +C
1.1 Integration by Parts 10
Z Z
sinh x dx = cosh x +C cosh x dx = sinh x +C

Z Z
tan x dx = ln | sec x| +C cot x dx = ln | sin x| +C

1 1 −1  x  1 x
Z Z
dx = tan +C √ dx = sin−1 +C, a > 0
x2 + a2 a a a2 − x2 a
Terms of use: This course material is strictly for class use only to students officially

1.1 Integration by Parts


enrolled in MATH 54, ELEMENTARY ANALYSIS II, SECOND SEMESTER, A.Y. 2023 - 2024

printed, reproduced, shared, or sold without permission from the author.


Integration by parts is used to a wide variety of functions and is particularly useful for

Copyright protection is governed by R.A. 10372. This work cannot be


integrands involving products of algebraic and transcendental functions. Here are some
integrals that uses integration by parts:
Z Z Z
2 x
x ln x dx x e dx ex cos x dx

Integration by parts is based on the formula for the derivative of a product


d dv du
[uv] = u + v = uv′ + vu′
dx dx dx
where u and v are differentiable functions of x. After interchanging both sides of the
equation we obtain
Z Z Z Z
′ ′
uv = uv dx + vu dx = u dv + v du

or
Z Z
uv − v du = u dv

and so we have the following theorem.


Theorem 1.1.1 (Integration by Parts)

If u and v are functions of x and have continuous derivatives, then


Z Z
u dv = uv − v du.
Z
■ Example 1.1 Find x ex dx.

Solution: We need to form the integral as in the above theorem. There are only two things
to pick, those are, u and dv. Usually, we select u to be the function with a much simpler
derivative and the remaining parts of the integral will be our dv. In our case,
1.1 Integration by Parts 11

Let u = x and dv = ex dx

Then we differentiate u and integrate dv giving us

du = dx and v = ex

We will just remove the constant of integration C in v for it will just cancel out in the
integration process later. Thus, we have
Terms of use: This course material is strictly for class use only to students officially
enrolled in MATH 54, ELEMENTARY ANALYSIS II, SECOND SEMESTER, A.Y. 2023 - 2024

Z Z
u dv = uv − v du

printed, reproduced, shared, or sold without permission from the author.


Copyright protection is governed by R.A. 10372. This work cannot be
Z Z
xex dx = xex − ex dx

= xex − ex +C.

Have you ever wondered on what is the integral of the function f (x) = ln x? Integration by
parts will give us the answer.
Z
■ Example 1.2 Find the ln x dx.

Solution: Let us form the formula by choosing u and dv.

Let u = ln x and dv = dx.

Then we differentiate u and integrate dv giving us

du = 1x dx and v = x.

Thus, we can form our integral as

Z Z
u dv = uv − v du
1
Z Z
ln x dx = x ln x − x · dx
Z x
= x ln x − dx

= x ln x − x +C.

There are also times when integration by parts are to be performed more than once as in
our next example.
1.1 Integration by Parts 12
Z
■ Example 1.3 Find x2 sin x dx.

Solution: Let us form the formula by choosing u and dv.

Let u = x2 and dv = sin x dx.

Then we differentiate u and integrate dv giving us

du = 2x dx and v = − cos x.
Terms of use: This course material is strictly for class use only to students officially
enrolled in MATH 54, ELEMENTARY ANALYSIS II, SECOND SEMESTER, A.Y. 2023 - 2024

Thus, we can form our integral as

printed, reproduced, shared, or sold without permission from the author.


Copyright protection is governed by R.A. 10372. This work cannot be
Z Z
u dv = uv − v du
Z Z
2 2
x sin x dx = −x cos x − (−2x cos x) dx
Z
= −x2 cos x + 2x cos x dx.
Z
Note that the integral on the right 2x cos x dx calls for another integration by parts. Let
us focus on this integral and perform the integration by parts for the second time.

Let u = 2x and dv = cos x dx.

Then we differentiate u and integrate dv giving us

du = 2 dx and v = sin x.

So,
Z Z
u dv = uv − v du
Z Z
2x cos x dx = 2x sin x − 2 sin x dx

= 2x sin x + 2 cos x +C.

Therefore,
Z Z
2 2
x sin x dx = −x cos x + 2x cos x dx

= −x2 cos x + 2x sin x + 2 cos x +C.

There are also cases when the given integral reappears on the right-hand side of the equation
upon applying integration by parts repeatedly like in our next example.
1.1 Integration by Parts 13
Z
■ Example 1.4 Find ex sin x dx.

Solution: Choosing u = ex and dv = sin x dx and finding du and v, we have

du = ex dx and v = − cos x.

So,
Z Z
x x
ex cos x dx
Terms of use: This course material is strictly for class use only to students officially

e sin x dx = −e cos x +
enrolled in MATH 54, ELEMENTARY ANALYSIS II, SECOND SEMESTER, A.Y. 2023 - 2024

printed, reproduced, shared, or sold without permission from the author.


Copyright protection is governed by R.A. 10372. This work cannot be
Z
The integral ex cos x dx is as pretty the same as our given integral. Hence, we employ
integration by parts for the second time. This time we use u = ex and dv = cos x dx. Thus, we
have

du = ex dx and v = sin x

Then
Z Z
x x
e cos x dx = e sin x − ex sin x dx

Upon inspection, we have seen an integral on the right exactly the same as what we have
started earlier. Moreover,
Z Z
x x x
e sin x dx = −e cos x + e sin x − ex sin x dx.

This means that


Z Z
x
e sin x dx + ex sin x dx = −ex cos x + ex sin x
Z
2 ex sin x dx = −ex cos x + ex sin x

Therefore,
1
Z
ex sin x dx = ex (sin x − cos x) +C.
2
If we combine the formula for integration by parts and the Fundamental Theorem of
Calculus, we obtain the formula
1.2 Trigonometric Integrals 14

Z b b Z b
f (x)g′ (x) dx = f (x)g(x) − g(x) f ′ (x) dx
a a a
Z 1
■ Example 1.5 Evaluate tan−1 x dx.
0
Solution: Let
u = tan−1 x and dv = dx.
Terms of use: This course material is strictly for class use only to students officially

Then,
enrolled in MATH 54, ELEMENTARY ANALYSIS II, SECOND SEMESTER, A.Y. 2023 - 2024

printed, reproduced, shared, or sold without permission from the author.


du = dx and v = x.
1 + x2

Copyright protection is governed by R.A. 10372. This work cannot be


Hence,

Z 1 1 Z 1
−1 −1 x
tan x dx = x tan x − dx
0 0 0 1 + x2
Z 1
−1 −1 x
= 1 · tan (1) − 0 · tan (0) − dx
0 1 + x2
Z 1
π x
= − dx
4 0 1 + x2
Z 1
x
Let us evaluate 2
dx by substitution. Let t = 1 + x2 . Then dt = 2x dx. Moreover,
0 1 + x
when x = 0, t = 1 and when x = 1, t = 2. Hence,

Z 1 2
x 1 2 dt 1
Z
dx = = ln |t|
0 1 + x2 2 1 t 2 1
1
= (ln 2 − ln 1)
2
1
= ln 2
2
Therefore,
Z 1 Z 1
π x π 1
tan−1 x dx = − dx = − ln 2.
0 4 0 1 + x2 4 2

1.2 Trigonometric Integrals

In this section, we use trigonometric identities to integrate certain combinations of trigonometic


functions. We begin with powers of sine and cosine.
1.2 Trigonometric Integrals 15
Z
■ Example 1.6 Find cos3 x dx.

Solution: Here, we can separate one cosine factor and convert the remaining cos2 x factor to an
expression involving sine using the identity sin2 x + cos2 x = 1:

cos3 x = cos2 x · cos x = (1 − sin2 x) cos x.


Terms of use: This course material is strictly for class use only to students officially

If u = sin x, then du = cos x dx and


enrolled in MATH 54, ELEMENTARY ANALYSIS II, SECOND SEMESTER, A.Y. 2023 - 2024

Z Z Z
cos3 x dx = cos2 x · cos x dx = (1 − sin2 x) cos x dx

printed, reproduced, shared, or sold without permission from the author.


Copyright protection is governed by R.A. 10372. This work cannot be
1
Z
= (1 − u2 )du = u − u3 +C
3
1 3
= sin x − sin x +C.
3
Z
■ Example 1.7 Find sin5 x cos2 x dx.

Solution: Here, we separate a single sine factor and rewrite the remaining sin4 x factor in terms
of cos x:
sin5 x cos2 x = (sin2 x)2 cos2 x sin x = (1 − cos2 x)2 cos2 x sin x

Substituting u = cos x, we have du = − sin x dx and so


Z Z
5 2
sin x cos x dx = (sin2 x)2 cos2 x sin x dx
Z
= (1 − cos2 x)2 cos2 x sin x dx
Z Z
= 2 2 2
(1 − u ) u (−du) = − (u2 − 2u4 + u6 ) du
 3 
u 2 5 1 7
=− − u + u +C
3 5 7
1 2 1
= − cos3 x + cos5 x − cos7 x +C.
3 5 7
An odd power of sine or cosine enabled us to separate a single factor and convert the
remaining even power. However, if the integrand contains an even power of both sine and
cosine, this strategy fails. In cases like these, we use the half-angle identities.
1 − cos 2x 1 + cos 2x
sin2 x = and cos2 x =
2 2
Z π
■ Example 1.8 Evaluate sin2 x dx.
0
1.2 Trigonometric Integrals 16

Solution: Using the half-angle formula for sin2 x, we have


1 π
Z π Z
2
sin x dx = (1 − cos 2x) dx
0 2 0
  π
1 1
= x − sin 2x
2 2
  0  
1 1 1 1 π
= π − sin 2π − 0 − sin 0 = .
2 2 2 2 2
Terms of use: This course material is strictly for class use only to students officially

To summarize, we list the guidelines in evaluating integrals of the form sinm x cosn x dx,
R
enrolled in MATH 54, ELEMENTARY ANALYSIS II, SECOND SEMESTER, A.Y. 2023 - 2024

printed, reproduced, shared, or sold without permission from the author.


where m ≥ 0 and n ≥ 0 are integers.

Copyright protection is governed by R.A. 10372. This work cannot be


Z
Strategy for Evaluating sinm x cosn x dx

(a) If the power of cosine is odd (n = 2k + 1), save one cosine factor and use
cos2 x = 1 − sin2 x to express the remaining factors in terms of sine:
Z Z
2k+1
m
sin x cos x dx = sinm x (cos2 x)k cos x dx
Z
= sinm x (1 − sin2 x)k cos x dx

Then substitute u = sin x.


(b) If the power of sine is odd (m = 2k + 1), save one sine factor and use
sin2 x = 1 − cos2 x to express the remaining factors in terms of cosine:
Z Z
2k+1
sin n
x cos x dx = (sin2 x)k cosn x sin x dx
Z
= (1 − cos2 x)k cosn x sin x dx

Then substitute u = cos x.


If the powers of both sine and cosine are odd, then either (a) or (b) can be used.
(c) If the powers of both sine and cosine are even, use the half-angle identities
1 − cos 2x 1 + cos 2x
sin2 x = and cos2 x =
2 2

A similar strategy can be used to evaluate integrals of the form tanm x secn x dx which we
R
1.2 Trigonometric Integrals 17

will summarize af follows.

Z
Strategy for Evaluating tanm x secn x dx

(a) If the power of secant is even (n = 2k), save a factor of


Terms of use: This course material is strictly for class use only to students officially

sec2 x and use sec2 x = 1 + tan2 x to express the remaining factors in terms of
enrolled in MATH 54, ELEMENTARY ANALYSIS II, SECOND SEMESTER, A.Y. 2023 - 2024

printed, reproduced, shared, or sold without permission from the author.


tan x:

Copyright protection is governed by R.A. 10372. This work cannot be


Z Z
2k
m
tan x sec x dx = tanm x (sec2 x)k−1 sec2 x dx
Z
= tanm x (1 + tan2 x)k−1 sec2 x dx

Then substitute u = tan x.


(b) If the power of tangent is odd (m = 2k + 1), save a factor of sec x tan x and use
tan2 x = sec2 x − 1 to express the remaining factors in terms of sec x:
Z Z
2k+1
tan n
x sec x dx = (tan2 x)k secn−1 x sec x tan x dx
Z
= (sec2 x − 1)k secn−1 x sec x tan x dx

Then substitute u = sec x.


Z
■ Example 1.9 Find tan6 x sec4 x dx.

Solution: This is case (a). We will evaluate the integral by substituting u = tan x so that
du = sec2 x dx.
Z Z
tan6 x sec4 x dx = tan6 x sec2 x sec2 x dx
Z
= tan6 x (1 + tan2 x) sec2 x dx
Z Z
6 2
= u (1 + u )du = (u6 + u8 ) du
u7 u9
= + +C
7 9
1 7 1
= tan x + tan9 x +C.
7 9
1.2 Trigonometric Integrals 18
Z
■ Example 1.10 Find tan5 θ sec7 θ dθ

Solution: This is case (b). We can evaluate the integral by substituting u = sec θ , so that
du = sec θ tan θ dθ :
Z Z
5 7
tan θ sec θ dθ = tan4 θ sec6 θ sec θ tan θ dθ
Z
(sec θ − 1)2 sec6 θ sec θ tan θ dθ
Terms of use: This course material is strictly for class use only to students officially

=
Z
enrolled in MATH 54, ELEMENTARY ANALYSIS II, SECOND SEMESTER, A.Y. 2023 - 2024

= (u2 − 1)2 u6 du

printed, reproduced, shared, or sold without permission from the author.


Copyright protection is governed by R.A. 10372. This work cannot be
Z
= (u10 − 2u8 + u6 )du
1 11 2 9 1 7
= u − u + u +C
11 9 7
1 2 1
= sec11 θ − sec9 θ + sec7 θ +C.
11 9 7
Z
■ Example 1.11 Find tan3 x dx.

Solution: Here only tan x occurs, so we use tan2 x = sec2 x − 1:


Z Z Z
3 2
tan x dx = tan x tan x dx = tan x (sec2 x − 1) dx
Z Z
2
= tan x sec x dx − tan x dx
tan2 x
= − ln | sec x| +C.
2
Some powers of sec x may require integrations by parts, as shown in the next example.
Z
■ Example 1.12 Find sec3 x dx.

Solution: Here, we integrate by parts with

u = sec x dv = sec2 x dx

du = sec x tan x dx v = tan x


1.3 Exercises 19

Then
Z Z
3
sec x dx = sec x tan x − sec x tan2 x dx
Z
= sec x tan x − sec x (sec2 x − 1) dx
Z Z
3
= sec x tan x − sec x dx + sec x dx
Z
Terms of use: This course material is strictly for class use only to students officially

= sec x tan x − sec3 x dx + ln | sec x + tan x| +C1 .


enrolled in MATH 54, ELEMENTARY ANALYSIS II, SECOND SEMESTER, A.Y. 2023 - 2024

This means that

printed, reproduced, shared, or sold without permission from the author.


Copyright protection is governed by R.A. 10372. This work cannot be
Z
2 sec3 x = sec x tan x + ln | sec x + tan x| +C1
1
Z
sec3 x = (sec x tan x + ln | sec x + tan x| +C1 )
2
1
Z
Therefore, sec3 x = (sec x tan x + ln | sec x + tan x|) +C.
2

1.3 Exercises
Exercise 1.1 Evaluate the following integrals:
Z
1. x2 sin 4x dx
Z
2. x sec−1 x dx
Z
3. cos4 x dx
Z π/4
4. tan4 x dx
Z0
5. tan5 x sec4 x dx
Terms of use: This course material is strictly for class use only to students officially
enrolled in MATH 54, ELEMENTARY ANALYSIS II, SECOND SEMESTER, A.Y. 2023 - 2024

II

2.2
2.1
Exercises
Trigonometric Substitution
Week Two

Techniques of Integration . . . . . . . . . . . . . 21
Copyright protection is governed by R.A. 10372. This work cannot be
printed, reproduced, shared, or sold without permission from the author.
Terms of use: This course material is strictly for class use only to students officially
enrolled in MATH 54, ELEMENTARY ANALYSIS II, SECOND SEMESTER, A.Y. 2023 - 2024

printed, reproduced, shared, or sold without permission from the author.


Copyright protection is governed by R.A. 10372. This work cannot be
2. Techniques of Integration

At the end of this topic you are expected to:


• find integrals using trigonometric substitution

2.1 Trigonometric Substitution

In the first week, we have learned to find integrals involving powers of trigonometric functions.
This time, we will use the technique called trigonometric substitution to find the integrals
involving radicals of the form
√ √ √
a2 − u2 , a2 + u2 , and u2 − a2 .

The objective with trigonometric substitution is to eliminate the radical in the integrand.
The Pythagorean identities will aid us in our transformation.
Terms of use: This course material is strictly for class use only to students officially
enrolled in MATH 54, ELEMENTARY ANALYSIS II, SECOND SEMESTER, A.Y. 2023 - 2024

2.1 Trigonometric Substitution


q
p

= a cos θ .
cos2 θ = 1 − sin2 θ
sec2 θ = 1 + tan2 θ

tan2 θ = sec2 θ − 1

= a2 cos2 θ
a2 − u2 = a2 − a2 sin2 θ

= a2 (1 − sin2 θ )
For example, for a > 0, if u = a sin θ , where π/2 ≤ θ ≤ π/2. Then
22

Copyright protection is governed by R.A. 10372. This work cannot be


printed, reproduced, shared, or sold without permission from the author.
2.1 Trigonometric Substitution 23

Trigonometric Substitution (a > 0)



1. For integrals involving a2 − u2 , let u = a sin θ .

Then a2 − u2 = a cos θ , where −π/2 ≤ θ ≤ π/2.

a
u
Terms of use: This course material is strictly for class use only to students officially

θ

enrolled in MATH 54, ELEMENTARY ANALYSIS II, SECOND SEMESTER, A.Y. 2023 - 2024

a2 − u2

printed, reproduced, shared, or sold without permission from the author.


Copyright protection is governed by R.A. 10372. This work cannot be

2. For integrals involving a2 + u2 , let u = a tan θ .

Then a2 + u2 = a sec θ , where −π/2 < θ < π/2.


a2 + u2
u

θ
a

3. For integrals involving u2 − a2 , let u = a sec θ .
Then

 a tan θ for u > a, where 0 ≤ θ < π/2
p 
u2 − a2 =
−a tan θ

for u < −a, where π/2 < θ ≤ π.

u √
u2 − a2
θ
a

Remark 2.1.1 The restrictions on θ ensure the function that defines the substitution is one-to-

one. In fact, these are the same intervals over which the arcsine, arctangent, and arcsecant are
defined.
dx
Z
■ Example 2.1 Find √ .
x2 9 − x2
2.1 Trigonometric Substitution 24

Solution: We cannot use the basic integration rules here. This calls for a trigonometric
√ √
substitution. Note that our integrand contains 9 − x2 which is of the form a2 − u2 . So we
let

x = 3 sin θ

Thus, our reference triangle would be


Terms of use: This course material is strictly for class use only to students officially
enrolled in MATH 54, ELEMENTARY ANALYSIS II, SECOND SEMESTER, A.Y. 2023 - 2024

printed, reproduced, shared, or sold without permission from the author.


x

Copyright protection is governed by R.A. 10372. This work cannot be


θ

9 − x2

Moreover, dx = 3 cos θ dθ , 9 − x2 = 3 cos θ , and x2 = 9 sin2 θ .
Hence,

dx 3 cos θ dθ
Z Z
√ =
x2 9 − x2 (9 sin2 θ )(3 cos θ )
1 dθ
Z
=
9 sin2 θ
1
Z
= csc2 θ dθ
9
1
= − cot θ +C
9
√ !
1 9 − x2
=− +C
9 x

9 − x2
=− +C.
9x
dx
Z
■ Example 2.2 Find .
4x2 + 1
√ p √
Solution: Our integrand contains 4x2 + 1 = (2x)2 + 12 which is of the form u2 + a2 . So
we let

2x = 1 tan θ

Thus, our reference triangle would be


2.1 Trigonometric Substitution 25


4x2 + 1
2x
θ
1

Moreover, dx = 12 sec2 θ dθ , and 4x2 + 1 = sec θ .
Terms of use: This course material is strictly for class use only to students officially

Following the appropriate substitution gives us,


enrolled in MATH 54, ELEMENTARY ANALYSIS II, SECOND SEMESTER, A.Y. 2023 - 2024

dx 1 sec2 θ dθ
Z Z

printed, reproduced, shared, or sold without permission from the author.


√ =
4x2 + 1 2 sec θ

Copyright protection is governed by R.A. 10372. This work cannot be


1
Z
= sec θ dθ
2
1
= ln | sec θ + tan θ | +C
2
1 p 2
= ln | 4x + 1 + 2x| +C.
2
dx
Z
■ Example 2.3 Find .
(x2 + 1)3/2

Solution: Our integrand contains (x2 + 1)3/2 which we can write as ( x2 + 1)3 . So we let

x = 1 tan θ

Thus, our reference triangle would be


x2 + 1
x

θ
1

Moreover, dx = sec2 θ dθ , and x2 + 1 = sec θ .
2.1 Trigonometric Substitution 26

Following the appropriate substitution gives us,


dx dx
Z Z
= √
(x + 1)3/2
2 ( x + 1)3
2

sec2 θ dθ
Z
=
sec3 θ

Z
=
Z sec θ
Terms of use: This course material is strictly for class use only to students officially

= cos θ d θ
enrolled in MATH 54, ELEMENTARY ANALYSIS II, SECOND SEMESTER, A.Y. 2023 - 2024

printed, reproduced, shared, or sold without permission from the author.


= sin θ +C

Copyright protection is governed by R.A. 10372. This work cannot be


x
=√ +C.
2
x +1
Z 2 √ 2
x −3
■ Example 2.4 Evaluate √ dx.
3 x
√ √
Solution: Note that x2 − 3 is of the form u2 − a2 . So we let

x= 3 sec θ

Thus, our reference triangle would be

x √
x2 − 3
θ

3

√ √ √
Moreover, dx = 3 sec θ tan θ dθ , and x2 − 3 = 3 tan θ .

Let us determine the upper and lower limits of integration. When x = 3, sec θ = 1 and so,
θ = 0. When x = 2, sec θ = √2 ans so, θ = π6 .
3
Hence,
2.1 Trigonometric Substitution 27

Z 2 √ 2 Z π/6 √ √
x −3 ( 3 tan θ )( 3 sec θ tan θ )
√ dx = √ dθ
3 x 0 3 sec θ
Z π/6 √
= 3 tan2 θ dθ
0
√ Z π/6
= 3 (sec2 θ − 1) dθ
0
Terms of use: This course material is strictly for class use only to students officially


 π/6
= 3 tan θ − θ
enrolled in MATH 54, ELEMENTARY ANALYSIS II, SECOND SEMESTER, A.Y. 2023 - 2024

0

printed, reproduced, shared, or sold without permission from the author.




1 π

Copyright protection is governed by R.A. 10372. This work cannot be


= 3 √ −
3 6

3
= 1− π.
6
Z −√3 √ 2
x −3
■ Example 2.5 Evaluate dx.
−2 x
√ √
Solution: Note that x2 − 3 is of the form u2 − a2 . So we let

x= 3 sec θ

Thus, our reference triangle would be

x √
x2 − 3
θ

3

√ √ √ √
Moreover, dx = 3 sec θ tan θ dθ , and x2 − 3 = − 3 tan θ , since a = 3 and u = x

in [−2, − 3] would imply u < −a.
−2
Let us determine the upper and lower limits of integration. When x = −2, sec θ = √ and so,
3
5π √
θ= . When x = − 3, sec θ = −1 ans so, θ = π. (θ is chosen within π/2 < θ ≤ π)
6
Hence,
2.2 Exercises 28

Z −√3 √ 2 √ √
x −3 (− 3 tan θ )( 3 sec θ tan θ )
Z π
dx = √ dθ
−2 x 5π/6 3 sec θ
Z π √
= − 3 tan2 θ dθ
5π/6
√ Zπ
=− 3 (sec2 θ − 1) dθ
5π/6
Terms of use: This course material is strictly for class use only to students officially


 π
= − 3 tan θ − θ
enrolled in MATH 54, ELEMENTARY ANALYSIS II, SECOND SEMESTER, A.Y. 2023 - 2024

5π/6

printed, reproduced, shared, or sold without permission from the author.



  
1 5π

Copyright protection is governed by R.A. 10372. This work cannot be


= − 3 (0 − π) − − √ −
3 6

3
= −1 + π.
6

2.2 Exercises
Exercise 2.1 Evaluate the following integrals:
1
Z
1. 2 3/2
dx
Z (16 − x )
4
2. √ dx
x 2 16 − x2
x3
Z
3. dx
x2 − 25
Z √
16 − x2
4. dx
x
x2
Z 6
5. √ dx
4 x2 − 9
Terms of use: This course material is strictly for class use only to students officially
enrolled in MATH 54, ELEMENTARY ANALYSIS II, SECOND SEMESTER, A.Y. 2023 - 2024

III

3.2
3.1
Exercises
Integration of Rational Functions by Partial Fractions
Week Three

Techniques of Integration . . . . . . . . . . . . 30
Copyright protection is governed by R.A. 10372. This work cannot be
printed, reproduced, shared, or sold without permission from the author.
Terms of use: This course material is strictly for class use only to students officially
enrolled in MATH 54, ELEMENTARY ANALYSIS II, SECOND SEMESTER, A.Y. 2023 - 2024

printed, reproduced, shared, or sold without permission from the author.


Copyright protection is governed by R.A. 10372. This work cannot be
3. Techniques of Integration

At the end of this topic you are expected to:


• decompose rational functions as sums of partial fractions
• integrate rational functions using partial fractions

3.1 Integration of Rational Functions by Partial Fractions

In this section we show how to integrate any rational function (a ratio of polynomials) by
expressing it as a sum of simpler fractions, called partial fractions, that we already know
how to integrate. To illustrate the method, observe that by taking the fractions 2/(x − 1) and
1/(x + 2) to a common denominator we obtain
2 1 2(x + 2) − (x − 1) x+5
− = = 2
x−1 x+2 (x − 1)(x + 2) x +x−2
If we now reverse the procedure, we see how to integrate the function on the right side of
this equation:
3.1 Integration of Rational Functions by Partial Fractions 31

Z  
x+5 2 1
Z
2
dx = − dx
x +x−2 x−1 x+2
= 2 ln |x − 1| − ln |x + 2| +C

To see how the method of partial fractions works in general, let’s consider a rational
function
Terms of use: This course material is strictly for class use only to students officially

P(x)
enrolled in MATH 54, ELEMENTARY ANALYSIS II, SECOND SEMESTER, A.Y. 2023 - 2024

f (x) =
Q(x)

printed, reproduced, shared, or sold without permission from the author.


Copyright protection is governed by R.A. 10372. This work cannot be
where P and Q are polynomials. It’s possible to express f as a sum of simpler fractions
provided that the degree of P is less than the degree of Q. Such a rational function is called
proper. Recall that if

P(x) = an xn + an−1 xn−1 + · · · + a1 x + a0

where an ̸= 0, then the degree of P is n and we write deg(P) = n.


If f is improper, that is, deg(P) ≥ deg(Q), then we must take the preliminary step of
dividing Q into P (by long division) until a remainder R(x) is obtained such that deg(R) <
deg(Q). The division statement is
P(x) R(x)
f (x) = = S(x) +
Q(x) Q(x)
where S and R are also polynomials.
x3 + x
Z
■ Example 3.1 Find dx.
x−1
Solution: Since the degree of the numerator is greater than the degree of the denominator, we
first perform the long division. This enables us to write

x3 + x
Z  
2
Z
2
dx = x +x+2+ dx
x−1 x−1
x3 x2
= + + 2x + 2 ln |x − 1| +C.
3 2
In the case of a rational equations whose denominator is more complicated, the next step is
to factor the denominator Q(x) as far as possible. Then express the proper rational function
R(x)/Q(x) as a sum of partial fractions of the form
3.1 Integration of Rational Functions by Partial Fractions 32

A Ax + B
or
(ax + b)i (ax2 + bx + c) j
A theorem in algebra guarantees that it is always possible to do this. We explain the details
for the four cases that occur.

3.1.1 Case I: The denominator Q(x) is a product of distinct linear factors.


Terms of use: This course material is strictly for class use only to students officially

This means that we can write


enrolled in MATH 54, ELEMENTARY ANALYSIS II, SECOND SEMESTER, A.Y. 2023 - 2024

Q(x) = (a1 x + b1 )(a2 x + b2 ) · · · (ak x + bk )

printed, reproduced, shared, or sold without permission from the author.


Copyright protection is governed by R.A. 10372. This work cannot be
where no factor is repeated (and no factor is a constant multiple of another). In this case the
partial fraction theorem states that there exist constants A1 , A2 , . . . , Ak such that
R(x) A1 A2 Ak
= + +···+
Q(x) a1 x + b1 a2 x + b2 ak x + bk
x2 + 2x − 1
Z
■ Example 3.2 Evaluate dx.
2x3 + 3x2 − 2x
Solution: Since the degree of the numerator is less than the degree of the denominator, we
don’t need to divide. We factor the denominator as

2x3 + 3x2 − 2x = x(2x2 + 3x − 2) = x(2x − 1)(x + 2)

Since the denominator has three distinct linear factors, the partial fraction decomposition of
the integrand has the form
x2 + 2x − 1 A B C
= + +
x(2x − 1)(x + 2) x 2x − 1 x + 2
To determine the values of A, B, and C, we multiply both sides of this equation by the product
of the denominators, x(2x − 1)(x + 2), obtaining

x2 + 2x − 1 = A(2x − 1)(x + 2) + Bx(x + 2) +Cx(2x − 1)

Expanding the right side and writing it in the standard form for polynomials, we get

x2 + 2x − 1 = (2A + B + 2C)x2 + (3A + 2B −C)x − 2A

The polynomials in the previous equation are identical, so their coefficients must be equal.
The coefficient of x2 on the right side, 2A + B + 2C, must equal the coefficient of x2 on the left
side-namely, 1. Likewise, the coefficients of x are equal and the constant terms are equal. This
gives the following system of equations for A, B, and C:
3.1 Integration of Rational Functions by Partial Fractions 33

2A + B + 2C = 1
3A + 2B −C = 2
−2A = −1

Solving, we get A = 12 , B = 51 , and C = − 10


1
, and so

x2 + 2x − 1
Z  
A B C
Z
Terms of use: This course material is strictly for class use only to students officially

dx = + + dx
2x3 + 3x2 − 2x x 2x − 1 x + 2
enrolled in MATH 54, ELEMENTARY ANALYSIS II, SECOND SEMESTER, A.Y. 2023 - 2024

Z  
11 1 1 1 1

printed, reproduced, shared, or sold without permission from the author.


= + − dx
2 x 5 2x − 1 10 x + 2

Copyright protection is governed by R.A. 10372. This work cannot be


1 1 1
= ln |x| + ln |2x − 1| − ln |x + 2| + K
2 10 10
In integrating the middle term we have made the mental substitution u = 2x − 1, which gives
1
du = 2dx and dx = du.
2
Remark 3.1.1 We can use an alternative method to find the coefficients A, B, and C in the

previous example. Let’s choose values of x that simplify the equation

x2 + 2x − 1 = A(2x − 1)(x + 2) + Bx(x + 2) +Cx(2x − 1).

If we put x = 0 in the equation, then the second and third terms on the right side vanish and the
1 1
equation then becomes −2A = −1, or A = 12 . Likewise, x = gives 5B/4 = and x = −2
2 4
1 1
gives 10C = −1, so B = and C = − .
5 10
dx
Z
■ Example 3.3 Find , where a ̸= 0.
x 2 − a2
Solution: The method of partial fractions gives
1 1 A B
2 2
= = +
x −a (x − a)(x + a) x − a x + a
and therefore

A(x + a) + B(x − a) = 1

Using the method of the preceding note, we put x = a in this equation and get A(2a) = 1, so
A = 1/(2a). If we put x = −a, we get B(−2a) = 1, so B = −1/(2a). Thus,
Z  
dx 1 1 1
Z
= − dx
x2 − a2 2a x−a x+a
1
= (ln |x − a| − ln |x + a|) +C
2a
3.1 Integration of Rational Functions by Partial Fractions 34

Since ln x − ln y = ln(x/y), we can write the integral as


dx 1 x−a
Z
= ln +C
x 2 − a2 2a x+a

3.1.2 Case II: Q(x) is a product of linear factors, some of which are repeated.
Suppose the first linear factor (a1 x + b1 ) is repeated r times; that is, (a1 x + b1 )r occurs in the
factorization of Q(x). Then instead of the single term Ai /(a1 x + b1 ), we would use
Terms of use: This course material is strictly for class use only to students officially

A1 A2 Ar
enrolled in MATH 54, ELEMENTARY ANALYSIS II, SECOND SEMESTER, A.Y. 2023 - 2024

+ 2
+···+
(a1 x + b1 )r

printed, reproduced, shared, or sold without permission from the author.


a1 x + b1 (a1 x + b1 )

Copyright protection is governed by R.A. 10372. This work cannot be


By way of illustration, we could write
x3 − x + 1 A B C D E
2 3
= + 2+ + 2
+
x (x − 1) x x x − 1 (x − 1) (x − 1)3
x4 − 2x2 + 4x + 1
Z
■ Example 3.4 Find dx.
x3 − x2 − x + 1
Solution: The first step is to divide. The result of long division is
x4 − 2x2 + 4x + 1 4x
3 2
= x+1+ 3 2
x −x −x+1 x −x −x+1
The second step is to factor the denominator Q(x) = x3 − x2 − x + 1. Since Q(1) = 0, we
know that x − 1 is a factor and we obtain

x3 − x2 − x + 1 = (x − 1)(x2 − 1) = (x − 1)(x − 1)(x + 1) = (x − 1)2 (x + 1)

Since the linear factor x − 1 occurs twice, the partial fraction decomposition is
4x A B C
= + +
(x − 1)2 (x + 1) x − 1 (x − 1)2 x + 1
Multiplying by the least common denominator, (x − 1)2 (x + 1), we get

4x = A(x − 1)(x + 1) + B(x + 1) +C(x − 1)2


= (A +C)x2 + (B − 2C)x + (−A + B +C)

Now we equate coefficients:

A +C = 0
B − 2C = 4
−A + B +C = 0
3.1 Integration of Rational Functions by Partial Fractions 35

Solving, we obtain A = 1, B = 2, and C = −1, so


Z 4
x − 2x2 + 4x + 1
Z  
1 2 1
dx = x+1+ + − dx
x3 − x2 − x + 1 x − 1 (x − 1)2 x + 1
x2 2
= + x + ln |x − 1| − − ln |x + 1| + K
2 x−1
x2 2 x−1
= +x− + ln +K
2 x−1 x+1
Terms of use: This course material is strictly for class use only to students officially
enrolled in MATH 54, ELEMENTARY ANALYSIS II, SECOND SEMESTER, A.Y. 2023 - 2024

3.1.3 Case III: Q(x) contains irreducible quadratic factors, none of which is repeated

printed, reproduced, shared, or sold without permission from the author.


If Q(x) has the factor ax2 + bx + c, where b2 − 4ac < 0, then, the expression for R(x)/Q(x)

Copyright protection is governed by R.A. 10372. This work cannot be


will have a term of the form
Ax + B
ax2 + bx + c
where A and B are constants to be determined. For instance, the function given by f (x) =
x/[(x − 2)(x2 + 1)(x2 + 4)] has a partial fraction decomposition of the form
x A Bx +C Dx + E
= + 2 + 2
(x − 2)(x2 + 1)(x2 + 4) x−2 x +1 x +4

2x2 − x + 4
Z
■ Example 3.5 Evaluate dx.
x3 + 4x
Solution: Since x3 + 4x = x(x2 + 4) can’t be factored further, we write
2x2 − x + 4 A Bx +C
= + 2
x(x2 + 4) x x +4
Multiplying by x(x2 + 4), we have

2x2 − x + 4 = A(x2 + 4) + (Bx +C)x = (A + B)x2 +Cx + 4A

Equating coefficients, we obtain A + B = 2, C = −1, and 4A = 4. Therefore, A = 1, B = 1,


and C = −1 and so,
2x2 − x + 4
Z  
1 x−1
Z
dx = + dx
x3 + 4x x x2 + 4
In order to integrate the second term we split it into two parts:
x−1 x 1
Z Z Z
dx = dx − dx.
x2 + 4 2
x +4 x2 + 4
3.1 Integration of Rational Functions by Partial Fractions 36

We make the substitution u = x2 + 4 in the first of these integrals so that du = 2xdx. We


dx 1 −1  x 
Z
evaluate the second integral by means of the previous result, dx = tan +C.
x2 + a2 a a
Therefore,
2x2 − x + 4 1 x 1
Z Z Z Z
dx = dx + dx − dx
x3 + 4x x 2
x +4 2
x +4
1 1
= ln |x| + ln(x2 + 4) − tan−1 (x/2) + K
Terms of use: This course material is strictly for class use only to students officially

2 2
enrolled in MATH 54, ELEMENTARY ANALYSIS II, SECOND SEMESTER, A.Y. 2023 - 2024

3.1.4 Case IV: Q(x) contains a repeated irreducible quadratic factor

printed, reproduced, shared, or sold without permission from the author.


Copyright protection is governed by R.A. 10372. This work cannot be
If Q(x) has the factor (ax2 + bx + c)r , where b2 − 4ac < 0, then instead of the single partial
fraction, the sum
A1 x + B1 A2 x + B2 Ar x + Br
2
+ 2 2
+···+
ax + bx + c (ax + bx + c) (ax2 + bx + c)r
occurs in the partial fraction decomposition of R(x)/Q(x). Each of the terms can be integrated
by using a substitution or by first completing the square if necessary.

■ Example 3.6 Write out the form of the partial fraction decomposition of the function

x3 + x2 + 1
.
x(x − 1)(x2 + x + 1)(x2 + 1)3
Solution:
x3 + x2 + 1 A B Cx + D Ex + F Gx + H Ix + J
2 2 3
= + + 2 + 2 + 2 2
+ 2
x(x − 1)(x + x + 1)(x + 1) x x − 1 x + x + 1 x + 1 (x + 1) (x + 1)3
1 − x + 2x2 − x3
Z
■ Example 3.7 Evaluate dx.
x(x2 + 1)2
Solution: The form of the partial fraction decomposition is
1 − x + 2x2 − x3 A Bx +C Dx + E
2 2
= + 2 + 2
x(x + 1) x x + 1 (x + 1)2
Multiplying by x(x2 + 1)2 , we have

−x3 + 2x2 − x + 1 = A(x2 + 1)2 + (Bx +C)x(x2 + 1) + (Dx + E)x


= A(x4 + 2x2 + 1) + B(x4 + x2 ) +C(x3 + x) + Dx2 + Ex

= (A + B)x4 +Cx3 + (2A + B + D)x2 + (C + E)x + A

If we equate coefficients, we get the system


3.2 Exercises 37

A + B = 0, C = −1, 2A + B + D = 2, C + E = −1, A=1

which has the solution A = 1, B = −1, C = −1, D = 1, and E = 0. Thus,

1 − x + 2x2 − x3
Z  
1 x+1 x
Z
dx = + + dx
x(x2 + 1)2 x x2 + 1 (x2 + 1)2
dx x dx xdx
Z Z Z Z
Terms of use: This course material is strictly for class use only to students officially

= − 2
dx − 2
+ dx
x x +1 x +1 (x + 1)2
2
enrolled in MATH 54, ELEMENTARY ANALYSIS II, SECOND SEMESTER, A.Y. 2023 - 2024

1 1
= ln |x| − ln(x2 + 1) − tan−1 x − + K.

printed, reproduced, shared, or sold without permission from the author.


2 2
2(x + 1)

Copyright protection is governed by R.A. 10372. This work cannot be


3.2 Exercises
Exercise 3.1 Find the following integrals.
x4
Z
1. dx
x−1
2
Z
2. 2
dx
2x + 3x + 1
x2 + x + 1
Z
3. dx
(x + 1)2 (x + 2)
y
Z
4. dy
(y + 4)(2y − 1)
1
Z
5. 3
dx
x −1
x2 − 3x + 7
Z
6. dx
(x2 − 4x + 6)2
x4 + 9x2 + x + 2
Z
7. dx
x2 + 9
x3 − 2x2 + 2x − 5
Z
8. dx
x4 + 4x2 + 3
Terms of use: This course material is strictly for class use only to students officially
enrolled in MATH 54, ELEMENTARY ANALYSIS II, SECOND SEMESTER, A.Y. 2023 - 2024

IV

4.3
4.2
4.1

Exercises
Improper Integrals with Infinite Discontinuities
Improper Integrals with Infinite Limits of Integration
Week Four

Improper Integrals . . . . . . . . . . . . . . . . . . . 39
Copyright protection is governed by R.A. 10372. This work cannot be
printed, reproduced, shared, or sold without permission from the author.
Terms of use: This course material is strictly for class use only to students officially
enrolled in MATH 54, ELEMENTARY ANALYSIS II, SECOND SEMESTER, A.Y. 2023 - 2024

printed, reproduced, shared, or sold without permission from the author.


Copyright protection is governed by R.A. 10372. This work cannot be
4. Improper Integrals

At the end of this topic you are expected to:


• evaluate an improper integral that has an infinite limit of integration
• evaluate an improper integral that has an infinite discontinuity

In defining the definite integral


Z b
f (x) dx
a
it was assumed that the limits of integration were finite and that the integrand was continu-
ous for every x in the closed interval [a, b]. If either of these conditions are not satisfied, then
the integral is called an improper integral.
Recall also that a function f is said to have an infinite discontinuity if

lim f (x) = +∞ or lim f (x) = −∞


x→c x→c

4.1 Improper Integrals with Infinite Limits of Integration


4.1 Improper Integrals with Infinite Limits of Integration 40

Definition 4.1.1 (Improper Integrals with Infinite Integration Limits)

1. If f is continuous on the interval [a, +∞) , then


Z +∞ Z b
f (x) dx = lim f (x) dx.
a b→+∞ a

2. If f is continuous on the interval (−∞, b] , then


Terms of use: This course material is strictly for class use only to students officially

Z b Z b
f (x) dx = lim f (x) dx.
enrolled in MATH 54, ELEMENTARY ANALYSIS II, SECOND SEMESTER, A.Y. 2023 - 2024

−∞ a→−∞ a

printed, reproduced, shared, or sold without permission from the author.


Copyright protection is governed by R.A. 10372. This work cannot be
3. if f is continuous on the interval (−∞, +∞), then
Z +∞ Z c Z +∞
f (x) dx = f (x) dx + f (x) dx
−∞ −∞ c

where c is any real number.


In the first two cases, the improper integral converges when the limit exists–otherwise,
the improper integral diverges. In the third case, the improper integral on the left
diverges when either of the improper integrals on the right diverges.
Z +∞
dx
■ Example 4.1 Evaluate .
1 x
Solution:
Z +∞ Z b
dx dx
= lim
1 x b→+∞ 1 x
 b
= lim ln x
b→+∞ 1

= lim (ln b − 0)
b→+∞

= +∞.
Z +∞
dx
Therefore, diverges.
1 x
Z +∞
1
Z ∞
−x
■ Example 4.2 Evaluate the foloowing integrals (a) e dx (b) dx
0 0 x2 + 1
4.1 Improper Integrals with Infinite Limits of Integration 41

Solution: (a)
Z +∞ Z b
−x
e dx = lim e−x dx
0 b→+∞ 0
 b
−x
= lim − e
b→+∞ 0

= lim (−e−b + 1)
b→+∞
Terms of use: This course material is strictly for class use only to students officially

= 1.
enrolled in MATH 54, ELEMENTARY ANALYSIS II, SECOND SEMESTER, A.Y. 2023 - 2024

printed, reproduced, shared, or sold without permission from the author.


Z ∞
Therefore, e−x dx is convergent.

Copyright protection is governed by R.A. 10372. This work cannot be


0

(b)
Z +∞ Z b
1 1
2
dx = lim dx
0 x +1 b→+∞ 0 x2 + 1
 b
−1
= lim tan x
b→+∞ 0
−1
= lim tan b
b→+∞
π
= .
2
1
Z ∞
Therefore, dx is convergent.
0 x2 + 1
Z +∞
■ Example 4.3 Evaluate (1 − x)e−x dx
1
Z
Solution: Here, we apply first integration by partsof the indefinite integral (1 − x)e−x dx. Let
u = (1 − x) and dv = e−x dx. Then du = −dx and v = −e−x . Thus,

Z Z
−x −x
(1 − x)e dx = −e (1 − x) − e−x dx

= −e−x + xe−x + e−x +C


= xe−x +C

Now, applying the definition of an improper integral, we have


4.1 Improper Integrals with Infinite Limits of Integration 42

Z +∞  b
−x −x
(1 − x)e dx = lim xe
1 b→+∞ 1

b 1
= lim −
b→+∞ eb e
b 1
= lim b − .
b→+∞ e e
Terms of use: This course material is strictly for class use only to students officially
enrolled in MATH 54, ELEMENTARY ANALYSIS II, SECOND SEMESTER, A.Y. 2023 - 2024

printed, reproduced, shared, or sold without permission from the author.


b ∞

Copyright protection is governed by R.A. 10372. This work cannot be


Since is of the form , we apply L’Hôpital’s Rule.
eb ∞

b 1
lim b
= lim b = 0
b→+∞ e b→+∞ e

Hence,
Z +∞
b 1
(1 − x)e−x dx = lim −
1 b→+∞ eb e
1
= 0−
e
1
=−
e
Z +∞
Therefore, (1 − x)e−x dx is convergent.
1
ex
Z +∞
■ Example 4.4 Evaluate 2x
dx.
−∞ 1 + e
Solution: Note that the given integrand is continuous for all real number x. To evaluate the
integral , we can break it into two parts. We chose c = 0 since it is a convenient value.
ex
Z
We have to solve first for the idefinite integral dx. Let u = ex . Then du = ex dx.
1 + e2x
So,

ex du
Z Z
dx =
1 + (ex )2 1 + u2
= tan−1 u +C
= tan−1 ex +C.

Now, we will evaluate the improper integral according to the definition.


4.2 Improper Integrals with Infinite Discontinuities 43

ex ex ex
Z +∞ Z 0 +∞ Z
dx = dx + dx
−∞ 1 + e2x −∞ 1 + e
2x
0 1 + e2x
ex ex
Z 0 Z b
= lim dx + lim dx
a→−∞ a 1 + e2x b→∞ 0 1 + e2x
 0  b
−1 x −1 x
= lim tan e + lim tan e
a→−∞
a b→+∞ 0
Terms of use: This course material is strictly for class use only to students officially

   
π −1 a −1 b π
= lim − tan e + lim tan e −
enrolled in MATH 54, ELEMENTARY ANALYSIS II, SECOND SEMESTER, A.Y. 2023 - 2024

a→−∞ 4 b→+∞ 4

printed, reproduced, shared, or sold without permission from the author.


π π π

Copyright protection is governed by R.A. 10372. This work cannot be


= −0+ −
4 2 4
π
= .
2
ex
Z +∞
Therefore, dx is convergent.
−∞ 1 + e2x

4.2 Improper Integrals with Infinite Discontinuities

Another basic type of improper intergal is one that has infinite discontinuity at or between the
limits of integration.

Definition 4.2.1 (Improper Integrals with Infinite Discontinuities)

1. If f is continuous on the interval [a, b) and has an infinite discontinuity at b, then


Z b Z c
f (x) dx = lim f (x) dx.
a c→b− a

2. If f is continuous on the interval (a, b] and has an infinite discontinuity at a, then


Z b Z b
f (x) dx = lim f (x) dx.
a c→a+ c

3. if f is continuous on the interval [a, b], except for some c in (a, b), then
Z b Z c Z b
f (x) dx = f (x) dx + f (x) dx.
a a c

In the first two cases, the improper integral converges when the limit exists–otherwise,
4.2 Improper Integrals with Infinite Discontinuities 44

the improper integral diverges. In the third case, the improper integral on the left
diverges when either of the improper integrals on the right diverges.
Z 1
dx
■ Example 4.5 Evaluate √3
.
x0
Solution: The integrand has an infinite discontinuity at x = 0. Thus,
Z 1 Z 1
dx dx
√ = lim √
Terms of use: This course material is strictly for class use only to students officially

3 3
0 x b→0+ b x
3 2/3 1
 
enrolled in MATH 54, ELEMENTARY ANALYSIS II, SECOND SEMESTER, A.Y. 2023 - 2024

= lim x

printed, reproduced, shared, or sold without permission from the author.


b→0+ 2 b

Copyright protection is governed by R.A. 10372. This work cannot be


3 2/3

= lim 1−b
b→0+ 2
3
= .
2
Z 1
dx
Therefore, √
3
is convergent.
0 x
Z 2
dx
■ Example 4.6 Evaluate .
x3 0
Solution: The integrand has an infinite discontinuity at x = 0. Thus,
Z 2 Z 2
dx dx
= lim
0 x3 b→0+ b x3
1 2
 
= lim − 2
b→0+ 2x b
 
1 1
= lim − + 2
b→0+ 8 2b
= +∞.
Z 2
dx
Therefore, is divergent.
0 x3
Z 2
dx
■ Example 4.7 Evaluate .
x3 −1
Solution: The integral is improper because the integrand has an infinite discontinuity at the
interior point x = 0. Now, we can write the given integral as
Z 2 Z 0 Z 2
dx dx dx
= + .
−1 x3 −1 x3 0 x3
Notice that the second integral on the right is divergent (from Example 4.6).
4.3 Exercises 45
Z 2
dx
Therefore, is also divergent.
x3 −1
There might be instances when you fail recognize that a particular integral is improper just
like in Example 4.7. If you are not careful, you will have obtained a result as illusrated below
1 2
Z 2  
dx 1 1 3
3
= − 2 =− + = which is not correct.
−1 x 2x −1 8 2 8
There are also integrals which are doubly improper. The next example has this property
Terms of use: This course material is strictly for class use only to students officially

– one limit is infinite and the integrand has an infinite discontinuity at the other limit of
enrolled in MATH 54, ELEMENTARY ANALYSIS II, SECOND SEMESTER, A.Y. 2023 - 2024

printed, reproduced, shared, or sold without permission from the author.


integration.

Copyright protection is governed by R.A. 10372. This work cannot be


Z +∞
dx
■ Example 4.8 Evaluate √ .
0 x(x + 1)
Solution: To evaluate this integral, we select a point where we split the given integral. Suppose
we split the integral at x = 1. Then

Z +∞ Z 1 +∞
dx dx dx
Z
√ = √ + √
0 x(x + 1) 0 x(x + 1) 1 x(x + 1)
Z 1 Z c
dx dx
= lim √ + lim √
b→0+ b x(x + 1) c→+∞ 1 x(x + 1)
1 c
−1 √ −1 √
 
= lim 2 tan x + lim 2 tan x
b→0+ b
c→+∞
1
√ −1 √
   
−1 −1 −1
= lim 2 tan (1) − 2 tan b + lim 2 tan c − 2 tan (1)
b→0+ c→+∞
π  π  π 
=2 −0+2 −2
4 2 4
= π.
Z +∞
dx
Therefore, √ is convergent.
0 x(x + 1)

4.3 Exercises
Exercise 4.1 Determine whether the given improper integral is convergent or divergent.
Z 1
1. ez dz
Z−∞
+∞ dy
2. √
5 y−1
Terms of use: This course material is strictly for class use only to students officially
enrolled in MATH 54, ELEMENTARY ANALYSIS II, SECOND SEMESTER, A.Y. 2023 - 2024

6.
5.
4.
3.

1
Z 1
4.3 Exercises

Z013/5
dx

p
dx
Z0+∞ 1 − x
2
dx

dy
x2 − 2x − 3
Z24 x x − 4

y2 − 1
46

Copyright protection is governed by R.A. 10372. This work cannot be


printed, reproduced, shared, or sold without permission from the author.
Terms of use: This course material is strictly for class use only to students officially
enrolled in MATH 54, ELEMENTARY ANALYSIS II, SECOND SEMESTER, A.Y. 2023 - 2024

5.3
5.2
5.1
Series
Exercises
Sequences
Week Five

Infinite Sequence and Series . . . . . . . . . 48


Copyright protection is governed by R.A. 10372. This work cannot be
printed, reproduced, shared, or sold without permission from the author.
Terms of use: This course material is strictly for class use only to students officially
enrolled in MATH 54, ELEMENTARY ANALYSIS II, SECOND SEMESTER, A.Y. 2023 - 2024

printed, reproduced, shared, or sold without permission from the author.


Copyright protection is governed by R.A. 10372. This work cannot be
5. Infinite Sequence and Series

At the end of this topic you are expected to:


• evaluate the limit of a sequence
• evaluate the sum of some infinite series

5.1 Sequences

A sequence can be thought of as a list of numbers written in a definite order:

a1 , a2 , a3 , a4 , . . . , an , . . .

The number a1 is called the first term, a2 is the second term, and in general an is the nth term.
We will deal exclusively with infinite sequences and so each term an will have a successor an+1 .

Notice that for every positive integer n there is a corresponding number an and so a sequence
can be defined as afuction whose domain is the set of positive integers. But we usually write
an instead of the function notation f (n) for the value of the function at the number n.
5.1 Sequences 49

NOTATION The sequence {a1 , a2 , a3 , . . .} is also denoted by

an or {an }∞
n=1

■ Example 5.1 Some sequences can be defined by giving a formula for the nth term. In the

following eamples we give three descriptions of the sequence: one by using the preceeding
notation, another by using the defining formula, and a third by writing out the terms of se-
Terms of use: This course material is strictly for class use only to students officially

quences. Notice that n doesn’t have to start at 1.


enrolled in MATH 54, ELEMENTARY ANALYSIS II, SECOND SEMESTER, A.Y. 2023 - 2024

printed, reproduced, shared, or sold without permission from the author.


 ∞  

Copyright protection is governed by R.A. 10372. This work cannot be


n n 1 2 3 4 n
(a) an = , , , ,..., ,...
 n + 1n n=1 ∞ n+1 2 3 4 5 n + 1
(−1)n (n + 1) (−1)n (n + 1)
 
(−1) (n + 1) 2 3 −4 5
(b) a n = − , , , ,..., ,...
3n n=1 3n n 3 √ 9 27 81 3n o
√ ∞ √ √ √
(c) n − 3 n=3 an = n − 3, n ≥ 3 0, 1, 2, 3, . . . , n − 3, . . .
( √ )
n nπ o∞ nπ 3 1 nπ
(d) cos an = cos , n ≥ 0 1, , , 0, . . . , cos , . . .
6 n=0 6 2 2 6
■ Example 5.2 Find a formula for the general term an of the sequence

 
3 4 5 6 7
,− , ,− , ,...
5 25 125 625 3125
assuming that the pattern of the first few terms continues.

Solution: We are given that


3 4 5 6 7
a1 = a2 = − a3 = a4 = − a5 =
5 25 125 625 3125
Notice that the numerators of these fractions start with 3 and increase by 1 whenever we
go to the next term. The second term has numerator 4, the third term has numerator 5; in
general, the nth term will have numerator n + 2. The denominators are the powers of 5, so an
has denominator 5n . The signs of the terms are alternately positive and negative, so we need to
multiply by a power of −1. Here we want to start with a positive term and so we use (−1)n−1
or (−1)n+1 . Therefore,
n+2
an = (−1)n−1 .
5n
■ Example 5.3 Here are some sequences that don’t have a simple defining equation.

(a) The sequence {pn } where pn is the population of the world as of January 1 in the year n.
5.1 Sequences 50

(b) If we let an be the digit in the nth decimal plate of the number e, then {an } is a well-
defined sequence whose first few terms are

{7, 1, 8, 2, 8, 1, 8, 2, 8, 4, 5, . . .}

(c) The Fibonacci Sequence { fn } is defined recursively by the conditions


Terms of use: This course material is strictly for class use only to students officially

f1 = 1 f2 = 1 fn = fn−1 + fn−2 n≥3


enrolled in MATH 54, ELEMENTARY ANALYSIS II, SECOND SEMESTER, A.Y. 2023 - 2024

Each term is the sum of the two preceding terms. The first few terms are

printed, reproduced, shared, or sold without permission from the author.


Copyright protection is governed by R.A. 10372. This work cannot be
{1, 1, 2, 3, 5, 8, 13, 21, . . .}

This sequence arose when the 13th-century Italian mathematician known as Fibonacci
solved a problem concerning the breeding of rabbits.
n
A sequence such as the one in Example 5.1(a), an = , can be pictured either by
n+1
plotting its terms on a number line, or by plotting its graph. Note that, since a sequence is a
function whose domain is the set of positive integers, its graph consists of isolated points with
coordinates
(1, a1 ) (2, a2 ) (3, a3 ) ... (n, an ) ...
n
It appears that the terms of the sequence an = are approaching 1 as n becomes large. In
n+1
fact, the difference
n 1
1− =
n+1 n+1
can be made small as we like by taking n sufficiently large. We indicate this by writing

n
lim =1
n→∞ n + 1

In general, the notation


lim an = L
n→∞

means that means that the terms of the sequence {an } approach L as n becomes large. Notice
that the following definition of the limit of a sequence is very similar to the definition of a
limit of a function at infinity on previous discussion.
5.1 Sequences 51

Definition 5.1.1 A sequence {an } has the limit L and we write

lim an = L or an → L as n→∞
n→∞

if we can make the terms an as close to L as we like by taking n sufficiently large. If


limn→∞ an exists, we say the sequence converges (or is convergent). Otherwise, we say
the sequence diverges (or is divergent).
Terms of use: This course material is strictly for class use only to students officially
enrolled in MATH 54, ELEMENTARY ANALYSIS II, SECOND SEMESTER, A.Y. 2023 - 2024

A more precise version of the above definition is as follows.

printed, reproduced, shared, or sold without permission from the author.


Copyright protection is governed by R.A. 10372. This work cannot be
Definition 5.1.2 A sequence {an } has the limit L and we write

lim an = L or an → L as n→∞
n→∞

if for every ε > 0 there is a corresponding integer N such that

if n>N then |an − L| < ε

Note that the only difference between lim an = L and lim f (x) = L is that n is required to
n→∞ x→∞
be an integer. Thus we have the following theorem.

Theorem 5.1.1 If lim f (x) = L and f (n) = an when n is an integer, then lim an = L.
x→∞ n→∞
 
1
In particular, since we know that lim = 0 when r > 0 we have
x→∞ xr

1
lim =0 if r > 0.
n→∞ nr

Remark 5.1.1 If an becomes large as n becomes large, we use the notation lim an = ∞. The
n→∞
following precise definition is similar to the definition of limits at infinity.

Definition 5.1.3 lim an = ∞ means that for every positive number M there is an integer N
n→∞
such that
if n>N then an > M

If limn→∞ an = ∞, then the sequence {an } is divergent but in a special way. We say that
5.1 Sequences 52

{an } diverges to ∞.
The Limit Laws discussed in Math 53 also hold for the limits of sequences and theirproofs
are similar.
If {an } and {bn } are convergent sequences and c is constant, then
Terms of use: This course material is strictly for class use only to students officially

lim (an + bn ) = lim an + lim bn


n→∞ n→∞ n→∞
enrolled in MATH 54, ELEMENTARY ANALYSIS II, SECOND SEMESTER, A.Y. 2023 - 2024

lim (an − bn ) = lim an − lim bn

printed, reproduced, shared, or sold without permission from the author.


n→∞ n→∞ n→∞

Copyright protection is governed by R.A. 10372. This work cannot be


lim can = c lim an lim c = c
n→∞ n→∞ n→∞

lim (an bn ) = lim an · lim bn


n→∞ n→∞ n→∞
an limn→∞ an
lim = if lim bn ̸= 0
n→∞ bn limn→∞ bn n→∞
h ip
lim anp = lim an if p > 0 and an > 0
n→∞ n→∞

The squeeze theorem can also be adapted for sequences as follows.

Theorem 5.1.2 If an ≤ bn ≤ cn for n ≥ 0 and lim an = lim cn = L, then lim bn = L.


n→∞ n→∞ n→∞

Another useful fact about limits of sequences is given by the following theorem, whose
proof is left as exercise.
Theorem 5.1.3 If lim |an | = 0, then lim an = 0.
n→∞ n→∞

n
■ Example 5.4 Find lim .
n→∞ n + 1

Solution: The method is similar to the one we used previously: Divide numerator and de-
nominator by the highest power of n that occurs in the denominator and then use the limit
laws.
5.1 Sequences 53

n 1 lim 1
n→∞
lim = lim =
n→∞ n + 1 n→∞ 1 1
1+ lim 1 + lim
n n→∞ n→∞ n
1
= = 1.
1+0
n
Example 5.5 Is the sequence an = √ convergent or divergent?
Terms of use: This course material is strictly for class use only to students officially


10 + n
enrolled in MATH 54, ELEMENTARY ANALYSIS II, SECOND SEMESTER, A.Y. 2023 - 2024

Solution: We divide numerator and denominator by n:

printed, reproduced, shared, or sold without permission from the author.


Copyright protection is governed by R.A. 10372. This work cannot be
n 1
lim √ = lim r =∞
n→∞ 10 + n n→∞ 10 1
+
n2 n
because the numerator is constant and the denominator approaches 0. So {an } is divergent.
ln n
■ Example 5.6 Calculate lim .
n→∞ n

Solution: Notice that both numerator and denominator approach infinity as n → ∞. We can’t
apply L’Hopital’s Rule directly because it applies not to sequences but to functions of a real
ln x
variable. However, we can apply L’Hopital’s Rule to the related function f (x) = and
x
obtain
1
ln x
lim = lim x = 0
x→∞ x x→∞ 1

Therefore, by Theorem 5.1.1, we have


ln n
lim = 0.
n→∞ n

■ Example 5.7 Determine whether the sequence an = (−1)n is convergent or divergent.

Solution: If we write out the terms of the sequence, we obtain

{−1, 1, −1, 1, −1, 1, −1, . . .}

The graph of this sequence is shown in Figure 8. Since the terms oscillate between 1 and -1
infinitely often, an does not approach any number. Thus lim (−1)n does not exist; that is, the
n→∞
sequence {(−1)n }
is divergent.
(−1)n
■ Example 5.8 Evaluate lim if it exists.
n→∞ n
5.1 Sequences 54

Solution: We first calculate the limit of the absolute value:

(−1)n 1
lim = lim = 0.
n→∞ n n→∞ n

Therefore, by Theorem 5.1.3,


(−1)n
lim = 0.
n→∞ n
Terms of use: This course material is strictly for class use only to students officially

The following theorem says that if we apply a continuous function to the terms of a
enrolled in MATH 54, ELEMENTARY ANALYSIS II, SECOND SEMESTER, A.Y. 2023 - 2024

convergent sequence, the result is also convergent.

printed, reproduced, shared, or sold without permission from the author.


Copyright protection is governed by R.A. 10372. This work cannot be
Theorem 5.1.4 If lim an = L and the function f is continuous at L, then
n→∞

lim f (an ) = f (L).


n→∞

π
■ Example 5.9 Find lim sin .
n→∞ n
Solution: Because the sine function is continuous at 0, Theorem 5.1.4 enables us to write
π  π
lim sin = sin lim = sin 0 = 0.
n→∞ n n→∞ n

n!
■ Example 5.10 Discuss the convergence of the sequence an = n .
n
Solution: Both numerator and denominator approach infinity as n → ∞ but here we have
no corresponding function for use with L’Hopital’s Rule (x! is not defined when x is not an
integer). Let’s write out a few terms to get a feeling for what happens to an as n gets large:
1·2 1·2·3
a1 = 1 a2 = a3 =
2·2 3·3·3

1·2·3·····n
an = (*)
n·n·n·····n
It appears from these expressions that the terms are decreasing and perhaps approach 0. To
confirm this, observe from Equation * that
 
1 2·3·····n
an =
n n·n·····n
Notice that the expression in parentheses is at most 1 because the numerator is less than (or
equal to) the denominator. So
1
0 < an ≤
n
5.1 Sequences 55

1
We know that → 0 as n → ∞. Therefore an → 0 as n → ∞ by the Squeeze Theorem.
n

The next theorem will give us an answer as to what values of r will the sequence {rn } be
convergent and its proof will be left as an exercise.

Theorem 5.1.5 The sequence {rn } is convergent if −1 < r ≤ 1 and divergent for all other
Terms of use: This course material is strictly for class use only to students officially

values of r. Moreover,
enrolled in MATH 54, ELEMENTARY ANALYSIS II, SECOND SEMESTER, A.Y. 2023 - 2024

printed, reproduced, shared, or sold without permission from the author.


0 if −1 < r < 1

Copyright protection is governed by R.A. 10372. This work cannot be


lim rn =
n→∞ 1 if r = 1

Definition 5.1.4 A sequence {an } is called increasing if an < an+1 for all n ≥ 1, that is,

a1 < a2 < a3 < . . .. It is called decreasing if an > an+1 for all n ≥ 1. A sequence is
monotonic if it is either increasing or decreasing.
 
3
■ Example 5.11 The sequence is decreasing because
n+5
3 3 3
> =
n + 5 (n + 1) + 5 n + 6
and so an > an+1 for all n ≥ 1.
n
■ Example 5.12 Show that the sequence an = 2 is decreasing.
n +1
Solution: We must show that an > an+1 , that is
n+1 n
2
< 2
(n + 1) + 1 n + 1
This inequality is equivalent to the one we get by cross-multiplication:
n+1 n
2
< 2 ⇔ (n + 1)(n2 + 1) < n[(n + 1)2 + 1]
(n + 1) + 1 n + 1
⇔ n3 + n2 + n + 1 < n3 + 2n2 + 2n
⇔ 1 < n2 + n

Since n ≥ 1, we know that 1 < n2 + n is true. Therefore an > an+1 and so {an } is decreasing.
5.2 Series 56

x
Solution: (Alternative Solution) Consider the function f (x) = :
x2 + 1
x2 + 1 − 2x2 1 − x2
f ′ (x) = = <0 whenever x2 > 1
(x2 + 1)2 (x2 + 1)2
Thus, f is decreasing on (1, ∞) and so f (n) > f (n + 1). Therefore, {an } is decreasing.

Definition 5.1.5 A sequence {an } is bounded above if there is a number M such that
Terms of use: This course material is strictly for class use only to students officially
enrolled in MATH 54, ELEMENTARY ANALYSIS II, SECOND SEMESTER, A.Y. 2023 - 2024

an ≤ M for all n ≥ 1

printed, reproduced, shared, or sold without permission from the author.


Copyright protection is governed by R.A. 10372. This work cannot be
A sequence {an } is bounded below if there is a number m such that

m ≤ an for all n ≥ 1

If it is bounded above and below, then {an } is a bounded sequence.

For instance, the sequence an = n is bounded below (an > 0) but not above. The sequence
n
an = is bounded because 0 < an < 1 for all n.
n+1
Theorem 5.1.6 Monotonic Sequence Theorem

Every bounded, monotonic sequence is convergent.

We will not show the proof of this theorem but the its proof of shows that a sequence that is
increasing and bounded above is convergent. (Likewise, a decreasing sequence that is bounded
below is convergent.) This fact is used many times in dealing with infinite series.

5.2 Series

What do we mean when we express a number as an infinite decimal? For instance, what does
it mean to write

π = 3.14159265358979323846264338327950288 . . .

The convention behind our decimal notation is that any number can be written as an infinite
sum. Here it means that
1 4 1 5 9 2 6 5
π = 3+ + 2 + 3 + 4 + 5 + 6 + 7 + 8 +...
10 10 10 10 10 10 10 10
5.2 Series 57

where the three dots (. . .) indicate that the sum continues forever, and the more terms we add,
the closer we get to the actual value of π.
In general, if we try to add the terms of an infinite sequence {an }∞
n=1

a1 + a2 + a3 + . . . + an + . . . (5.1)

which is called an infinite series (or just a series) and is denoted, for short, by the symbol
Terms of use: This course material is strictly for class use only to students officially


∑ an or ∑ an
enrolled in MATH 54, ELEMENTARY ANALYSIS II, SECOND SEMESTER, A.Y. 2023 - 2024

n=1

printed, reproduced, shared, or sold without permission from the author.


Copyright protection is governed by R.A. 10372. This work cannot be
Does it make sense to talk about the sum of infinitely many terms?
We consider the partial sums

s1 = a1
s2 = a1 + a2

s3 = a1 + a2 + a3

s4 = a1 + a2 + a3 + a4

and, in general,
n
sn = a1 + a2 + a3 + . . . + an = ∑ ai
i=1
These partial sums form a new sequence {sn }, which may or may not have a limit. If
lim sn = s exists (as a finite number), then, as in the preceesing example, we call it the sum of
n→∞
the infinite series ∑ an .

Definition 5.2.1 Given a series ∑ an = a1 + a2 + . . . , let sn denote its nth partial sum:
n=1
n
sn = a1 + a2 + a3 + . . . + an = ∑ ai
i=1

If the sequence {sn } is convergent and ∑ sn = s exists as a real number, then the series
n=1
∑ an is called convergent and we write

a1 + a2 + · · · + an + · · · = s or ∑ an = s
n=1

The number s is called the sum of the series. If the sequence {sn } is divergent, then the
5.2 Series 58

series is called divergent.

Thus the sum of a series is the limit of the sequence of partial sums. So when we write

∑ an = s, we mean that by adding sufficiently many terms of the series we can get as close
n=1
as we like to the number s. Notice that
Terms of use: This course material is strictly for class use only to students officially

∞ n
∑ an = limn→∞
∑ ai .
enrolled in MATH 54, ELEMENTARY ANALYSIS II, SECOND SEMESTER, A.Y. 2023 - 2024

n=1 i=1

printed, reproduced, shared, or sold without permission from the author.


Copyright protection is governed by R.A. 10372. This work cannot be


■ Example 5.13 Suppose that we know that the sum of the first n terms of the series ∑ an is
n=1

2n
sn = a1 + a2 + · · · + an =
3n + 5
Then the sum of the series is the limit of the sequence {sn }:

2n 2 2
∑ an = n→∞
lim sn = lim
n→∞ 3n + 5
= lim
n→∞ 5
= .
3
n=1 3+
n
■ Example 5.14 An important example of an infinite series is the geometric series

a + ar + ar2 + ar3 + · · · + arn−1 + · · · = ∑ arn−1 a ̸= 0
n=1

Each term is obtained from the preceeding one by multiplying it by the commmon ratio r.

Remark 5.2.1 The geometric series



a + ar + ar2 + ar3 + · · · + arn−1 + · · · = ∑ arn−1
n=1

is convergent if |r| < 1 and its sum is



a
∑ arn−1 = 1 − r , |r| < 1
n=1

If |r| ≥ 1, the geometric series is divergent.

■ Example 5.15 Find the sum of the geometric series

10 20 40
5− + − +··· .
3 9 27
5.2 Series 59

2 2
Solution: The first term is a = 5 and thecommon ratio is r = − . Since |r| = < 1, the series
3 3
is convergent and its sum is
10 20 40 5
5− + − +··· =   = 3.
3 9 27 2
1− −
3

Example 5.16 Is the series ∑ 22n31−n convergent or divergent?
Terms of use: This course material is strictly for class use only to students officially

n=1
enrolled in MATH 54, ELEMENTARY ANALYSIS II, SECOND SEMESTER, A.Y. 2023 - 2024

Solution: Let’s rewrite the nth term of the series in the form arn−1 :

printed, reproduced, shared, or sold without permission from the author.


Copyright protection is governed by R.A. 10372. This work cannot be
∞  n−1

2n 1−n

2 n −(n−1)

4n 4
∑2 3 = ∑ (2 ) 3 = ∑ n−1 = ∑ 4
n=1 n=1 n=1 3 n=1 3
4
We recognize this series as a geometric series with a = 4 and r = . Since r > 1, the series
3
diverges.

■ Example 5.17

1
1 + x + x2 + x3 + x4 + · · · = ∑ xn = 1 − x , |x| < 1.
n=0

1
■ Example 5.18 Show that the series ∑ n(n + 1) is convergent and find its sum.
n=1
Solution: This is not a geometric series, so we go back to the definition of a convergent series
and compute the partial sums.
n
1 1 1 1 1
sn = ∑ = + + +···+
i=1 i(i + 1) 1·2 2·3 3·4 n · (n + 1)
We can simplify this expression if we use the partial fraction decomposition

1 1 1
= −
i(i + 1) i i+1
Thus we have
n n  
1 1 1
sn = ∑ =∑ −
i=1 i(i + 1) i=1 i i+1
       
1 1 1 1 1 1 1
= 1− + − + − +···+ −
2 2 3 3 4 n n+1
1
= 1−
n+1
5.2 Series 60

and so  
1
lim sn = lim 1 − = 1 − 0 = 1.
n→∞ n→∞ n+1
■ Example 5.19 The harmonic series

1 1 1 1
∑ n = 1+ 2 + 3 + 4 +···
n=1
Terms of use: This course material is strictly for class use only to students officially

is divergent.
enrolled in MATH 54, ELEMENTARY ANALYSIS II, SECOND SEMESTER, A.Y. 2023 - 2024

printed, reproduced, shared, or sold without permission from the author.


Theorem 5.2.1 If the series lim an = 0.
∑ an is convergent, then n→∞

Copyright protection is governed by R.A. 10372. This work cannot be


n=1

The converse of Theorem 5.2.1 is not true in general. If lim an = 0, we cannot conclude
n→∞
that ∑ an is convergent. Observe that for the harmonic series ∑ 1/n we have an = 1/n → 0 as
n → ∞, but the harmonic series is divergent.

Theorem 5.2.2 Test for Divergence



If lim an does not exist or if lim an ̸= 0, then the series ∑ an is divergent.
n→∞ n→∞
n=1


n2
■ Example 5.20 Show that the series ∑ 2 diverges.
n=1 5n + 4
Solution:
n2 1
lim an = lim 2
= ̸= 0
n→∞ n→∞ 5n + 4 5
So the series diverges by the test for divergence.

Theorem 5.2.3 If ∑ an and ∑ bn are convergent series, then so are the series ∑ can (where c

is a constant), ∑(an + bn ), ∑(an − bn ), and


∞ ∞
(i) ∑ can = c ∑ an
n=1 n=1
∞ ∞ ∞
(ii) ∑ (an + bn) = ∑ an + ∑ bn
n=1 n=1 n=1
∞ ∞ ∞
(iii) ∑ (an − bn) = ∑ an − ∑ bn
n=1 n=1 n=1
5.3 Exercises 61

∞  
3 1
■ Example 5.21 Find the sum of the series ∑ + .
n=1 n(n + 1) 2n

1 1 1
Solution: The series ∑ 2n is a geometric series with a = 2 and r = 2 , so
n=1
∞ 1
1 2
∑ n 1 − 1 = 1.
=
n=1 2 2
Terms of use: This course material is strictly for class use only to students officially

On the other hand (by previous example),


enrolled in MATH 54, ELEMENTARY ANALYSIS II, SECOND SEMESTER, A.Y. 2023 - 2024


1
∑ n(n + 1) = 1

printed, reproduced, shared, or sold without permission from the author.


Copyright protection is governed by R.A. 10372. This work cannot be
n=1
So by Theorem 5.2.3, the given series is convergent and
∞   ∞ ∞
3 1 1 1
∑ n(n + 1) 2n + = 3 ∑ n(n + 1) ∑ 2n = 3 · 1 + 1 = 4.
+
n=1 n=1 n=1
Remark 5.2.2 A finite number of terms doesn’t affect the convergence or divergence of a series.

For instance, suppose that we were able to show that the series

n
∑ n3 + 1
n=4
is convergent. Since
∞ ∞
n 1 2 3 n
= + + +
∑ n3 + 1 2 9 28 ∑ n3 + 1
n=1 n=4

n
it follows that the entire series ∑ n3 + 1 is convergent. Similarly, if it is known that the series
n=1

∑ an converges, then the full series
n=N+1
∞ N ∞
∑ an = ∑ an + ∑ an
n=1 n=1 n=N+1
is also convergent.

5.3 Exercises
Exercise 5.1 Do as directed.

1. Determine whether the given sequence is increasing, decreasing or not monotonic. Is


the sequence bounded? Explain your answer.
1
(a) an =
2n + 3
5.3 Exercises 62

1−n
(b) an =
2+n
(−1)n
(c) an = 2 + .
n
2. Explain the difference between
n n
(a) ∑ ai and ∑ aj
i=1 j=1
n n
(b) ∑ ai and ∑ a j.
Terms of use: This course material is strictly for class use only to students officially

i=1 i=1
3. Determine whether the given series is convergent or divergent. If it is convergent, find
enrolled in MATH 54, ELEMENTARY ANALYSIS II, SECOND SEMESTER, A.Y. 2023 - 2024

printed, reproduced, shared, or sold without permission from the author.


its sum.

Copyright protection is governed by R.A. 10372. This work cannot be



(a) ∑ [(−0.2)n + (0.6)n−1]
n=1
∞ n
2 + 4n
(b) ∑
n=1 en
∞ √ −k
(c) ∑( 2) .
k=0
4. Use geometric series to express the following as a ratio of two integers
(a) 0.8
(b) 10.135.
Terms of use: This course material is strictly for class use only to students officially
enrolled in MATH 54, ELEMENTARY ANALYSIS II, SECOND SEMESTER, A.Y. 2023 - 2024

VI
6

6.4
6.3
6.2
6.1

Exercises
The Integral Test

Alternating Series
The Comparison Tests
Week Six

Infinite Sequence and Series . . . . . . . . . . 64


Copyright protection is governed by R.A. 10372. This work cannot be
printed, reproduced, shared, or sold without permission from the author.
Terms of use: This course material is strictly for class use only to students officially
enrolled in MATH 54, ELEMENTARY ANALYSIS II, SECOND SEMESTER, A.Y. 2023 - 2024

printed, reproduced, shared, or sold without permission from the author.


Copyright protection is governed by R.A. 10372. This work cannot be
6. Infinite Sequence and Series

At the end of this topic you are expected to:


• determine the convergence or divergence of a series using integral test
• determine the convergence or divergence of a series using comparison tests
• identify whether an alternating series is convergent or divergent

6.1 The Integral Test

In general, it is difficult to find the exact sum of a series. We were able to accomplish this
for geometric series and the series ∑ 1/[n(n + 1)] because in each of those cases we could
find a simple formula for the nth partial sum sn . But usually it isn’t easy to discover such
a formula. Therefore, in the next few sections, we develop several tests that enable us to
determine whether a series is convergent or divergent without explicitly finding its sum. (In
some cases, however, our methods will enable us to find good estimates of the sum.) Our first
test involves improper integrals.
6.1 The Integral Test 65

Theorem 6.1.1 The Integral Test

Suppose f is a continuous, positive, decreasing function on [1, ∞) and let an = f (n). Then
∞ Z ∞
the series ∑ an is convergent if and only if the improper integral f (x) dx is convergent.
n=1 1
In other words:
Z ∞ ∞
1. If f (x) dx is convergent, then ∑ an is convergent.
1 n=1
Terms of use: This course material is strictly for class use only to students officially

Z ∞ ∞
2. If f (x) dx is divergent, then ∑ an is divergent.
enrolled in MATH 54, ELEMENTARY ANALYSIS II, SECOND SEMESTER, A.Y. 2023 - 2024

1 n=1

printed, reproduced, shared, or sold without permission from the author.


Copyright protection is governed by R.A. 10372. This work cannot be
Remark 6.1.1 When we use the Integral Test, it is not necessary to start the series or the

integral at n = 1. For instance, in testing the series



1
∑ (n − 3)2 ,
n=4
we use
1
Z ∞
dx.
4 (x − 3)2
Also, it is not necessary that f be always decreasing. What is important is that f be ultimately

decreasing, that is, decreasing for x larger than some number N. Then ∑ an is convergent, so
n=N

∑ an is convergent.
n=1

1
■ Example 6.1 Test the series ∑ n2 + 1 for convergence or divergence.
n=1
Solution: The function f (x) = 1/(x2 + 1) is continuous, positive, and decreasing on [1, ∞) so
we use the Integral Test:
Z t
1 1
Z ∞
2
dx = lim dx
1 x +1 t→∞ 1 x2 + 1
t
= lim tan−1 x
t→∞ 1
 π
= lim tan−1 t −
t→∞ 4
π π
= −
2 4
π
=
4
6.1 The Integral Test 66


1 1
Z ∞
Thus, 2
dx is a convergent integral and so, by the Integral Test, the series ∑ 2
1 x +1 n=1 n + 1
is convergent.

1
■ Example 6.2 For what values of p is the series
∑ n p convergent?
n=1
Solution: If p < 0, then lim (1/n p ) = ∞. If p = 0, then lim (1/n p ) = 1. In either case
n→∞ n→∞
lim (1/n p ) ̸= 0, so the given series diverges by the Test for Divergence.
Terms of use: This course material is strictly for class use only to students officially

n→∞
If p > 0, then the function f (x) = 1/x p is clearly continuous, positive, and decreasing on
enrolled in MATH 54, ELEMENTARY ANALYSIS II, SECOND SEMESTER, A.Y. 2023 - 2024

printed, reproduced, shared, or sold without permission from the author.


[1, ∞). We found that

Copyright protection is governed by R.A. 10372. This work cannot be


1
Z ∞
dx converges if p > 1 and diverges if p ≤ 1.
1 xp
It follows from the Integral Test that the series ∑ 1/n p converges if p > 1 and diverges if
0 < p ≤ 1. (For p = 1, this series is the harmonic series).
The series in the above example is called the p-series. It is important in the rest of this
chapter.

1
Theorem 6.1.2 The p-series ∑ n p is convergent if p > 1 and divergent if p ≤ 1.
n=1

■ Example 6.3 1. The series



1 1 1 1 1
∑ n3 = 13 + 23 + 33 + 43 + · · ·
n=1

is convergent because it is a p-series with p = 3 > 1.


2. The series
∞ ∞
1 1 1 1 1
∑ n1/3 = ∑ √3 n = 1 + √3 2 + √3 3 + √3 4 + · · ·
n=1 n=1

1
is divergent because it is a p-series with p =
< 1.
3
Remark 6.1.2 We should not infer from the Integral Test that the sum of the series is equal to

the value of the integral. In fact,



1 π2 1
Z ∞
∑ n2 = 6 whereas
1 x2
dx = 1
n=1

Therefore, in general,
6.2 The Comparison Tests 67

∞ Z ∞
∑ an ̸= 1
f (x)dx.
n=1

ln n
■ Example 6.4 Determine whether the series ∑ converges or diverges.
n=1 n
Solution: The function f (x) = (ln n)/x is positive and continuous for x > 1 because the
logarithm function is continuous. But it is not obvious whether or not f is decreasing, so we
Terms of use: This course material is strictly for class use only to students officially

compute its derivative:


enrolled in MATH 54, ELEMENTARY ANALYSIS II, SECOND SEMESTER, A.Y. 2023 - 2024

(1/x)x − ln x 1 − ln x
f ′ (x) = =

printed, reproduced, shared, or sold without permission from the author.


x2 x2

Copyright protection is governed by R.A. 10372. This work cannot be


Thus f ′ (x) < 0 when ln x > 1, that is, when x > e. It follows that f is decreasing when x > e
and so we can apply the Integral Test:
Z t
ln x ln x
Z ∞
dx = lim dx
1 x x
t→∞ 1
t
(ln x)2
= lim
t→∞ 2 1
(lnt)2
= lim =∞
t→∞ 2

ln n
Since this improper integral is divergent, the series ∑ is also divergent by the Integral
n=1 n
Test.

6.2 The Comparison Tests

The idea in the comparison test is to compare a given series from a series that is known to
converge or diverge. For instance,

1
∑ 2n + 1
n=1
would make us remember the series

1
∑ 2n
n=1
which is a geometric series a = 1/2 and r = 1/2 and is therefore convergent. Thus, we can
consider the second series as our comparison series. Comparing their respective nth terms we
have,
1 1
< ,
2n + 1 2n
6.2 The Comparison Tests 68

for all n ≥ 1. This means also that


∞ ∞
1 1
∑ 2n + 1 < ∑ 2n .
n=1 n=1

We all know that the series on the right is equal to 1. Hence,



1
∑ 2n + 1 < 1.
n=1
Terms of use: This course material is strictly for class use only to students officially

This implies that the given series is convergent since its sum is less than 1.
enrolled in MATH 54, ELEMENTARY ANALYSIS II, SECOND SEMESTER, A.Y. 2023 - 2024

printed, reproduced, shared, or sold without permission from the author.


Copyright protection is governed by R.A. 10372. This work cannot be
On the other hand, if a given series is greater than a series which is known to diverge, then
the given series must also diverge. The following theorem will give the general rule of the
above statements.

Theorem 6.2.1 The Comparison Test

Suppose that ∑ an and ∑ bn are series with positive terms.


(i) If ∑ bn is convergent and an ≤ bn for all n, then ∑ an is also convergent.
(ii) If ∑ bn is divergent and an ≥ bn for all n, then ∑ an is also divergent.

5
■ Example 6.5 Determine whether the series ∑ 2n2 + 4n + 3 converges or diverges.
n=1

5 5 ∞ 1
Solution: We can compare the given series with =
∑ 2 2 ∑ n2 which is a convergent,
n=1 2n n=1
since it is 5/2 times a convergent p-series (p = 2. Observe that
5 5
<
2n2 + 4n + 3 2n2
for all n. This means that
∞ ∞
5 5
∑ 2n2 + 4n + 3 ∑ 2n2 .
<
n=1 n=1
Since the series on the right is convergent and by part (i) of the Comparison Test, the given
series is also convergent.


ln n
■ Example 6.6 Test the series ∑ for convergence or divergence.
n=1 n
6.2 The Comparison Tests 69

Solution: The most useful series that we can think of at the moment is the harmonic series

1
∑ n which is known to diverge. Observe that ln n > 1 for n ≥ 3. Thus,
n=1

ln n 1
> , n ≥ 3.
n n
This means that
∞ ∞
ln n 1
Terms of use: This course material is strictly for class use only to students officially

∑ >∑ , n ≥ 3.
n=3 n n=3 n
enrolled in MATH 54, ELEMENTARY ANALYSIS II, SECOND SEMESTER, A.Y. 2023 - 2024

The series on the right is divergent since it is just the harmonic series less than the sum of the

printed, reproduced, shared, or sold without permission from the author.


Copyright protection is governed by R.A. 10372. This work cannot be
first two terms. Thus, the series on the left is also divergent by part (ii) of the Comparison test.
Therefore, the given series is divergent.


1
Consider the series ∑ 2n − 1 . You might to compare it with the convergent geometric
n=1

1
series ∑ n and notice that
n=1 2
∞ ∞
1 1
>
∑ 2n − 1 ∑ 2n .
n=1 n=1
Here, the given series is greater than our (convergent) comparison series. Hence, we cannot
use the Comaparison Test for this. In such cases the following test can be used.

Theorem 6.2.2 The Limit Comparison Test

Suppose that ∑ an and ∑ bn are series with positive terms. If


an
lim =c
n→∞ bn

where c is a finite number and c > 0, then either both series converge or both diverge.

■ Example 6.7 We use the Limit Comparison Test with

1 1
an = and bn = .
2n − 1 2n
Note that bn is the nth term of a convergent geometric series. Now,
an 1/(2n − 1) 2n 1
lim = lim = lim = lim = 1 > 0.
n→∞ bn n→∞ 1/2n n→∞ 2n − 1 n→∞ 1 − 1/2n

Therefore, by the Limit Comparison Test, the given series is convergent.


6.3 Alternating Series 70

6.3 Alternating Series

An alternating series is a series whose terms are alternately positive and negative. Here are
two examples:


1 1 1 1 1 1
1 − + − + − + · · · = ∑ (−1)n−1
Terms of use: This course material is strictly for class use only to students officially

2 3 4 5 6 n=1 n

enrolled in MATH 54, ELEMENTARY ANALYSIS II, SECOND SEMESTER, A.Y. 2023 - 2024

1 2 3 4 5 6 n
− + − + − + + · · · = ∑ (−1)n

printed, reproduced, shared, or sold without permission from the author.


2 3 4 5 6 7 n=1 n+1

Copyright protection is governed by R.A. 10372. This work cannot be


We see from these examples that the nth term of an alternating series is of the form

an = (−1)n−1 bn or an = (−1)n bn

where bn is a positive number. (In fact bn = |an |.)


The following test says that if the terms of an alternating series decrease toward 0 in
absolute value, then the series converges.

Theorem 6.3.1 The Alternating Series Test

If the alternating series



∑ (−1)n−1bn = b1 − b2 + b3 − b4 + b5 − b6 + · · · , bn > 0
n=1

satisfies
(i) bn+1 ≤ bn
(ii) lim bn = 0
n→∞
then the series is convergent.

■ Example 6.8 The alternating harmonic series



1 1 1 1 1 1
1 − + − + − + · · · = ∑ (−1)n−1
2 3 4 5 6 n=1 n
satisfies
1 1
(i) bn+1 < bn because <
n+1 n
1
(ii) lim bn = lim =0
n→∞ n→∞ n
6.4 Exercises 71

so the series is convergent by the Alternating Series Test.



3n
■ Example 6.9 The series
∑ (−1)n 4n − 1 is alternating,
n=1

3n 3 3
lim bn = lim = =
n→∞ n→∞ 4n − 1 1 4
4−
n
so condition (ii) is not satisfied. Instead, we look at the limit of the nth term of the series:
Terms of use: This course material is strictly for class use only to students officially

(−1)n 3n
enrolled in MATH 54, ELEMENTARY ANALYSIS II, SECOND SEMESTER, A.Y. 2023 - 2024

lim an = lim

printed, reproduced, shared, or sold without permission from the author.


n→∞ n→∞ 4n − 1

Copyright protection is governed by R.A. 10372. This work cannot be


This limit does not exist, so the series diverges by the Test for Divergence.

6.4 Exercises
Exercise 6.1 Determine whether the following series is convergent or divergent.

1
1. ∑ √n
n=1

1
2. ∑ n(n + 1)
n=1

1
3. ∑ ln n
n=1

1
4. ∑ n2n
n=1

1
5. ∑ (−1)n+1 ln n
n=3
Terms of use: This course material is strictly for class use only to students officially
enrolled in MATH 54, ELEMENTARY ANALYSIS II, SECOND SEMESTER, A.Y. 2023 - 2024

VII
7

7.4
7.3
7.2
7.1

Exercises
Rearrangements
Absolute Convergence

Strategy for Testing Series


Week Seven

Infinite Sequence and Series . . . . . . . . . 73


Copyright protection is governed by R.A. 10372. This work cannot be
printed, reproduced, shared, or sold without permission from the author.
Terms of use: This course material is strictly for class use only to students officially
enrolled in MATH 54, ELEMENTARY ANALYSIS II, SECOND SEMESTER, A.Y. 2023 - 2024

printed, reproduced, shared, or sold without permission from the author.


Copyright protection is governed by R.A. 10372. This work cannot be
7. Infinite Sequence and Series

At the end of this topic you are expected to:


• determine the convergence or divergence of series using ratio and root tests
• apply the different strategies for testing the convergence or divergence of series

7.1 Absolute Convergence

Given any series ∑ an , we can consider the corresponding series



∑ |an| = |a1| + |a2| + |a3| + · · ·
n=1

whose terms are the absolute values of the terms of the original series.

Definition 7.1.1 A series ∑ an is called absolutely convergent if the series of absolute

values ∑ |an | is convergent.

Notice that if ∑ an is a series with positive terms, then |an | = an and so absolute convergence
is the same as convergence in this case.
7.1 Absolute Convergence 74

■ Example 7.1 The series



(−1)n−1 1 1 1
∑ n2 = 1 − 22 + 32 − 42 + · · ·
n=1

is absolutely convergent because


(−1)n−1 1 1 1
∑ = 1+ 2 + 2 + 2 +···
Terms of use: This course material is strictly for class use only to students officially

n 2 2 3 4
n=1
enrolled in MATH 54, ELEMENTARY ANALYSIS II, SECOND SEMESTER, A.Y. 2023 - 2024

is a convergent p-series (p = 2).

printed, reproduced, shared, or sold without permission from the author.


Copyright protection is governed by R.A. 10372. This work cannot be
■ Example 7.2 We know that the alternating harmonic series

(−1)n−1 1 1 1 1 1
∑ n = 1− 2 + 3 − 4 + 5 − 6 +···
n=1

is convergent, but it is not absolutely convergent because the corresponding series of absolute
values is

(−1)n−1 ∞
1 1 1 1 1 1
∑ = ∑ = 1+ + + + + +···
n=1 n n=1 n 2 3 4 5 6
which is the harmonic series (p-series with p = 1) and is therefore divergent.
Definition 7.1.2 A series ∑ an is called conditionally convergent if it is convergent but not

absolutely convergent.

■ Example 7.3 The alternating harmonic series is conditionally convergent since this series is

convergent but the corresponding series of absolute values is divergent as shown in Example
7.2.

Thus it is possible for a series to be convergent but not absolutely convergent. However,
the next theorem shows that absolute convergence implies convergence.

Theorem 7.1.1 If a series ∑ an is absolutely convergent, then it is convergent.

■ Example 7.4 Determine whether the series



cos n cos 1 cos 2 cos 3
∑ 2
= 2 + 2 + 2 +···
n=1 n 1 2 3

is convergent or divergent.
7.1 Absolute Convergence 75

Solution: This series has both positive and negative terms, but it is not alternating. (The first
term is positive, the next three are negative, and the following three are positive: the signs
change irregularly.) We can apply the Comparison Test to the series of absolute values
∞ ∞
cos n |cos n|
∑ 2
= ∑ 2
n=1 n n=1 n

Since | cos n| ≤ 1 for all n, we have


Terms of use: This course material is strictly for class use only to students officially

|cos n| 1
enrolled in MATH 54, ELEMENTARY ANALYSIS II, SECOND SEMESTER, A.Y. 2023 - 2024

2
≤ 2
n n

printed, reproduced, shared, or sold without permission from the author.


Copyright protection is governed by R.A. 10372. This work cannot be
We know that ∑ 1/n2 is convergent (p-series with p = 2) and therefore ∑ | cos n|/n2 is con-
vergent by the Comparison Test. Thus the given series ∑(cos n)/n2 is absolutely convergent
and therefore convergent by Theorem 7.1.1.

The following test is very useful in determining whether a given series is absolutely
convergent.

7.1.1 The Ratio Test

Theorem 7.1.2 The Ratio Test



an+1
(i) If lim = L < 1, then the series ∑ an is absolutely convergent (and therefore
n→∞ an
n=1
convergent).

an+1 an+1
(ii) If lim = L > 1 or lim = ∞ or then the series ∑ an is divergent.
n→∞ an n→∞ an
n=1
an+1
(iii) If lim = 1, the Ratio Test is inconclusive, that is, no conclusion can be drawn
n→∞ an
about the convergence or divergence of ∑ an .

(−1)n n3
■ Example 7.5 Test the series ∑ 3n for absolute convergence.
n=1
(−1)n n3
Solution: We use the Ratio Test with an = :
3n
7.1 Absolute Convergence 76

(−1)n+1 (n + 1)3
an+1 3 n+1 (n + 1)3 3n
= = ·
an (−1)n n3 3n+1 n3
3n
1 n+1 3 1
   
1 1
= = 1+ → <1
3 n 3 n 3
Terms of use: This course material is strictly for class use only to students officially

Thus, by the Ratio Test, the given series is absolutely convergent.


enrolled in MATH 54, ELEMENTARY ANALYSIS II, SECOND SEMESTER, A.Y. 2023 - 2024

∞ n
n

printed, reproduced, shared, or sold without permission from the author.


■ Example 7.6 Test the convergence of the series
∑ n! .

Copyright protection is governed by R.A. 10372. This work cannot be


n=1
nn
Solution: Since the terms an = are positive, we don’t need the absolute value signs.
n!
(n + 1)n+1
an+1 (n + 1)! (n + 1)(n + 1)n n!
= = · n
an nn (n + 1)n! n
n!
n+1 n 1 n
   
= = 1+ →e as n→∞
n n
Since e > 1, the given series is divergent by the Ratio Test.

The following test is convenient to apply when nth powers occur.

7.1.2 The Root Test

Theorem 7.1.3 The Root Test


p ∞
n
(i) If lim |an | = L < 1, then the series ∑ an is absolutely convergent (and therefore
n→∞
n=1
convergent).
p p ∞
(ii) If lim n |an | = L > 1 or lim |an | = ∞ or then the series ∑ an is divergent.
n→∞ n→∞
p n=1
n
(iii) If lim |an | = 1, the Root Test is inconclusive, that is, no conclusion can be drawn
n→∞
about the convergence or divergence of ∑ an .
p
n
If lim |an | = 1 , then part (iii) of the Root test says the tests gives no information. The se-
n→∞
ries ∑ an could converge or diverge. (If L = 1 in the Ratio Test, don’t try the Root Test because
7.2 Rearrangements 77

L will again be 1. And if L = 1 in the Root Test, don’t try the Ratio Test because it will fail too.)

∞  n
2n + 3
■ Example 7.7 Test the convergence of the series ∑ .
n=1 3n + 2
Solution:  n
2n + 3
an =
3n + 2
Terms of use: This course material is strictly for class use only to students officially

3
2+
enrolled in MATH 54, ELEMENTARY ANALYSIS II, SECOND SEMESTER, A.Y. 2023 - 2024

p
n 2n + 3 n → 2 <1
|an | = =

printed, reproduced, shared, or sold without permission from the author.


3n + 2 2 3

Copyright protection is governed by R.A. 10372. This work cannot be


3+
n
Thus, the given series is absolutely convergent ( and therefore convergent) by the Root Test.

7.2 Rearrangements

If we rearrange the order of the terms in a finite sum, then of course the value of the sum
remains unchanged. But this is not always the case for an infinite series. By rearrangement
of an infinite series ∑ an , we mean a series obtained by simply changing the order of the terms.
For instance, a rearragement of ∑ an could start as follows:

a1 + a2 + a5 + a3 + a4 + a15 + a6 + a7 + a20 + · · ·

It turns out that

if ∑ an is an absolutely convergent series with sum s,


then any rearrangement of ∑ an has the same sum s.

However, any conditionally convergent series can be rearranged to give a different sum. To
illustrate this fact, let’s consider the alternating harmonic series
1 1 1 1 1 1 1
1 − + − + − + − + · · · = ln 2 (7.1)
2 3 4 5 6 7 8
Multiplying both sides by 21 , we get
1 1 1 1
− + − + · · · = 12 ln 2 (7.2)
2 4 6 8
7.3 Strategy for Testing Series 78

Inserting zeros between the terms of this series, we have


1 1 1 1
0 + + 0 − + 0 + + 0 − + · · · = 12 ln 2 (7.3)
2 4 6 8
Finally, adding the first and last equations, we have
1 1 1 1 1
1 + − + + − · · · = 23 ln 2 (7.4)
3 2 5 7 4
Terms of use: This course material is strictly for class use only to students officially

Notice that the resulting series contains the same terms as in the very first equation but
enrolled in MATH 54, ELEMENTARY ANALYSIS II, SECOND SEMESTER, A.Y. 2023 - 2024

rearranged. The sums of these series, however, are different. In fact, Riemann proved that

printed, reproduced, shared, or sold without permission from the author.


Copyright protection is governed by R.A. 10372. This work cannot be
if ∑ an is a conditionally convergent series and r is any real number
whatsoever, then there is a rearrangement of ∑ an that has a sum equal to r.

7.3 Strategy for Testing Series

We now have several ways of testing a series for convergence or divergence; the problem is to
decide which test to use on which series. In this respect, testing series is similar to integrating
functions. Again there are no hard and fast rules about which test to apply to a given series,
but you may find the following advice of some use. It is not wise to apply a list of the tests in
a specific order until one finally works. That would be a waste of time and effort. Instead, as
with integration, the main strategy is to classify the series according to its form.
1. If the series is of the form ∑ 1/n p , it is a p-series, which we know to be convergent if
p > 1 and divergent if p < 1.
2. If the series has the form ∑ arn−1 or ∑ arn , it is a geometric series, which converges if
|r| < 1 and diverges if |r| ≥ 1. Some preliminary algebraic manipulation may be required
to bring the series into this form.
3. If the series has a form that is similar to a p-series or a geometric series, then one of
the comparison tests should be considered. In particular, if an is a rational function or
an algebraic function of n (involving roots of polynomials), then the series should be
compared with a p-series. The comparison tests apply only to series with positive terms,
but if ∑ an has some negative terms, then we can apply the Comparison Test to ∑ an and
test for absolute convergence.
7.3 Strategy for Testing Series 79

4. If you can see at a glance that lim an ̸= 0, then the Test for Divergence should be used.
n→∞
5. If the series is of the form ∑(−1)n−1 bn or ∑(−1)n bn , then the Alternating Series Test is
an obvious possibility.
6. Series that involve factorials or other products (including a constant raised to the nth
power) are often conveniently tested using the Ratio Test. Bear in mind that |an+1 /an | →
1 as n → ∞ for all p-series and therefore all rational or algebraic functions of n. Thus the
Terms of use: This course material is strictly for class use only to students officially

Ratio Test should not be used for such series.


enrolled in MATH 54, ELEMENTARY ANALYSIS II, SECOND SEMESTER, A.Y. 2023 - 2024

7. If an is of the form (bn )n , then the Root Test may be useful.

printed, reproduced, shared, or sold without permission from the author.


Copyright protection is governed by R.A. 10372. This work cannot be
R∞
8. If an = f (n), where 1 f (x) dx is easily evaluated, then the Integral Test is effective
(assuming the hypotheses of this test are satisfied).

In the following examples we don’t work out all the details but simply indicate which
tests should be used.


n−1
■ Example 7.8 ∑ 2n + 1
n=1

1
Since an → ̸= 0 as n → ∞, we should use the test for divergence.
2


n3 + 1
■ Example 7.9
∑ 3 2
n=1 3n + 4n + 2

Since an is an algebraic function of n, we compare the given series with a p-series. The
comparison series for the Limit Comparison Test is ∑ bn , where
√ 3
n3 n2 1
bn = 3 = 3 = 3
3n 3n 3n 2

2
■ Example 7.10 ∑ ne−n
n=1
2
xe−x dx is easily evaluated, we use the integral test. The Ratio Test also
R∞
Since the integral 1
works.

n3
■ Example 7.11 ∑ (−1)n n4 + 1
n=1
Since the series is alternating, we use the Alternating Series Test.
7.4 Exercises 80

2k

■ Example 7.12 ∑
k=1 k!
Since the series involves k!, we use the Ratio Test.


1
■ Example 7.13 ∑ 2 + 3n
n=1
Since the series is closely related to the geometric series ∑ 1/3n , we use the Comparison Test.
Terms of use: This course material is strictly for class use only to students officially
enrolled in MATH 54, ELEMENTARY ANALYSIS II, SECOND SEMESTER, A.Y. 2023 - 2024

7.4 Exercises

printed, reproduced, shared, or sold without permission from the author.


Copyright protection is governed by R.A. 10372. This work cannot be
Exercise 7.1 Test the following series for convergence or divergence.

n2 − 1
1. ∑ 3
n=1 n + 1
∞ 2
nn −1
2. ∑ (−1)
n=1 n3 + 1
∞ n
e
3. ∑ n2
n=1

1
4. ∑ n√ln n
n=2

5k
5. ∑ k k
k=1 3 + 4
Terms of use: This course material is strictly for class use only to students officially
enrolled in MATH 54, ELEMENTARY ANALYSIS II, SECOND SEMESTER, A.Y. 2023 - 2024

VIII

8.2
8.1
Exercises
Power Series
Week Eight

Infinite Sequences and Series . . . . . . . . 82


Copyright protection is governed by R.A. 10372. This work cannot be
printed, reproduced, shared, or sold without permission from the author.
Terms of use: This course material is strictly for class use only to students officially
enrolled in MATH 54, ELEMENTARY ANALYSIS II, SECOND SEMESTER, A.Y. 2023 - 2024

printed, reproduced, shared, or sold without permission from the author.


Copyright protection is governed by R.A. 10372. This work cannot be
8. Infinite Sequences and Series

At the end of this topic you are expected to:


• determine the convergence of a power series
• solve for the radius and interval of convergence of a power series

8.1 Power Series

A power series is a series of the form



∑ cnxn = c0 + c1x + c2x2 + c3x3 + ...
n=0

where x is a variable and the c′n s are constants called the coefficients of the series. For
each fixed x, the series ∑∞ n
n=0 cn x is a series of constants that we can test for convergence or
divergence. A power series may converge for some values of x and diverge for other values of
x. The sum of the series is a function

f (x) = c0 + c1 x + c2 x2 + · · · + cn xn + · · ·
8.1 Power Series 83

whose domain is the set of all x for which the series converges. Notice that f resembles a
polynomial. The only difference is that f has infinitely many terms.
For instance, if we take cn = 1 for all n, the power series becomes the geometric series

∑ xn = 1 + x + x2 + · · · + xn + · · ·
n=0

which converges when −1 < x < 1 and diverges when |x| ≥ 1.


Terms of use: This course material is strictly for class use only to students officially

More generally, a series of the form


enrolled in MATH 54, ELEMENTARY ANALYSIS II, SECOND SEMESTER, A.Y. 2023 - 2024

printed, reproduced, shared, or sold without permission from the author.


∑ cn(x − a)n = c0 + c1(x − a) + c2(x − a)2 + ...

Copyright protection is governed by R.A. 10372. This work cannot be


n=0

is called a power series in (x − a) ad or a power series centered at a or a power series


about a.

■ Example 8.1 For what values of x is the series ∑ n!xn convergent?
n=0
Solution: We use the Ratio Test. If we let an , as usual, denote the nth term of the series, then
an = n!xn . If x ̸= 0, we have
an+1 (n + 1)!xn+1
lim = lim = lim (n + 1)|x| = ∞
n→∞ an n→∞ n!xn n→∞

By the Ratio Test, the series diverges when x ̸= 0. Thus the given series converges only when
x = 0.

(x − 3)n
■ Example 8.2 For what values of x does the series ∑ n converge?
n=1
Solution: Let an = (x − 3)n /n. Then

an+1 (x − 3)n+1 n
= ·
an n+1 (x − 3)n
1
= |x − 3| → |x − 3| n → ∞
1
1+
n
By the Ratio Test, the given series is absolutely convergent, and therefore convergent, when
|x − 3| < 1 and divergent when |x − 3| > 1. Now

|x − 3| < 1 ⇔ −1 < x − 3 < 1 ⇔ 2 < x < 4

so the series converges when 2 < x < 4 and diverges when x < 2 or x > 4.
8.1 Power Series 84

The Ratio Test gives no information when |x − 3| = 1 so we must consider x = 2 and x = 4


separately. If we put x = 4 in the series, it becomes ∑ 1/n, the harmonic series, which is
divergent. If x = 2, the series is ∑(−1)n /n, which converges by the Alternating Series Test.
Thus, the given power series converges for 2 ≤ x < 4.

■ Example 8.3 Find the domain of the Bessel function of order 0 defined by

(−1)n x2n
Terms of use: This course material is strictly for class use only to students officially

J0 (x) = ∑ 2n 2
n=0 2 (n!)
enrolled in MATH 54, ELEMENTARY ANALYSIS II, SECOND SEMESTER, A.Y. 2023 - 2024

printed, reproduced, shared, or sold without permission from the author.


Solution: Let an = (−1)n x2n /[22n (n!)2 ]. Then

Copyright protection is governed by R.A. 10372. This work cannot be


an+1 (−1)n+1 x2(n+1) 22n (n!)2
= 2(n+1) ·
an 2 [(n + 1)!]2 (−1)n x2n
x2n+2 22n (n!)2
= ·
22n+2 (n + 1)2 (n!)2 x2n
x2
= →0<1 for all x
4(n + 1)2
Thus, by the Ratio Test, the given series converges for all values of x. In other words, the
domain of the Bessel function J0 is (−∞, ∞) = R.

Theorem 8.1.1 For a given power series ∑∞ n


n=0 cn (x − a) , there are only three possibilities:
(i) The series converges only when x = a.
(ii) The series converges for all x.
(iii) There is a positive number R such that the series converges if |x − a| < R and diverges
if |x − a| > R.

The number R in case (iii) is called the radius of convergence of the power series. By
convention, the radius of convergence is R = 0 in case (i) and R = ∞ in case (ii). The interval
of convergence of a power series is the interval that consists of all values of x for which the
series converges.

■ Example 8.4 Find the radius of convergence and interval of convergence of the series

(−3)n xn
∑ √n + 1 .
n=0
8.1 Power Series 85

Solution: Let an = (−3)n xn / n + 1. Then

an+1 (−3)n+1 xn+1 n+1
= √ ·
an n+2 (−3)n xn
r
n+1
= −3x
n+2
s
1 + (1/n)
=3 |x| → 3|x| as n → ∞
Terms of use: This course material is strictly for class use only to students officially

1 + (2/n)
enrolled in MATH 54, ELEMENTARY ANALYSIS II, SECOND SEMESTER, A.Y. 2023 - 2024

By the Ratio Test, the given series converges if 3|x| < 1 and diverges if 3|x| > 1. Thus it

printed, reproduced, shared, or sold without permission from the author.


1 1

Copyright protection is governed by R.A. 10372. This work cannot be


converges if |x| < and diverges if |x| > . This means that the radius of convergence is
3 3
1
R= .
3
We know the series converges in the interval (− 13 , 13 ), but we must now test for convergence
at the endpoints of this interval. If x = − 13 , the series becomes
∞ (−3)n (− 13 )n ∞
1 1 1 1 1
∑ √ = ∑ √ = √ + √ + √ + √ +···
n=0 n+1 n=0 n + 1 1 2 3 4
1
which diverges. (Use the Integral Test or simply observe that it is a p-series with p = 2 < 1.)
If x = 31 , the series is
∞ (−3)n ( 13 )n ∞
(−1)n

∑ n+1 ∑ n+1 = √
n=0 n=0
which converges by the Alternating Series Test. Therefore, the given power series converges
when − 13 < x ≤ 13 , so the interval of convergence is (− 13 , 31 ].

■ Example 8.5 Find the radius of convergence and interval of convergence of the series

n(x + 2)n
∑ n+1
.
n=0 3

Solution: Let an = n(x + 2)n /3n+1 . Then


an+1 (n + 1)(x + 2)n+1 3n+1
= ·
an 3n+2 n(x + 2)n
 
1 |x + 2| |x + 2|
= 1+ → as n → ∞
n 3 3
Using the Ratio Test, we see that the series converges if |x + 2|/3 < 1 and it diverges if
|x + 2|/3 > 1. So it converges if |x + 2| < 3 and diverges if |x + 2| > 3. Thus the radius of
convergence is R = 3.
8.2 Exercises 86

The inequality |x + 2| < 3 can be written as −5 < x < 1, so we test the series at the
endpoints −5 and 1. When x = −5, the series is

n(−3)n 1 ∞
∑ 3n+1 = 3 ∑ (−1)nn
n=0 n=0

which diverges by the Test for Divergence [(−1)n n doesn’t converge to 0]. When x = 1, the
series is
Terms of use: This course material is strictly for class use only to students officially


n(3)n 1 ∞
= ∑n
enrolled in MATH 54, ELEMENTARY ANALYSIS II, SECOND SEMESTER, A.Y. 2023 - 2024

∑ n+1
n=0 3 3 n=0

printed, reproduced, shared, or sold without permission from the author.


Copyright protection is governed by R.A. 10372. This work cannot be
which also diverges by the Test for Divergence. Thus, the series converges only when
−5 < x < 1, so the interval of convergence is (−5, 1).

8.2 Exercises
Exercise 8.1 Find the radius of convergence and interval of convergence of the following

series:

1. ∑ (−1)nnxn
n=1

(−1)n xn
2. ∑ √3 n
n=1

(−1)n xn
3. ∑
n=1 n2

4. ∑ nnxn
n=1

5. ∑ 2nn2xn.
n=1
Terms of use: This course material is strictly for class use only to students officially
enrolled in MATH 54, ELEMENTARY ANALYSIS II, SECOND SEMESTER, A.Y. 2023 - 2024

IX
9

9.4
9.3
9.2
9.1

Exercises
Taylor and Maclaurin Series
Representations of Functions as Power Series
Differentiation and Integration of Power Series
Week Nine

Infinite Sequences and Series . . . . . . . . 88


Copyright protection is governed by R.A. 10372. This work cannot be
printed, reproduced, shared, or sold without permission from the author.
Terms of use: This course material is strictly for class use only to students officially
enrolled in MATH 54, ELEMENTARY ANALYSIS II, SECOND SEMESTER, A.Y. 2023 - 2024

printed, reproduced, shared, or sold without permission from the author.


Copyright protection is governed by R.A. 10372. This work cannot be
9. Infinite Sequences and Series

At the end of this topic you are expected to:


• write representations of functions as power series
• find the Maclaurin and Taylor series of a function

9.1 Representations of Functions as Power Series

In this section we will learn how to represent certain types of fucntions as sums of power
series by manipulating geometric series or by differentiating or integrating such series. You
might wonder why we would ever want to express a known function as a sum of infinitely
many terms.
We begin with an equation that we have encountered before:

1
= 1 + x + x2 + x3 + · · · = ∑ xn |x| < 1
1−x n=0

We have encountered this equation before where we obtain it by observing that the series is
a geometric series with a = 1 and r = x. But our point of view is different. We now regard the
9.2 Differentiation and Integration of Power Series 89

above equation as expressing the function f (x) = 1/(1 − x) as a sum of a power series.

■ Example 9.1 Express 1/(1 + x2 ) as the sum of a power series and find the interval of

convergence.

Solution: Replacing x by −x2 as in the given series above, w have



1 1
= = ∑ (−x2 )n
Terms of use: This course material is strictly for class use only to students officially

1 + x2 1 − (−x2 ) n=0

enrolled in MATH 54, ELEMENTARY ANALYSIS II, SECOND SEMESTER, A.Y. 2023 - 2024

= ∑ (−1)nx2n = 1 − x2 + x4 − x6 + x8 − · · ·

printed, reproduced, shared, or sold without permission from the author.


n=0

Copyright protection is governed by R.A. 10372. This work cannot be


Beacause this is a geometric series, it converges when | − x2 | < 1, that is x2 < 1, or |x| < 1.
Therefore, the interval of convergence is (−1, 1).

1
■ Example 9.2 Find a power series representation for
x+2
Solution: We factor first a 2 from the denominator:
1 1 1
=  =
2+x 2 1+ x x i
 h 
2 1− −
2 2
1 ∞  x n ∞
(−1)n n
= ∑ − = ∑ n+1 x
2 n=0 2 n=0 2

This series converges when | − x/2| < 1, that is, |x| < 2. So the interval of convergence is
(−2, 2).

9.2 Differentiation and Integration of Power Series

The sum of a power series is a function f (x) = ∑∞ n


n=0 cn (x − a) whose domain is the interval
of convergence of the series. We would like to be able to differentiate and integrate such
functions, and the following theorem says that we can do so by differentiating or integrating
each individual term in the series. This process is known as term-by-term differentiation or
integration.
9.2 Differentiation and Integration of Power Series 90

Theorem 9.2.1 If the power series ∑∞ n


n=0 cn (x − a) has radius of convergence R > 0, then
the function f defined by

f (x) = c0 + c1 (x − a) + c2 (x − a)2 + · · · = ∑ cn(x − a)n
n=0

is differentiable on the interval (a − R, a + R) and



f ′ (x) = c1 + 2c2 (x − a) + 3c3 (x − a)2 + · · · = ∑ ncn(x − a)n−1
Terms of use: This course material is strictly for class use only to students officially

(i)
n=1
enrolled in MATH 54, ELEMENTARY ANALYSIS II, SECOND SEMESTER, A.Y. 2023 - 2024

(x − a)2 (x − a)3 ∞
(x − a)n+1
Z
(ii) f (x) dx = C + c0 (x − a) + c1 + c2 + · · · = C + ∑ cn

printed, reproduced, shared, or sold without permission from the author.


2 3 n+1

Copyright protection is governed by R.A. 10372. This work cannot be


n=0
The radii of convergence of the power series in equations (i) and (ii) are both R.

■ Example 9.3 Express 1/(1 − x)2 as a power series by differentiating the power series for

1/(1 − x). What is the radius of convergence?

Solution: Differentiating each side of the equation



1 2 3
= 1 + x + x + x + · · · = ∑ xn
1−x n=0

we get

1 2
2
= 1 + 2x + 3x + · · · = ∑ nxn−1 .
(1 − x) n=1
If we wish, we can replace n by n + 1 and write the answer as


1
= ∑ (n + 1)xn.
(1 − x)2 n=0
According to the above theorem, the radius of convergence of the differentiated series is
the same as the radius of covergence of the original series which is R = 1.

■ Example 9.4 Find a power series representation for ln(1 + x) and its radius of convergence.

Solution: We notice that the derivative of this function is 1/(1 + x). Thus, after integrating
each side of the power series
1 1
= = 1 − x + x2 − x3 + · · · |x| < 1
1 + x 1 − (−x)
9.3 Taylor and Maclaurin Series 91

we have
1
Z Z
ln(1 + x) = dx = (1 − x + x2 − x3 + · · · ) dx
1+x
x2 x3 x4
= x − + − + . . . +C
2 3 4

x n
= ∑ (−1)n−1 +C |x| < 1.
n=1 n
Terms of use: This course material is strictly for class use only to students officially

To determine the value of C we put x = 0 in this equation and obtain ln(1 + 0) = C which
enrolled in MATH 54, ELEMENTARY ANALYSIS II, SECOND SEMESTER, A.Y. 2023 - 2024

implies that C = 0. Thus,

printed, reproduced, shared, or sold without permission from the author.


Copyright protection is governed by R.A. 10372. This work cannot be

xn
ln(1 + x) = ∑ (−1)n−1 n
|x| < 1.
n=1
The radius of convergence is the same as for the orginal series: R = 1.

■ Example 9.5 Find the power series representation for f (x) = tan−1 x.

Solution: We can find the required series by integrating the power series for 1/(1 + x2 ).
1
Z Z
−1
tan x= dx = (1 − x2 + x4 − x6 + · · · ) dx
1 + x2
x3 x5 x7
= C +x− + − +···
3 5 7
To find C, we put x = 0 and obtain C = 0. Therefore,
x3 x5 x7
tan−1 x = x − + − +···
3 5 7

x 2n+1
= ∑ (−1)n .
n=0 2n + 1

Since the radius of convergence of the series for 1/(1 + x2 ) is 1, the radius of convergence of
this series for tan−1 x is also 1.

9.3 Taylor and Maclaurin Series

In the precceding section, we were able to find the power series representaions for a certain
restricted class of functions. Here we investigate more general problems: Which functions
have power series representations? How can we find such representations?
9.3 Taylor and Maclaurin Series 92

We start by supposing that f is any function that can be represented by a power series

f (x) = c0 + c1 (x − a) + c2 (x − a)2 + c3 (x − a)3 + c4 (x − a)4 + · · · |x − a| < R

If we put x = a in the above equation, then all the terms after the first are 0 and we get

f (a) = c0 .
Terms of use: This course material is strictly for class use only to students officially

Also,
enrolled in MATH 54, ELEMENTARY ANALYSIS II, SECOND SEMESTER, A.Y. 2023 - 2024

printed, reproduced, shared, or sold without permission from the author.


f ′ (x) = c1 + 2c2 (x − a) + 3c3 (x − a)2 + 4c4 (x − a)3 + · · · |x − a| < R

Copyright protection is governed by R.A. 10372. This work cannot be


and substitution of x = a in this equation gives

f ′ (a) = c1 .

Following the same process we obtain the following results:

f ′′ (a) = 2c2 and f ′′′ (a) = 2 · 3c3 = 3!c3 .

If we continue to differntiate and substitute x = a, we obtaib

f n (a) = 2 · 3 · 4 · ncn = n!cn .

Solving this equation for the nth coefficient, we get

f (n) (a)
cn = .
n!
This formula remains valid even when n is 0 if we adopt the conventions that 0! = 1 and
f (0) = f . Thus, we have the following theorem.
Theorem 9.3.1 If f has the power series representation (expansion) at a, that is, if

f (x) = ∑ cn(x − a)n |x − a| < R
n=0

then its coefficients are given by the formula

f (n) (a)
cn = .
n!

Substituting this formula for cn back into the series, we have


9.3 Taylor and Maclaurin Series 93


f (n) (a)
f (x) = ∑ (x − a)n
n=0 n!
f ′ (a) f ′′ (a) f ′′′ (a)
= f (a) + (x − a) + (x − a)2 + (x − a)3 + · · ·
1! 2! 3!

The series in the boxed equation is called the Taylor series of the function f at a or
Terms of use: This course material is strictly for class use only to students officially

(about a). For the special case where a = 0, the Taylor series becomes
enrolled in MATH 54, ELEMENTARY ANALYSIS II, SECOND SEMESTER, A.Y. 2023 - 2024

printed, reproduced, shared, or sold without permission from the author.


Copyright protection is governed by R.A. 10372. This work cannot be

f (n) (0) n f ′ (0) f ′′ (0) 2
f (x) = ∑ x = f (0) + x+ x +···
n=0 n! 1! 2!

This case arises frequently enough tht it is given the special name Maclaurin series.

■ Example 9.6 Find the Maclaurin series of the function f (x) = ex and its radius of conver-

gence.

Solution: If f (x) = ex then f (n) (x) = ex , so f (n) (0) = e0 = 1 for all n. Therefore, the Taylor
series for f at 0 (that is, the Maclaurin series) is

f (n) (0) n ∞ n
x x x2 x3
∑ x =∑ = 1+ + + +···
n=0 n! n=0 n! 1! 2! 3!

To find the radius of convergence we let an = xn /n!. Then

an + 1 xn+1 n! |x|
= · n = →0<1
an (n + 1)! x n+1
so by the Ratio Test, the series converges for all x and the radius of convergence is R = ∞.

It can be shown also that ex is equal to the sum of its Maclaurin series, that is

xn
ex = ∑ for all x
n=0 n!

.
9.3 Taylor and Maclaurin Series 94

In particular, if we put x = 1, we obtain



1 1 1 1
e= ∑ n! = 1 + 1! + 2! + 3! + · · ·
n=0
.

■ Example 9.7 Find the Taylor series for f (x) = ex at a = 2.


Terms of use: This course material is strictly for class use only to students officially

Solution: We have f (n) (2) = e2 and so, putting a = 2 in the definition of a Taylor series, we
enrolled in MATH 54, ELEMENTARY ANALYSIS II, SECOND SEMESTER, A.Y. 2023 - 2024

printed, reproduced, shared, or sold without permission from the author.


get

Copyright protection is governed by R.A. 10372. This work cannot be



f (n) (2) ∞ 2
e
∑ (x − 2)n = ∑ (x − 2)n .
n=0 n! n=0 n!
Again it can be verified that the radius of convergence is R = ∞. Therefore,


e2
ex = ∑ n! (x − 2)n for all x.
n=0

■ Example 9.8 Find the Maclaurin series for sin x.

Solution: We arrange our computation as follows

f (x) = sin x f (0) = 0

f ′ (x) = cos x f ′ (0) = 1

f ′′ (x) = − sin x f ′′ (0) = 0

f ′′′ (x) = − cos x f ′′′ (0) = −1


f (4) (x) = sin x f (4) (0) = 0

Since the derivatives repeat in a cycle of four, we can write the Maclaurin series as follows:

f ′ (0) f ′′ (0) 2 f ′′′ (0) 3


f (0) + x+ x + x +···
1! 2! 3!
x3 x5 x7 ∞
x2n+1
= x − + − + · · · = ∑ (−1)n .
3 5! 7! n=0 (2n + 1)!
9.4 Exercises 95

It can be shown also that sin x is equal to the sum of its Maclaurin series, that is,

x3 x5 x7
sin x = x − + − +···
3 5! 7!

x2n+1
= ∑ (−1)n for all x.
n=0 (2n + 1)!
Terms of use: This course material is strictly for class use only to students officially
enrolled in MATH 54, ELEMENTARY ANALYSIS II, SECOND SEMESTER, A.Y. 2023 - 2024

Example 9.9 Find the Maclaurin series for cos x.

printed, reproduced, shared, or sold without permission from the author.


Copyright protection is governed by R.A. 10372. This work cannot be


Solution: It would be easier to obtain the Maclaurin series of the given function by differentiat-
ing the Macalurin series for sin x knowing that the derivative of sin x is cos x.

x3 x5 x7
 
d d
cos x = (sin x) = x− + − +···
dx dx 3 5! 7!
3x 2 5x 4 7x 6
= 1− += − +···
3! 5! 7!
x2 x4 x6
= 1− + − +···
2! 4! 6!

x2n
= ∑ (−1)n for all x.
n=0 (2n)!

■ Example 9.10 Find the Maclaurin series for the function f (x) = x cos x.

Solution: Instead of computing derivatives and substituting to the formula of a Taylor series, it
is easier to multiply the series for cos x by x:

x2n ∞
x2n+1
x cos x = x ∑ (−1)n = ∑ (−1)n .
n=0 (2n)! n=0 (2n)!

9.4 Exercises
Exercise 9.1 Do as directed.

1. Find the power series of the first function by differentiating the power series of the
second function:
(a) sec2 x ; tan x
Terms of use: This course material is strictly for class use only to students officially
enrolled in MATH 54, ELEMENTARY ANALYSIS II, SECOND SEMESTER, A.Y. 2023 - 2024

9.4 Exercises

(b) sin−1 x ; √
second function:

1
(a) ln cos x ; tan x

1 − x2
.
(b) sin x + x cos x; x sin x.
2. Find the power series of the first function by differentiating the power series of the
96

Copyright protection is governed by R.A. 10372. This work cannot be


printed, reproduced, shared, or sold without permission from the author.
Terms of use: This course material is strictly for class use only to students officially
enrolled in MATH 54, ELEMENTARY ANALYSIS II, SECOND SEMESTER, A.Y. 2023 - 2024

X
10

10.4
10.3
10.2
10.1

Exercises
Limits and Continuity
Functions of Several Variables
Three-Dimensional Coordinate Systems
Week Ten

Partial Derivatives . . . . . . . . . . . . . . . . . . . . 98
Copyright protection is governed by R.A. 10372. This work cannot be
printed, reproduced, shared, or sold without permission from the author.
Terms of use: This course material is strictly for class use only to students officially
enrolled in MATH 54, ELEMENTARY ANALYSIS II, SECOND SEMESTER, A.Y. 2023 - 2024

printed, reproduced, shared, or sold without permission from the author.


Copyright protection is governed by R.A. 10372. This work cannot be
10. Partial Derivatives

At the end of this topic you are expected to:


• identify the domain of functions in two variables
• find the limit of some functions in two variables

10.1 Three-Dimensional Coordinate Systems

10.1.1 3D Space
To locate a point in a plane, we need two numbers. We know that any point in the plane can be
represented as an ordered pair (a, b) of real numbers, where a is the x-coordinate and b is the
y-coordinate. For this reason, a plane is called two-dimensional. To locate a point in space,
three numbers are required. We represent any point in space by an ordered triple (a, b, c) of
real numbers.
10.1 Three-Dimensional Coordinate Systems 99

z
Terms of use: This course material is strictly for class use only to students officially

y
enrolled in MATH 54, ELEMENTARY ANALYSIS II, SECOND SEMESTER, A.Y. 2023 - 2024

printed, reproduced, shared, or sold without permission from the author.


Copyright protection is governed by R.A. 10372. This work cannot be
x

Figure 1.1: A 3-Dimensional Coordinate System.

Because many people have some difficulty visualizing diagrams of three-dimensional


figures, you may find it helpful to do the following [Figure 1.2]. Look at any bottom corner of
a room and call the corner the origin. The wall on your left is in the xz-plane, the wall on your
right is in the yz-plane, and the floor is in the xy-plane. The x-axis runs along the intersection
of the floor and the left wall. The y-axis runs along the intersection of the floor and the right
wall. The z-axis runs up from the floor toward the ceiling along the intersection of the two
walls. You are situated in the first octant, and you can now imagine seven other rooms situated
in the other seven octants [Figure 1.3] (three on the same floor and four on the floor below),
all connected by the common corner point O.
10.1 Three-Dimensional Coordinate Systems 100
Terms of use: This course material is strictly for class use only to students officially
enrolled in MATH 54, ELEMENTARY ANALYSIS II, SECOND SEMESTER, A.Y. 2023 - 2024

printed, reproduced, shared, or sold without permission from the author.


Copyright protection is governed by R.A. 10372. This work cannot be
Figure 1.2: A model of xy-, xz- and yz- plane.

Figure 1.3: A 3-D model the octants.

Usually we think of the x− and y−axes as being horizontal and the z−axis as being vertical.
The direction of the z−axis is determined by the right-hand rule as illustrated below [Figure
1.4]: If you curl the fingers of your right hand around the z-axis in the direction of a 90◦
10.1 Three-Dimensional Coordinate Systems 101

counterclockwise rotation from the positive x-axis to the positive y-axis, then your thumb
points in the positive direction of the z-axis.
Terms of use: This course material is strictly for class use only to students officially
enrolled in MATH 54, ELEMENTARY ANALYSIS II, SECOND SEMESTER, A.Y. 2023 - 2024

printed, reproduced, shared, or sold without permission from the author.


Copyright protection is governed by R.A. 10372. This work cannot be
Figure 1.4: The Right-Hand Rule.

The point P(a, b, c) determines a rectangular box as in Figure 1.5 . If we drop a perpendic-
ular from P to the xy-plane, we get a point Q with coordinates (a, b, 0) called the projection
of P onto the xy-plane. Similarly, R(0, b, c) and S(a, 0, c) are the projections of P onto the
yz-plane and xz-plane, respectively.
10.1 Three-Dimensional Coordinate Systems 102
Terms of use: This course material is strictly for class use only to students officially
enrolled in MATH 54, ELEMENTARY ANALYSIS II, SECOND SEMESTER, A.Y. 2023 - 2024

printed, reproduced, shared, or sold without permission from the author.


Copyright protection is governed by R.A. 10372. This work cannot be
Figure 1.5

Example. Plot the points (1, 6, 0), (3, 3, −2), (−2, 5, 4) and (2, −5, 3).

Solution:

The Cartesian product R × R × R = {(x, y, z) | x, y, z ∈ R} is the set of all ordered triples of


real numbers and is denoted by R3 . We have given a one-to-one correspondence between points
10.2 Functions of Several Variables 103

P in space and ordered triples (a, b, c) in R3 . It is called a three-dimensional rectangular


coordinate system. Notice that, in terms of coordinates, the first octant can be described as
the set of points whose coordinates are all positive.

10.2 Functions of Several Variables


Terms of use: This course material is strictly for class use only to students officially

In our previous calculus courses, we dealt with the functions of a single variable. This is
enrolled in MATH 54, ELEMENTARY ANALYSIS II, SECOND SEMESTER, A.Y. 2023 - 2024

usually denoted by f (x). This time, we will extend this to functions of several variables.

printed, reproduced, shared, or sold without permission from the author.


Copyright protection is governed by R.A. 10372. This work cannot be
The following are examples of functions of several variables.
1. The volume of a right circular cylinder, V = πr2 h, is a function of two variables.
2. The area of a rectangle, A = lw, is a function of two variables.
3. The volume of a rectangular solid, V = lwh, is a function of three variables.
We use the notation f (x, y) to denote a function in two variables x and y. Similarly, for
three vaiables, we have f (x, y, z).

■ Example 10.1 The following are functioons of more than one variabe:

1. f (x, y) = x2 + 2xy + y2
2. f (x, y, z) = x2 + yx + 4z
Definition 10.2.1 (Function of Two Variables)

Let D be the set of ordered pairs of real numbers. If to each ordered pair (x, y) in D there
corresponds a unique real number f (x, y), then f is a function of x and y. The set D is
the domain of f , and the corresponding set of values for f (x, y) is the range of f . For
thefunction
z = f (x, y),

x and y are called the independent variables and z called the dependent variable.

■ Example 10.2 Find the domain of the function f (x, y) = ln(y2 − x).

Solution: Since ln(y2 − x) is only defined when y2 − x > 0, the domain of f is therefore
D = {(x, y) | x < y2 }.

■ Example 10.3 Find the domain and range of f (x, y) = x2 + y2 .


10.3 Limits and Continuity 104

Solution: Since f is defined for any ordered pair (x, y), the domain is the entire xy-plane or
{(x, y) : x, y ∈ R}. Notice also that f nonnegative (x2 + y2 ≥ 0). Thus, the range is the set
[0, +∞) or the set of nonnegative real numbers.

10.3 Limits and Continuity


Terms of use: This course material is strictly for class use only to students officially

The study of the limit of a function of two variables begins by defining a two-dimensional
enrolled in MATH 54, ELEMENTARY ANALYSIS II, SECOND SEMESTER, A.Y. 2023 - 2024

analog to an interval on the real number line.

printed, reproduced, shared, or sold without permission from the author.


Copyright protection is governed by R.A. 10372. This work cannot be
Using the formula for the distance between two points (x, y) and (x0 , y0 ) in the plane, we
can define the δ -neighborhood about (x0 , y0 ) to be the disk centered at (x0 , y0 ) with radius
δ >0
q
{(x, y) : (x − x0 )2 + (y − y0 )2 < δ }.

Because the inequality sign is strictly less than δ , we have an open disk centered at (x0 , y0 )
as in the figure below.
10.3 Limits and Continuity 105

Definition 10.3.1 (Limit of a Function of Two Variables)

Let f be a function of two variables defined, except possibly at (x0 , y0 ) on an open disk
centered at (x0 , y0 ), and let L be a real number. Then

lim f (x, y) = L
(x,y)→(x0 ,y0 )

if for every ε > 0, there corresponds δ > 0 such that


Terms of use: This course material is strictly for class use only to students officially

p
| f (x, y) − L| < ε whenever 0 < (x − x0 )2 + (y − y0 )2 < δ .
enrolled in MATH 54, ELEMENTARY ANALYSIS II, SECOND SEMESTER, A.Y. 2023 - 2024

printed, reproduced, shared, or sold without permission from the author.


Copyright protection is governed by R.A. 10372. This work cannot be
An illustration of the above definition is shown below.

4x2 y
■ Example 10.4 Find lim .
(x,y)→(1,2) x2 + y2
Solution: Note that the limit of the quotient is just the quotient of the limits. In our case,

lim 4x2 y = 4(1)2 (2) = 8 and lim x2 + y2 = 12 + 22 = 5


(x,y)→(1,2) (x,y)→(1,2)
Therefore,
4x2 y 8
lim 2 2
= .
(x,y)→(1,2) x + y 5
10.3 Limits and Continuity 106

Remember that in two dimensions when we approach a real number x on the x-axis, we
have only two directions, either from the right or from the left. In three dimensions, a point
(x, y) in the plane may be approached in an infinitely many directions.

xy
■ Example 10.5 Does lim exist?
(x,y)→(0,0) x2 + y2
Terms of use: This course material is strictly for class use only to students officially

Solution: By just looking at the function, you will immediately notice that direct substitution
enrolled in MATH 54, ELEMENTARY ANALYSIS II, SECOND SEMESTER, A.Y. 2023 - 2024

would not give you the limit for both the numerator and denominator will be zero.

printed, reproduced, shared, or sold without permission from the author.


Let us consider approaching (0, 0) about the x-axis (y = 0). Then

Copyright protection is governed by R.A. 10372. This work cannot be


x(0)
lim = lim 0 = 0.
(x,0)→(0,0) x2 + 02 (x,0)→(0,0)

This time let us consider approaching (0, 0) about the y-axis (x = 0). Then
(0)y
lim = lim 0 = 0.
(0,y)→(0,0) 02 + y2 (0,y)→(0,0)

However, these are not the only directions of approach and will not immediatelytell us the
existence of the limit.
Let us consider approaching (0, 0) about the the line y = x. Then
xx x2 1 1
lim 2 2
= lim 2
= lim = .
(x,x)→(0,0) x + x (0,y)→(0,0) 2x (x,x)→(0,0) 2 2
Now, we have found another value for the limit from a different direction.
xy
Therefore, lim does not exist.
(x,y)→(0,0) x + y2
2

In your previous calculus course, showing that a function of single variable is continuous
at a real number a is simply done by showing that lim f (x) = f (a). Similarly, continuity of
x→a
functions of two variables are also defined by direct substitution.
2x2 y
■ Example 10.6 Show that lim = 0.
(x,y)→(0,0) x2 + y2
Solution: Let ε > 0 be given. First, note that
p x2
|y| ≤ x2 + y2 and 2 ≤1
x + y2
Then in a δ -neighborhood of (0, 0),
p
0< x2 + y2 < δ ,
10.3 Limits and Continuity 107

2x2 y
| f (x, y) − 0| =
x2 + y2
x2
= 2|y| 2
x + y2
≤ 2|y|
p
≤ 2 x2 + y2
Terms of use: This course material is strictly for class use only to students officially

< 2δ .
enrolled in MATH 54, ELEMENTARY ANALYSIS II, SECOND SEMESTER, A.Y. 2023 - 2024

printed, reproduced, shared, or sold without permission from the author.


If we take δ = ε2 , then

Copyright protection is governed by R.A. 10372. This work cannot be


| f (x, y) − 0| < 2δ = 2(ε/2) = ε.

Thus, we have shown that whenever


p
0 < x2 + y2 < δ , we have | f (x, y) − 0| < ε.

Therefore,
2x2 y
lim = 0.
(x,y)→(0,0) x2 + y2

Definition 10.3.2 (Continuity of a Function of Two Variables)

A function f of two variables is said to be continuous at (a, b) if

lim f (x, y) = f (a, b).


(x,y)→(a,b)

We say that f is continuous on D if f is continuous at every point (a, b) in D.

Here are some continuous functions.


1. Polynomial functions are continuous on R.
2. Rational functions are continuous on their respective domains.

■ Example 10.7 Find lim (2xy2 − x2 y + 2x − 3y).


(x,y)→(1,3)

Solution: Since f (x, y) = 2xy2 − x2 y + 2x − 3y is a polynomial function, it is continuous


everywhere and so, we can apply direct substitution. Thus,

lim (2xy2 − x2 y + 2x − 3y) = 2(1)(3)2 − (1)2 (3) + 2(1) − 3(3) = 8.


(x,y)→(1,3)

x 2 − y2
■ Example 10.8 Where is the function f (x, y) = 2 continuous?
x + y2
10.4 Exercises 108

Solution: Note that f is a rational function and thus, it is continuous on its domain which is
D = {(x, y) : (x, y) ̸= (0, 0)}.

■ Example 10.9 Determine whether the function below is continuous at (0, 0) or not.

xy

 if (x, y) ̸= (0, 0)
f (x) = x2 + y2

 0 if (x, y) = (0, 0)
Terms of use: This course material is strictly for class use only to students officially
enrolled in MATH 54, ELEMENTARY ANALYSIS II, SECOND SEMESTER, A.Y. 2023 - 2024

Solution: Note that f (0, 0) = 0, thus f (0, 0) is defined.

printed, reproduced, shared, or sold without permission from the author.


Let us take a look at lim f (x, y). Now,

Copyright protection is governed by R.A. 10372. This work cannot be


(x,y)→(0,0)

xy
lim f (x, y) = lim
(x,y)→(0,0) (x,y)→(0,0) x2 + y2

does not exists (shown previously).


Therefore, f is not continuous at (0, 0).

■ Example 10.10 Dtermine whether the given function below is continuous at (0, 0) ot not

2x2 y



2 2
if (x, y) ̸= (0, 0)
f (x) = x + y

 0 if (x, y) = (0, 0)

Solution: Note that f (0, 0) = 0, thus f (0, 0) is defined.


Let us take a look at lim f (x, y). Now,
(x,y)→(0,0)

2x2 y
lim f (x, y) = lim = 0.
(x,y)→(0,0) (x,y)→(0,0) x2 + y2

The solution for this limit is already shown before.


Finally, we have
lim f (x, y) = 0 = f (0, 0)
(x,y)→(0,0)

Therefore, f continuous at (0, 0).

10.4 Exercises
10.4 Exercises 109

Exercise 10.1 Do as directed.

1. Suppose that lim f (x, y) = 6. What can you say about the value of f (3, 1)?
(x,y)→(3,1)
What is f is continuous?
2. Find the limit, if it exists, or show that the limit does not exist.
x2 y + xy2
(a) lim
(x,y)→(2,−1) x2 − y2
(b) lim y sin(x − y)
Terms of use: This course material is strictly for class use only to students officially

(x,y)→(π,π/2)
xy4
enrolled in MATH 54, ELEMENTARY ANALYSIS II, SECOND SEMESTER, A.Y. 2023 - 2024

(c) lim

printed, reproduced, shared, or sold without permission from the author.


(x,y)→(0,0) x2 + y8

Copyright protection is governed by R.A. 10372. This work cannot be


Terms of use: This course material is strictly for class use only to students officially
enrolled in MATH 54, ELEMENTARY ANALYSIS II, SECOND SEMESTER, A.Y. 2023 - 2024

XI

11

11.3
11.2
11.1

Exercises
The Chain Rule
Partial Derivatives
Week Eleven

Partial Derivatives . . . . . . . . . . . . . . . . . . . . 111


Copyright protection is governed by R.A. 10372. This work cannot be
printed, reproduced, shared, or sold without permission from the author.
Terms of use: This course material is strictly for class use only to students officially
enrolled in MATH 54, ELEMENTARY ANALYSIS II, SECOND SEMESTER, A.Y. 2023 - 2024

printed, reproduced, shared, or sold without permission from the author.


Copyright protection is governed by R.A. 10372. This work cannot be
11. Partial Derivatives

At the end of this topic you are expected to:


• evaluate partial derivatives
• use chain rule in obtaining derivatives of composite functions

11.1 Partial Derivatives

You might ask “What will happen to the value of a function when there is a change in one of
the two independent variables?”. This commonly happens in a science experiments when a
researcher wants to know the effect if one of the the variables in an experiment is changing
while keeping other variables fixed. For instance, an experiment fixing the temperature while
pressure is changing. Similarly, we can determine the rate of change of a function f with
respect to one of its several variables. This procedure is known as partial differentiation and
the resulting function is called the partial derivative.
11.1 Partial Derivatives 112

Definition 11.1.1 (Partial Derivative of a Function of Two Variables)

If z = f (x, y), then the first partial derivatives of f with respect to x and y are the functions
defined by
f (x + h, y) − f (x, y)
fx (x, y) = lim
h→0 h
and
f (x, y + h) − f (x, y)
fy (x, y) = lim
Terms of use: This course material is strictly for class use only to students officially

h→0 h
enrolled in MATH 54, ELEMENTARY ANALYSIS II, SECOND SEMESTER, A.Y. 2023 - 2024

provided that the limits exist.

printed, reproduced, shared, or sold without permission from the author.


Copyright protection is governed by R.A. 10372. This work cannot be
The above definition tells that if you want to find fx , treat y as constant and differentiate
with respect to x. Similarly, if you want to find fy , treat x as constant and differentiate with
respect to y.

■ Example 11.1 Below are examples of partial derivatives.

1. To find fx for f (x, y) = 5x2 − x2 y2 + 3xy3 , we treat y as constant and differentiate with
respect to x. Thus,
fx (x, y) = 10x − 2xy2 + 3y3 .

To find fy for f (x, y) = 5x2 − x2 y2 + 3xy3 , we treat x as constant and differentiate with
respect to y. Thus,
fy (x, y) = −2x2 y + 9xy2 .

2. To find fx for f (x, y) = (ln x)(tan x2 y), we treat y as constant and differentiate with
respect to x. Thus,
 
2 2 1 2
fx (x, y) = (ln x)(sec x y)(2xy) + (tan x y)
x

To find fy for f (x, y) = (ln x)(tan x2 y), we treat x as constant and differentiate with
respect to y. Thus,
fy (x, y) = (ln x)(sec2 x2 y)(x2 )
11.1 Partial Derivatives 113

Notations for First Partial Derivatives


For z = f (x, y), the partial derivatives fx and fy are denoted by
∂ ∂z
f (x, y) = fx (x, y) = zx =
∂x ∂x
and
∂ ∂z
Terms of use: This course material is strictly for class use only to students officially

f (x, y) = fy (x, y) = zy =
∂y ∂y
enrolled in MATH 54, ELEMENTARY ANALYSIS II, SECOND SEMESTER, A.Y. 2023 - 2024

From evaluation of partial derivatives at point (a, b), we have

printed, reproduced, shared, or sold without permission from the author.


Copyright protection is governed by R.A. 10372. This work cannot be
∂z
= fx (a, b)
∂x
(a,b)

and
∂z
= fy (a, b).
∂y
(a,b)

2
■ Example 11.2 For f (x, y) = x ex y , find fx and fy and evaluate eacch at (1, ln 2).

Solution: For fx , we have


2 2y
fx (x, y) = x ex y (2xy) + ex

Thus,
fx (1, ln 2) = eln 2 (2 ln 2) + eln 2 = 4 ln 2 + 2.

For fy , we have
2 2
fy (x, y) = x ex y (x2 ) = x3 ex y .

Thus,
fy (1, ln 2) = eln 2 = 2.

11.1.1 Geometric Interpretations of Partial Derivatives

In the two-dimensional coordinate system, we have learned the geometric interpretation of the
the derivative of a funcion at the point (a, b) which is just the slope of the tangent line at that
point.
11.1 Partial Derivatives 114

Informally, the values of fx and fy at the point (a, b, c) on the surface denote the slopes of
the surface in the x− and y− directions, respectively as shown in the figure below.
Terms of use: This course material is strictly for class use only to students officially
enrolled in MATH 54, ELEMENTARY ANALYSIS II, SECOND SEMESTER, A.Y. 2023 - 2024

printed, reproduced, shared, or sold without permission from the author.


Copyright protection is governed by R.A. 10372. This work cannot be
■ Example 11.3 Find the slopes in the x-direction and y-direction of the surface

x2
f (x, y) =
y3
at the point (4, 2, 2).

Solution: Solving for the partial derivatives, we get


2x 3x2
fx (x, y) = 3 and fy (x, y) = − 4 .
y y
Evaluating each of these partial derivatives at (4, 2), we have
2(4) 3(4)2
fx (4, 2) = 3 = 1 and fy (4, 2) = − 4 = −3.
2 2
Therefore, the slopes of the surface in the x- and y-direction at the point (4, 2, 2) are 1 and −3
respectively.
11.1 Partial Derivatives 115

We can also apply implicit differentiation to equation in three variables in which one
variable, say z, is implicitly defined as a function of the other variables, say x and y.

■ Example 11.4 Find ∂ z/∂ x and ∂ z/∂ y if z is defined implicitly as a function of x and y by the

equation
x3 + y3 + z3 + 3xyz = 0.
Terms of use: This course material is strictly for class use only to students officially
enrolled in MATH 54, ELEMENTARY ANALYSIS II, SECOND SEMESTER, A.Y. 2023 - 2024

Solution: To find ∂ z/∂ x, we differentiate implicitly with respect to x and carefully treating y

printed, reproduced, shared, or sold without permission from the author.


as constant.

Copyright protection is governed by R.A. 10372. This work cannot be


∂z ∂z
3x2 + 0 + 3z2 + 3yz + 3xy =0
∂x ∂x
∂ z ∂z
x2 + z2 + yz + xy =0
∂x ∂x
∂z
(z2 + xy) = −x2 − yz
∂x
∂z x2 + yz
=− .
∂x z2 + xy
Similarly, implicit differentiation with respect to y gives
∂z y2 + xz
=− 2 .
∂y z + xy

11.1.2 Higher-Order Partial Derivatives

Higher-order derivatives are denoted by the order in which the differentiation occurs. For
instance, the function z = f (x, y) has the following partial derivatives.

1. Differentiate twice with respect to x:


∂2 f
 
∂ ∂f
= 2 = fxx
∂x ∂x ∂x
2. Differentiate twice with respect to y:
∂2 f
 
∂ ∂f
= 2 = fyy
∂y ∂y ∂y
3. Differentiate first with respect to x, then with respect to y:
∂2 f
 
∂ ∂f
= = fxy
∂y ∂x ∂y∂x
11.2 The Chain Rule 116

4. Differentiate first with respect to y, then with respect to x:


∂2 f
 
∂ ∂f
= = fyx
∂x ∂y ∂x∂y
The third and the fourth cases are called mixed partial derivatives.

Example 11.5 Find all second partial derivatives of the function


Terms of use: This course material is strictly for class use only to students officially


enrolled in MATH 54, ELEMENTARY ANALYSIS II, SECOND SEMESTER, A.Y. 2023 - 2024

f (x, y) = 2x2 y − 3y + 3x2 y2 .

printed, reproduced, shared, or sold without permission from the author.


Copyright protection is governed by R.A. 10372. This work cannot be
and determine the value of fxy (1, 2).

Solution: Solving for the first partial derivatives, we get

fx (x, y) = 4xy + 6xy2 and fy (x, y) = 2x2 − 3 + 6x2 y.

Now, differentiate each of these with respect to x and y, we obtain

fxx (x, y) = 4y − 6y2 and fxy (x, y) = 4x + 12xy

fyx (x, y) = 4x + 12xy and fyy (x, y) = 6x2

Finally, fxy (1, 2) = 4(1) + 12(1)(2) = 28.

11.2 The Chain Rule

In your previous calculus course, you have learned Chain Rule. That is: If y = f (x) and
x = g(t), where f and g are differentiable functions, then
dy dy dx
= + .
dt dx dt
For functions of more than one variable, the Chain Rule has several versions, each of them
giving a rule for differentiating a composite function.
11.2 The Chain Rule 117

The Chain Rule (Case 1)


Suppose z = f (x, y) is a differentiable function of x and y, where x = g(t) and y = h(t)
are both differentiable functions of t. Then z is a differentiable function of t and
dz ∂ f dx ∂ f dy
= + .
dt ∂ x dt ∂ y dt
Terms of use: This course material is strictly for class use only to students officially

Usually, we write ∂ z/∂ x in place of ∂ f /∂ x and wer can rewrite the chain rule as
enrolled in MATH 54, ELEMENTARY ANALYSIS II, SECOND SEMESTER, A.Y. 2023 - 2024

dz ∂ z dx ∂ z dy

printed, reproduced, shared, or sold without permission from the author.


= + .

Copyright protection is governed by R.A. 10372. This work cannot be


dt ∂ x dt ∂ y dt
■ Example 11.6 Let z = x2 y − y2 , where x = sin t and y = et . Find dz/dt when t = 0.

Solution: By the chain rule,


dz ∂ z dx ∂ z dy
= +
dt ∂ x dt ∂ y dt
= 2xy(cos t) + (x2 − 2y)et
= 2(sin t)(et )(cos t) + (sin2 t − 2et )et

= 2et sint cos t + et sin2 t − 2e2t .

When t = 0,
dz
= 2et sint cos t + et sin2 t − 2e2t

= −2.
dt t=0 t=0

The Chain Rule (Case 2)


Suppose that z = f (x, y) is a differentiable function of x and y, where x = g(s,t) and
y = h(s,t) are both differentiable functions of s and t. Then
∂z ∂z ∂x ∂z ∂y ∂z ∂z ∂x ∂z ∂y
= + = + .
∂s ∂x ∂s ∂y ∂s ∂t ∂ x ∂t ∂ y ∂t

■ Example 11.7 If z = ex sin y, where x = st 2 and y = s2t, find ∂ z/∂ s and ∂ z/∂t.

Solution: Using the chain rule (case 2), we have


∂z ∂z ∂x ∂z ∂y
= + = (ex sin y)(t 2 ) + (ex cos y)(2st)
∂s ∂x ∂s ∂y ∂s
2 2
= t 2 est sin(s2t) + 2stest cos(s2t)
11.2 The Chain Rule 118

and
∂z ∂z ∂x ∂z ∂y
= + = (ex sin y)(2st) + (ex cos y)(s2 )
∂t ∂ x ∂t ∂ y ∂t
2 2
= 2stest + s2 est cos(s2t).

11.2.1 Implicit Differentiation


Terms of use: This course material is strictly for class use only to students officially
enrolled in MATH 54, ELEMENTARY ANALYSIS II, SECOND SEMESTER, A.Y. 2023 - 2024

The implicit differentiation that we used to know may be completely described by the chain

printed, reproduced, shared, or sold without permission from the author.


rule.

Copyright protection is governed by R.A. 10372. This work cannot be


Suppose that y is a differntiable function of x, then applying implicit differentiation to the
equation x3 + y3 = 6xy to find dy/dx, we have

x3 + y3 = 6xy
dy dy
3x2 + 3y2 = 6y + 6x
dx dx
dy
(3y2 − 6x) = 6y − 3x2
dx
dy 6y − 3x2
= 2
dx 3y − 6x
dy 2y − x2
= 2 .
dx y − 2x
The next theorem will give us an alternative formula in finding derivatives of implicitly
defined functions as well as their partial derivatives.

Theorem 11.2.1 (Chain Rule: Implicit Differentiation)

If the equation F(x, y) = 0 defines y impliciltly as a differentiable function of x, then


dy Fx (x, y)
=− , Fy (x, y) ̸= 0
dx Fy (x, y)
If the equation F(x, y, z) = 0 defines z impliciltly as a differentiable function of x and y,
then
∂z Fx (x, y, z) ∂z Fy (x, y, z)
=− and =− Fz (x, y, z) ̸= 0.
∂x Fz (x, y, z) ∂y Fz (x, y, z)

■ Example 11.8 Find dy/dx if x3 + y3 = 6xy.


11.3 Exercises 119

Solution: This is already solved above. Let us confirm this using the theorem. First, we rewrite
the equation as x3 + y3 − 6xy = 0. So,

F(x, y) = x3 + y3 − 6xy.

Then

Fx (x, y) = 3x2 − 6y and Fy (x, y) = 3y2 − 6x.


Terms of use: This course material is strictly for class use only to students officially
enrolled in MATH 54, ELEMENTARY ANALYSIS II, SECOND SEMESTER, A.Y. 2023 - 2024

Thus,

printed, reproduced, shared, or sold without permission from the author.


dy Fx (x, y) 3x2 − 6y x2 − 2y

Copyright protection is governed by R.A. 10372. This work cannot be


=− =− 2 =− 2 .
dx Fy (x, y) 3y − 6x y − 2x

■ Example 11.9 Find ∂ z/∂ x and ∂ z/∂ y for

3x2 z − x2 y2 + 2z3 + 3yz − 5 = 0.

Solution: Let
f (x, y, z) = 3x2 z − x2 y2 + 2z3 + 3yz − 5.

Then

Fx (x, y, z) = 6xz − 2xy2 , Fy (x, y, z) = −2x2 y

and
Fz (x, y, z) = 3x2 + 6z2 + 3y.

Using the theorem, we have


∂z Fx (x, y, z) 2x2 y − 6xz
=− = 2
∂x Fz (x, y, z) 3x + 6z2 + 3y
and
∂z Fy (x, y, z) 2x2 y − 3z
=− = 2 .
∂y Fz (x, y, z) 3x + 6z2 + 3y

11.3 Exercises
Terms of use: This course material is strictly for class use only to students officially
enrolled in MATH 54, ELEMENTARY ANALYSIS II, SECOND SEMESTER, A.Y. 2023 - 2024

11.3 Exercises

(c) f (x, y) =
x
y
(b) z = ln(x + t 2 )

(b) yz + x ln y = z2
(a) x2 + 2y2 + 3z2 = 1
(a) f (x, y) = x4 + 5xy3
Exercise 11.1 Do as directed.

1. Find the first partial derivatives of the following functions

2. Use implicit differentiation to find ∂ z/∂ x and ∂ z/∂ y of the following.


120

Copyright protection is governed by R.A. 10372. This work cannot be


printed, reproduced, shared, or sold without permission from the author.
Terms of use: This course material is strictly for class use only to students officially
enrolled in MATH 54, ELEMENTARY ANALYSIS II, SECOND SEMESTER, A.Y. 2023 - 2024

XII

12

12.2
12.1
Exercises
Maximum and Minimum Values of Functions
Week Twelve

Partial Derivatives . . . . . . . . . . . . . . . . . . . . 122


Copyright protection is governed by R.A. 10372. This work cannot be
printed, reproduced, shared, or sold without permission from the author.
Terms of use: This course material is strictly for class use only to students officially
enrolled in MATH 54, ELEMENTARY ANALYSIS II, SECOND SEMESTER, A.Y. 2023 - 2024

printed, reproduced, shared, or sold without permission from the author.


Copyright protection is governed by R.A. 10372. This work cannot be
12. Partial Derivatives

At the end of this topic you are expected to:


• find the critical point of functions of several variables
• use the partial derivative tests in finding extrema of functions

12.1 Maximum and Minimum Values of Functions

We will extend our techniques in finding extrema of functions in two variables to functions in
three variables. An analogous approach will be dealt.

Theorem 12.1.1 (Extreme Value Theorem)

Let f be a continuous function of two variables x and y defined on a closed and bounded
region R in the xy-plane
1. There is at least one point in R at which f takes on a minimum value.
2. There is at least one point in R at which f takes on a maximum value.
12.1 Maximum and Minimum Values of Functions 123

Definition 12.1.1 (Relative Extrema)

Let f be a function defined on a region R containing (x0 , y0 ).


1. The function f has a relative mimimum at (x0 , y0 ) if f (x, y) ≥ f (x0 , y0 ) for all (x, y)
in the open disk containing (x0 , y0 ).
2. The function f has a relative maximum at (x0 , y0 ) if f (x, y) ≤ f (x0 , y0 ) for all (x, y)
in the open disk containing (x0 , y0 ).
Terms of use: This course material is strictly for class use only to students officially

Definition 12.1.2 (Critical Point)


enrolled in MATH 54, ELEMENTARY ANALYSIS II, SECOND SEMESTER, A.Y. 2023 - 2024

printed, reproduced, shared, or sold without permission from the author.


Let f be a function defined on a region R containing (x0 , y0 ). The point (x0 , y0 ) is a critical

Copyright protection is governed by R.A. 10372. This work cannot be


point of f if one of the following is true.
1. fx (x0 , y0 ) = 0 and fy (x0 , y0 ) = 0
2. fx (x0 , y0 ) or fy (x0 , y0 ) does not exist.

Theorem 12.1.2 (Relative Extrema Occur Only at Critical Points)

If f has a relative extremum at (x0 , y0 ) on an open region R, then (x0 , y0 ) is a critical point
of f .

■ Example 12.1 Find all relative extrema of the function

f (x, y) = x2 + y2 − 2x − 6y + 14.

Solution: Let us find first fx and fy . In our case,

fx (x, y) = 2x − 2 and fy (x, y) = 2x − 6.

Setting fx (x, y) = 0 and fy (x, y) = 0, and solving for x and y, we obtain x = 1 and y = 3. Hence,
(1, 3) is the only critical point. Moreover, f (1, 3) = 4.
This time we have no tools yet to confirm whether this point corresponds to a relative
maximum or a relative minimum. But we can still manage to confirm it by completing the
square.
We can rewrite the given function as

f (x, y) = 4 + (x − 1)2 + (y − 3)2 .

Note that (x − 1)2 ≥ 0 and (y − 3)2 ≥ 0. Hence, f (x, y) ≥ 4 for all x and y.
12.1 Maximum and Minimum Values of Functions 124

Therefore, f (1, 3) = 4 is a relative minimum. In fact it is the absolute minimum as


confirmed in the graph below.
Terms of use: This course material is strictly for class use only to students officially
enrolled in MATH 54, ELEMENTARY ANALYSIS II, SECOND SEMESTER, A.Y. 2023 - 2024

printed, reproduced, shared, or sold without permission from the author.


Copyright protection is governed by R.A. 10372. This work cannot be
An elliptic paraboloid, minimum at the vertex (1, 3, 4)

Just like for functions in one variable, a critical point of a function of two variables do not
always yield relative maxima or minima. There are times when a critical point yield a saddle
point, which is neither relative maximum or relative minimum. The graph of the surface
below is the hyperbolic paraboloid z = x2 − y2 with saddle point indicated by a black dot. A
shape similar to this is the “pringles” potato chip.
12.1 Maximum and Minimum Values of Functions 125
Terms of use: This course material is strictly for class use only to students officially
enrolled in MATH 54, ELEMENTARY ANALYSIS II, SECOND SEMESTER, A.Y. 2023 - 2024

printed, reproduced, shared, or sold without permission from the author.


Copyright protection is governed by R.A. 10372. This work cannot be
A hyperbolic paraboloid with saddle point (0, 0, 0)-black dot

The next theorem will give us the answer on how do we identify critical points in relation
to extrema.

Theorem 12.1.3 (Second Partials Tests)

Suppose that f have continuous second partial derivatives on an open region containing a
point (a, b) for which

fx (x, y) = 0 and fy (a, b) = 0.

To test for relative extrema of f , consider the quantity

d = fxx (a, b) fyy (a, b) − [ fxy (a, b)]2 .

1. If d > 0 and fxx (a, b) > 0, then f has a relative minimum at (a, b).
2. If d > 0 and fxx (a, b) < 0, then then f has a relative maximum at (a, b).
3. If d < 0, then (a, b, f (a, b)) is a saddle point.
4. The test is inconclusive if d = 0.

■ Example 12.2 Find the relative extrema of f (x, y) = −x3 + 4xy − 2y2 + 1.
12.1 Maximum and Minimum Values of Functions 126

Solution: We will find first the critical points. Taking the first partials,

fx (x, y) = −3x2 + 4y and fy (x, y) = 4x − 4y

and setting them to 0 we have,

−3x2 + 4y = 0 and 4x − 4y = 0
Terms of use: This course material is strictly for class use only to students officially

From the second equation, we have x = y. Substituting x = y to the first equation, we have
enrolled in MATH 54, ELEMENTARY ANALYSIS II, SECOND SEMESTER, A.Y. 2023 - 2024

−3x2 + 4x = 0 or −x(3x − 4) = 0 which means that x = 0 or x = 4/3.

printed, reproduced, shared, or sold without permission from the author.


Since x = y, the points (0, 0) and (4/3, 4/3) are critical points. Taking the second partials,

Copyright protection is governed by R.A. 10372. This work cannot be


we have

fxx (x, y) = −6x, fyy (x, y) = −4, and fxy (x, y) = 4

For the critical point (0, 0),

d = fxx (0, 0) fyy (0, 0) − [ fxy (0, 0)]2 = 0 − 16 < 0.

Thus, by the Second Partials Test, we can conclude that (0, 0, 1) is a saddle point of f .
For the critical point (4/3, 4/3), we have

d = fxx (4/3, 4/3) fyy (4/3, 4/3) − [ fxy (4/3, 4/3)]2 = −8(−4) − 16 = 16 > 0.

and since fxx ( 43 , 43 ) = −8 by the Second Partials Test, we can conclude that ( 43 , 43 , 27
59
) is a
relative maximum of f .

■ Example 12.3 A rectangular box with no lid is to be made from 12 m2 cardboard. Find the

maximum volume of such box.

Solution: Let the length, width and height of the box be represented by the variables x, y, and z
respectively. Then the volume of the box is

V = xyz.

The total surface area of the box is 2xz + 2yz + xy. Hence,

2xz + 2yz + xy = 12
12.2 Exercises 127

Solving for z in 2xz + 2yz + xy = 12, we have


12 − xy
z=
2(x + y)
Thus, we can write V as
12 − xy 12xy − x2 y2
V = xy =
2(x + y) 2(x + y)
Terms of use: This course material is strictly for class use only to students officially

Taking the partial derivatives of V , we obtain


enrolled in MATH 54, ELEMENTARY ANALYSIS II, SECOND SEMESTER, A.Y. 2023 - 2024

y2 (12 − 2xy − x2 ) x2 (12 − 2xy − y2 )


Vx = and Vy =

printed, reproduced, shared, or sold without permission from the author.


2(x + y)2 2(x + y)2

Copyright protection is governed by R.A. 10372. This work cannot be


One possible critical point is (0, 0) beacause x = 0 and y = 0 makes both partial derivatives
0. However, this would also mean that V = 0 which we do not want.
The other point that would make Vx and Vy equal to zero is requiring that

12 − 2xy − x2 = 0 and 12 − 2xy − y2 = 0

From the second equation we have 2xy = 12 − y2 and substituting to the first equation we
obtain y2 − x2 = 0 which means that (y − x)(y + x) = 0. Hence, x = y or x = −y. We reject
x = −y since x and y are all positive (lengths).
If x = y, then from 12 − 2xy − x2 = 0, we have 12 − 2x2 − x2 = 0 or 12 − 3x2 = 0 for which
x = 2 and it follows that y = 2.
12 − 2(2)
Solving for z, we have z = = 1.
2(2 + 2)
We can use the second partial derivatives test to verify that the point (2, 2) will give us the
maximum.
Therefore, the maximum volume that can be obtained is V = 2 · 2 · 1 = 4 m3 .

12.2 Exercises
Exercise 12.1 Do as directed.

1. Identify any extrema of the function by recognizing its given form or its form after
completing the square.
(a) f (x, y) = (x − 1)2 + (y − 3)2
(b) f (x, y) = x2 + y2 + 2x − 6y + 6
2. Find all relative extrema and saddle points of the function. Use the Second Partials
Terms of use: This course material is strictly for class use only to students officially
enrolled in MATH 54, ELEMENTARY ANALYSIS II, SECOND SEMESTER, A.Y. 2023 - 2024

12.2 Exercises

Test where applicable.

(b) f (x, y) = x2 − y2 − x − y
(a) f (x, y) = x2 + y2 + 8x − 12y − 3
128

Copyright protection is governed by R.A. 10372. This work cannot be


printed, reproduced, shared, or sold without permission from the author.
Terms of use: This course material is strictly for class use only to students officially
enrolled in MATH 54, ELEMENTARY ANALYSIS II, SECOND SEMESTER, A.Y. 2023 - 2024

XIII

13

13.2
13.1
Exercises
Double Integrals Over Rectangles
Week Thirteen

Multiple Integrals . . . . . . . . . . . . . . . . . . . . 130


Copyright protection is governed by R.A. 10372. This work cannot be
printed, reproduced, shared, or sold without permission from the author.
Terms of use: This course material is strictly for class use only to students officially
enrolled in MATH 54, ELEMENTARY ANALYSIS II, SECOND SEMESTER, A.Y. 2023 - 2024

printed, reproduced, shared, or sold without permission from the author.


Copyright protection is governed by R.A. 10372. This work cannot be
13. Multiple Integrals

At the end of this topic you are expected to:


• evaluate multiple integrals
• evaluate double integrals over rectangles

13.1 Double Integrals Over Rectangles

For integrals of functions of a single variable, we begin by a continuous function f defined


on a closed interval [a, b] and then dividing this interval into n subintervals, each with length
∆x = (b − a)/n. Then we form the Riemann sum
n
∑ f (xi∗)∆x
i=1

which is just the sum of the areas of n rectangles. Next, we take the limit of the above sum
as n → ∞ and we obtain
n Z b
lim ∑ f (xi∗ )∆x = f (x) dx
n→∞ a
i=1
13.1 Double Integrals Over Rectangles 131

which is called the definite integral of f evaluated from a to b and is interpreted as the area
of the region under the curve f , above the x-axis and between the lines x = a and x = b.

13.1.1 Volumes and Double Integrals

A similar approach for functions in two variables will be considered in obtainingthe volume of
Terms of use: This course material is strictly for class use only to students officially

a solid.
enrolled in MATH 54, ELEMENTARY ANALYSIS II, SECOND SEMESTER, A.Y. 2023 - 2024

Let f be a function defined on a a closed rectangle

printed, reproduced, shared, or sold without permission from the author.


Copyright protection is governed by R.A. 10372. This work cannot be
R = [a, b] × [c, d] = {(x, y) ∈ R2 : a ≤ x ≤ b, c ≤ y ≤ d}

with f (x, y) ≥ 0. Let S be the solid above the region R and under the surface z = f (x, y) as
shown in the figure below

Thus, S = {(x, y, z) ∈ R3 : 0 ≤ z ≤ f (x, y), (x, y) ∈ R}. Then we subdivide R into subrect-
angles with width (along x-axis) ∆x = (b − a)/m and width (along y-axis) ∆y = (c − d)/n as
seen in the figue below.
13.1 Double Integrals Over Rectangles 132
Terms of use: This course material is strictly for class use only to students officially
enrolled in MATH 54, ELEMENTARY ANALYSIS II, SECOND SEMESTER, A.Y. 2023 - 2024

printed, reproduced, shared, or sold without permission from the author.


Copyright protection is governed by R.A. 10372. This work cannot be
The shaded rectangle in figure above is called Ri j or the rectangle in the ith row and jth
column. The area of this rectangle is ∆A = ∆x∆y. Suppose we pick a sample point (xi∗j , y∗i j ) on
Ri j . Then we can construct a small box whose base area is ∆A and whose height is f (xi∗j , y∗i j )
as shown in the figure below.

Similarly, for any other reactngles there corresponds a rectangular box after picking a
sample point. This m × n rectangles will approximate the volume of the solid as illustared in
the blue blocks below.
Terms of use: This course material is strictly for class use only to students officially
enrolled in MATH 54, ELEMENTARY ANALYSIS II, SECOND SEMESTER, A.Y. 2023 - 2024

that is
13.1 Double Integrals Over Rectangles

m
V≈∑
n

i=1 j=1
∑ f (xi∗j , y∗i j )∆A.
133

If we add the volumes of these rectangular solid, we have an approximate volume for S,
Copyright protection is governed by R.A. 10372. This work cannot be
printed, reproduced, shared, or sold without permission from the author.
13.1 Double Integrals Over Rectangles 134

As m and n get larger, we have


m n
V = lim
m,n→∞
∑ ∑ f (xi∗j , y∗i j )∆A.
i=1 j=1

Definition 13.1.1 (Double Integral)

The double integral of f over the reactangle R is


x m n
Terms of use: This course material is strictly for class use only to students officially

f (x, y) dA = lim
m,n→∞
∑ ∑ f (xi∗j , y∗i j )∆A.
R i=1 j=1
enrolled in MATH 54, ELEMENTARY ANALYSIS II, SECOND SEMESTER, A.Y. 2023 - 2024

printed, reproduced, shared, or sold without permission from the author.


if this limit exists.

Copyright protection is governed by R.A. 10372. This work cannot be


Theorem 13.1.1 (Properties of Double Integrals)

Let f and g be continuous over a closed, bounded plane region R, and let c be a constant.
Then
x x
1. c f (x, y) dA = c f (x, y) dA
R R
x x x
2. [ f (x, y) ± g(x, y)] dA = f (x, y) dA ± g(x, y) dA
R R R
x
3. f (x, y) dA ≥ 0, if f (x, y) ≥ 0
R
x x
4. f (x, y) dA ≥ g(x, y) dA, if f (x, y) ≥ g(x, y)
R R
x x x
5. f (x, y) dA = f (x, y) dA + f (x, y) dA
R R1 R2

where R is the union of two nonoverlapping subregions R1 and R2 .

■ Example 13.1 Estimate the volume of the solid that lies above the square R = [0, 2] × [0, 2]

and below z = 16 − x2 − 2y2 . Divide R into four equal squares and choose the sample point to
be the upper right corner of each square.

Solution: We will subdivide the square into four equal regions

The area of each square is ∆A = 1. Approximating the volume, we have


13.1 Double Integrals Over Rectangles 135
Terms of use: This course material is strictly for class use only to students officially
enrolled in MATH 54, ELEMENTARY ANALYSIS II, SECOND SEMESTER, A.Y. 2023 - 2024

printed, reproduced, shared, or sold without permission from the author.


Copyright protection is governed by R.A. 10372. This work cannot be
m n
V≈∑ ∑ f (xi∗j , y∗i j )∆A
i=1 j=1

= f (1, 1) ∆A + f (1, 2) ∆A + f (2, 1) ∆A + f (2, 2) ∆A

= 13(1) + 7(1) + 10(1) + 4(1) = 34.

Therfore, V ≈ 34 cubic units.

13.1.2 The Midpoint Rule

The methods that we used for approximating single integrals (the Midpoint Rule, the Trape-
zoidal Rule, Simpson’s Rule) all have counterparts for double integrals. Here we consider
only the Midpoint Rule for double integrals. This means that we use a double Riemann sum to
approximate the double integral, where the sample point (xi∗j , y∗i j ) in Ri j is chosen to be the
center (xi , y j ). In other words, xi is the midpoint of [xi−1 , xi ] and yi is the midpoint of [yi−1 , yi ].
13.1 Double Integrals Over Rectangles 136

Midpoint Rule for Double Integrals

x m n
f (x, y) dA ≈ ∑ ∑ f (xi, y j )∆A
R i=1 j=1

where xi is the midpoint of [xi−1 , xi ] and yi is the midpoint of [yi−1 , yi ].


Terms of use: This course material is strictly for class use only to students officially


x
Example 13.2 Use midpoint rule for double integrals with n = m = 2 to estimate the value of
enrolled in MATH 54, ELEMENTARY ANALYSIS II, SECOND SEMESTER, A.Y. 2023 - 2024

the integral (x − 3y2 ) dA, where R = {(x, y) | 0 ≤ x ≤ 2, 1 ≤ y ≤ 2}.

printed, reproduced, shared, or sold without permission from the author.


Copyright protection is governed by R.A. 10372. This work cannot be
R
Solution: In using the midpoint rule with m = n = 2, we evaluate f (x, y) = x − 3y2 at the
centers of the four subrectangles as shown below.

So, x1 = 12 , x2 = 32 , y1 = 45 , and y2 = 74 . Thus, the desired midpoints are ( 12 , 54 ), ( 12 , 74 ), ( 32 , 54 )


13.2 Exercises 137

and ( 23 , 74 ). The area of each subrectangle is ∆A = 21 . Hence,


x 2 2
(x − 3y2 ) dA ≈ ∑ ∑ f (xi, y j )∆A
R i=1 j=1

= f (x1 , y1 ) ∆A + f (x1 , y2 ) ∆A + f (x2 , y1 ) ∆A + f (x2 , y2 ) ∆A


       
1 5 1 7 3 5 3 7
=f , ∆A + f , ∆A + f , ∆A + f , ∆A
2 4 2 4 2 4 2 4
Terms of use: This course material is strictly for class use only to students officially

       
67 1 139 1 51 1 123 1
= − + − + − + −
enrolled in MATH 54, ELEMENTARY ANALYSIS II, SECOND SEMESTER, A.Y. 2023 - 2024

16 2 16 2 16 2 16 2

printed, reproduced, shared, or sold without permission from the author.


95

Copyright protection is governed by R.A. 10372. This work cannot be


=−
8
x
Therefore, (x − 3y2 ) dA ≈ −11.875.
R

13.2 Exercises
Exercise 13.1 Do as directed.

1. Estimate the volume of the solid that lies below the surface z = xy and above the
rectangle R = {(x, y) | 0 ≤ x ≤ 6, 0 ≤ y ≤ 4}
(a) using Riemann sum with m = 3, n = 2, and take the sample point to be the upper
right corner of each square
(b) using midpoint rule with same values of m and n in problem (a).
Terms of use: This course material is strictly for class use only to students officially
enrolled in MATH 54, ELEMENTARY ANALYSIS II, SECOND SEMESTER, A.Y. 2023 - 2024

XIV

14

14.2
14.1
Exercises
Iterated Integrals
Week Fourteen

Multiple Integrals . . . . . . . . . . . . . . . . . . . . 139


Copyright protection is governed by R.A. 10372. This work cannot be
printed, reproduced, shared, or sold without permission from the author.
Terms of use: This course material is strictly for class use only to students officially
enrolled in MATH 54, ELEMENTARY ANALYSIS II, SECOND SEMESTER, A.Y. 2023 - 2024

printed, reproduced, shared, or sold without permission from the author.


Copyright protection is governed by R.A. 10372. This work cannot be
14. Multiple Integrals

At the end of this topic you are expected to:


• evaluate iterated integrals
• find the area of plane regions using iterated integrals

14.1 Iterated Integrals

Suppose that f is a function of two variables that is integrable on the rectangle R = [a, b]×[c, d].
We use the notation
Z
f (x, y) dx

to mean that f (x, y) is integrated with respect to x and the variable y is held fixed. This is also
called partial integration with respect to x.

Z
■ Example 14.1 Find 2xy dx
14.1 Iterated Integrals 140

Solution:
Z Z
2xy dx = 2y x dx
x2

= 2y +C(y)
2
= x2 y +C(y)
Terms of use: This course material is strictly for class use only to students officially

where C(y) is a function of y.


enrolled in MATH 54, ELEMENTARY ANALYSIS II, SECOND SEMESTER, A.Y. 2023 - 2024

printed, reproduced, shared, or sold without permission from the author.


Copyright protection is governed by R.A. 10372. This work cannot be
Rd
Now if c f (x, y) dy, a number that depends on the value of x, so it defines a function of x,
say,
Z d
A(x) = f (x, y) dy.
c
If we integrate A with respect to x from x = a to x = b, we get
Z b Z b Z d 
A(x) dx = f (x, y) dy dx.
a a c

The integral on the right side of the equation is called an iterated integral. Most of the times
the brackets are omitted.

■ Example 14.2 Evaluate the following integrals

Z 3Z 2 Z 2Z 3
2
1. x y dy dx 2. x2 y dx dy
0 1 1 0
Solution:
Z 3Z 2 Z 3 Z 2  Z 3 2 y=2
2 2 2y
1. x y dy dx = x y dy dx = x dx
0 1 0 1 0 2 y=1
22 12
Z 3    
2 2
= x −x dx
0 2 2
Z 3
3 2
= x dx
0 2
3
x3

27
= = .
2 0 2
14.1 Iterated Integrals 141

Z 2Z 3 Z 2 Z 2  Z 3  3 x=3
2 2 x
2. x y dx dy = x y dx dy = y dy
1 0 1 1 0 3 x=0
Z 2  3   3 
3 0
= y− y dy
1 3 3
Z 2
= 9y dy
Terms of use: This course material is strictly for class use only to students officially

1
9 2 2 27
 
= y = .
enrolled in MATH 54, ELEMENTARY ANALYSIS II, SECOND SEMESTER, A.Y. 2023 - 2024

2 1 2

printed, reproduced, shared, or sold without permission from the author.


Copyright protection is governed by R.A. 10372. This work cannot be
Notice that in the example above, we obtained the same result. In general, two iterated
integrals are equal, that is, the order of integration doesn’t matter whether you are integrating
with respect to x or y first.
The following theorem gives a practical method for evaluating a double integral by express-
ing it as an iterated integral (in either order).

Theorem 14.1.1 (Fubini’s Theorem)

If f is continuous on the rectangle R = {(x, y) | a ≤ x ≤ b, c ≤ y ≤ d}, then


x Z bZ d Z dZ b
f (x, y) dA = f (x, y) dy dx = f (x, y) dx dy
a c c a
R
x
■ Example 14.3 Evaluate the double integral (x − 3y2 ) dA, where R is the region given by
R
R = {(x, y) | 0 ≤ x ≤ 2, 1 ≤ y ≤ 2}.

Solution: Fubini’s Theorem gives


x 2
Z 2Z 2
(x − 3y ) dA = (x − 3y2 ) dy dx
0 1
R
Z 2 y=2
= xy − y3 y=1
dx
0
Z 2
= (x − 7) dx
0
 2  2
x
= − 7x = −12.
2 0
14.1 Iterated Integrals 142

Note that you can also use the other way and arrived at the same value, that is
x 2
Z 2Z 2
(x − 3y ) dA = (x − 3y2 ) dx dy = −12.
1 0
R

Notice the negative answer in the example above; nothing is wrong with that. The function
f is not a positive function, so its integral doesn’t represent a volume. Here, f is always
negative on R, so the value of the integral is the negative of the volume that lies above the
Terms of use: This course material is strictly for class use only to students officially

graph of f and below R.


enrolled in MATH 54, ELEMENTARY ANALYSIS II, SECOND SEMESTER, A.Y. 2023 - 2024

printed, reproduced, shared, or sold without permission from the author.


Example 14.4 Evaluate y sin (xy) dA, where R = [1, 2] × [0, π]

Copyright protection is governed by R.A. 10372. This work cannot be


R
Solution: If we first integrate with respect to x, we get
x Z πZ 2
y sin (xy) dA = y sin (xy) dx dy
0 1
R
Z π x=2
= − cos (xy) x=1 dy
Z0π
= (− cos 2y + cos y) dy
0
  π
1
= − sin 2y + sin y
2 0

= 0.
s
For a function f that takes on both positive and negative values, R y sin (xy) dA is a
difference of volumes: V1 and V2 , where V1 is the volume above R and below the graph of f ,
and V2 is the volume below R and above the graph. The fact that the integral in Example 14.4
is 0 means that these two volumes V1 and V2 are equal.
If we reverse the order of integration and first integrate with respect to y in Example 14.4,
we get
x Z 2Z π
y sin (xy) dA = y sin (xy) dy dx
1 0
R
but this order of integration is much more difficult than the method given in the example
because it involves integration by parts twice. Therefore, when we evaluate double integrals it
is wise to choose the order of integration that gives simpler integrals.
14.1 Iterated Integrals 143

In the special case where f (x, y) can be factored as the product of a function of x only and a
function of y only, the double integral of f can be written in a particularly simple form. To be
specific, suppose that f (x, y) = g(x)h(y) and R = [a, b] × [c, d]. Then Fubini’s Theorem gives
x Z dZ b Z d Z b 
f (x, y) dA = g(x) h(y) dx dy = g(x) h(y) dx dy
c a c a
R
In the inner integral, y is a constant, so h(y) is a constant and we can write
Terms of use: This course material is strictly for class use only to students officially

Z d Z b  Z d Z b  Z b Z d
g(x) h(y) dx dy = h(y) g(x) dx dy = g(x) dx h(y) dy
enrolled in MATH 54, ELEMENTARY ANALYSIS II, SECOND SEMESTER, A.Y. 2023 - 2024

c a c a a c

printed, reproduced, shared, or sold without permission from the author.


Therefore, in this case the double integral of f can be written as the product of two single

Copyright protection is governed by R.A. 10372. This work cannot be


integrals:

x Z b Z d
g(x) h(y) dA = g(x) dx h(y) dy, where R = [a, b] × [c, d]
a c
R

■ Example 14.5
Z 3Z 2 Z 3 Z 2
2 2
x y dy dx = x dx y dy
0 1 0 1
1 3 3 1 2 2
   
= x · y
3 0 2 1
 
1
= (9 − 0) · 2 −
2
3
= 9·
2
27
= .
2
Iterated integrals can also be used in finding the area of plane regions. In your previous
calculus course, you have learned that the area between two curves can be obtained by
integration. Particularly, if R is the region between continuous functions f (x) and g(x) such
that g(x) ≤ R ≤ f (x) from x = a to x = b, then the area A of region R is given by
Z b
A= [ f (x) − g(x)] dx
a
The region g(x) ≤ R ≤ f (x) is called vertically simple while g(y) ≤ R ≤ f (y) is called
horizontally simple. The figures below are simple regions.
14.1 Iterated Integrals 144
Terms of use: This course material is strictly for class use only to students officially
enrolled in MATH 54, ELEMENTARY ANALYSIS II, SECOND SEMESTER, A.Y. 2023 - 2024

printed, reproduced, shared, or sold without permission from the author.


Copyright protection is governed by R.A. 10372. This work cannot be
14.1.1 Area of a Plane Region

Now, we will deal with area of plane regions but this time we will make use of iterated integrals.

Area of a Plane Region

1. If R is defined by a ≤ x ≤ b and g1 (x) ≤ y ≤ g2 (x), where g1 and g2 are continuous


on [a, b], then the area of R is
Z b Z g2 (x)
A= dy dx.
a g1 (x)

2. If R is defined by c ≤ y ≤ d and h1 (y) ≤ x ≤ h2 (y), where h1 and h2 are continuous


on [c, d], then the area of R is
Z d Z h2 (y)
A= dx dy.
c h1 (y)

■ Example 14.6 Use iterated integrals to find the area of the simple region which is bounded
π 5π
by the graphs of f (x) = sin x and g(x) = cos x between x = 4 and x = 4 .
14.1 Iterated Integrals 145

Solution: We can easily verify by sketching that this is indeed a simple region with f (x) = sin x
as the upper curve and g(x) = cos x as the lower curve. Thus,
Z 5π/4 Z sin x
A= dy dx
π/4 cos x
Z 5π/4 sin x
= y dx
π/4 cos x
Terms of use: This course material is strictly for class use only to students officially

Z 5π/4
= (sin x − cos x) dx
enrolled in MATH 54, ELEMENTARY ANALYSIS II, SECOND SEMESTER, A.Y. 2023 - 2024

π/4
5π/4

printed, reproduced, shared, or sold without permission from the author.


= (− cos x − sin x)

Copyright protection is governed by R.A. 10372. This work cannot be


π/4

= 2 2.

Therefore, the area of the region is 2 2 square units.

■ Example 14.7 Use iterated integrals to find the area of the simple region which is bounded

by the graphs of f (x) = 4x − x2 and g(x) = 4 − x.

Solution: First, we sketch the region


14.2 Exercises 146

We can see that the region is vertically simple. Thus


Z 4 Z 4x−x2
A= dy dx
1 4−x
2
Z 4 4x−x
= y dx
1 4−x
Z 4
= [(4 − x) − (4x − x2 ))] dx
Terms of use: This course material is strictly for class use only to students officially

1
Z 4
(−x2 + 5x − 4) dx
enrolled in MATH 54, ELEMENTARY ANALYSIS II, SECOND SEMESTER, A.Y. 2023 - 2024

=
1

printed, reproduced, shared, or sold without permission from the author.


4

Copyright protection is governed by R.A. 10372. This work cannot be


 
1 3 5 2
= − x + x − 4x
3 2 1
9
= .
2
9
Therefore, the area of the region is 2 square units.

14.2 Exercises
Exercise 14.1 Do as directed.

x
1. Evaluate the following double integrals.
(a) (2x + 1) dA, R = {(x, y) | 2 ≤ x ≤ 6, 0 ≤ y ≤ 4}
x
R
(b) (4 − 2y) dA, R = [0, 1] × [0, 1]
R
2. Calculate the following iterated integrals.
Z 1Z 1
(a) (x + y)2 dx dy
Z0 0
π/6 Z π/2
(b) (sin x + sin y) dy dx
Z04 Z 2 0 
x y
(c) + dy dx
1 1 y x
Terms of use: This course material is strictly for class use only to students officially
enrolled in MATH 54, ELEMENTARY ANALYSIS II, SECOND SEMESTER, A.Y. 2023 - 2024

XV

15

15.3
15.2
15.1

Exercises
Double Integrals in Polar Coordinates
Double Integrals Over General Regions
Week Fifteen

Multiple Integrals . . . . . . . . . . . . . . . . . . . . 148


Copyright protection is governed by R.A. 10372. This work cannot be
printed, reproduced, shared, or sold without permission from the author.
Terms of use: This course material is strictly for class use only to students officially
enrolled in MATH 54, ELEMENTARY ANALYSIS II, SECOND SEMESTER, A.Y. 2023 - 2024

printed, reproduced, shared, or sold without permission from the author.


Copyright protection is governed by R.A. 10372. This work cannot be
15. Multiple Integrals

At the end of this topic you are expected to:


• evaluate double integrals over general region
• evaluate double integrals in polar coordinates

15.1 Double Integrals Over General Regions

For single integrals, the region over which we integrate is always an interval. But for double
integrals, we want to be able to integrate a function f not just over rectangles but also over
regions D of more general shape, such as the one illustrated below
15.1 Double Integrals Over General Regions 149
Terms of use: This course material is strictly for class use only to students officially
enrolled in MATH 54, ELEMENTARY ANALYSIS II, SECOND SEMESTER, A.Y. 2023 - 2024

printed, reproduced, shared, or sold without permission from the author.


Copyright protection is governed by R.A. 10372. This work cannot be
We suppose that D is a bounded region, which means that D can be enclosed in a rectangular
region R as shown above (in blue rectangle). Then we define a new function F with domain R
by 
 f (x, y) if (x, y) is in D

F(x, y =)
0 if (x, y) is in R but not in D

If F is integrable over R, then we define the double integral of f over D by

x x
f (x, y) dA = F(x, y) dA
D R

s
This definition makes sense because R is a rectangle and so R F(x, y) dA has been previ-
ously defined. The procedure that we have used is reasonable because the values of F(x, y)
are 0 when (x, y) lies outside D and so they contribute nothing to the integral. This means that
it doesn’t matter what rectangle R we use as long as it contains D.
s
In the case where f (x, y) ≥ 0, we can still interpret D f (x, y) dA as the volume of the solid
15.1 Double Integrals Over General Regions 150

that lies above D and under the surface z = f (x, y) (the graph of f ).

A plane region D is said to be of Type I if it lies between the graphs of two continuous
functions of x, that is,

D = {(x, y) | a ≤ x ≤ b, g1 (x) ≤ y ≤ g2 (x)}.


Terms of use: This course material is strictly for class use only to students officially

Below are sketches of regions of type I.


enrolled in MATH 54, ELEMENTARY ANALYSIS II, SECOND SEMESTER, A.Y. 2023 - 2024

printed, reproduced, shared, or sold without permission from the author.


Copyright protection is governed by R.A. 10372. This work cannot be
Theorem 15.1.1 If f is continuous on a type I region D such that

D = {(x, y) | a ≤ x ≤ b, g1 (x) ≤ y ≤ g2 (x)},

then
x Z b Z g2 (x)
f (x, y) dA = f (x, y) dy dx.
a g1 (x)
D
x
■ Example 15.1 Evaluate (x + 2y) dA, where D is the region bounded by the parabolas
D
y = 2x2 and y = 1 + x2 .
15.1 Double Integrals Over General Regions 151

Solution: The parabolas intersect when 2x2 = 1 + x2 , that is x2 = 1 which means that x = ±1.
Then we sketch region D
Terms of use: This course material is strictly for class use only to students officially
enrolled in MATH 54, ELEMENTARY ANALYSIS II, SECOND SEMESTER, A.Y. 2023 - 2024

printed, reproduced, shared, or sold without permission from the author.


Copyright protection is governed by R.A. 10372. This work cannot be
We can see that D is a region of type I but not of type II and we can write D as

D = {(x, y) | − 1 ≤ x ≤ 1, 2x2 ≤ y ≤ 1 + x2 }.

Since the lower boundary is y = 2x2 and the upper boundary is y = 1 + x2 , we have
x Z 1 Z 1+x2
f (x, y) dA = (x + 2y) dy dx
−1 2x2
D
Z 1 y=1+x2
2
= xy + y dx
−1 y=2x2
Z 1  
2 2 22 2 2
= x(1 + x ) + (1 + x ) − x(2x ) + (2x ) dx
−1
Z 1
= (−3x4 − x3 + 2x2 + x + 1) dx
−1
 1
3 5 x4 2 3 x2

= − x − + x + +x
5 4 3 2 −1
32
= .
15
15.1 Double Integrals Over General Regions 152

A plane region D is said to be of Type II if it lies between the graphs of two continuous
functions of y, that is,

D = {(x, y) | c ≤ y ≤ d, h1 (y) ≤ x ≤ h2 (y)}.

Below are sketches of regions of type II.


Terms of use: This course material is strictly for class use only to students officially
enrolled in MATH 54, ELEMENTARY ANALYSIS II, SECOND SEMESTER, A.Y. 2023 - 2024

printed, reproduced, shared, or sold without permission from the author.


Copyright protection is governed by R.A. 10372. This work cannot be
Theorem 15.1.2 If f is continuous on a type II region D such that

D = {(x, y) | c ≤ y ≤ d, h1 (y) ≤ x ≤ h2 (y)},

then
x Z d Z h2 (y)
f (x, y) dA = f (x, y) dy dx.
c h1 (y)
D
x
■ Example 15.2 Evaluate xy dA, where D is the region bounded by the line y = x − 1 and
D
the parabola y2 = 2x + 6

Solution: Let us sketch first the region D.


15.1 Double Integrals Over General Regions 153
Terms of use: This course material is strictly for class use only to students officially
enrolled in MATH 54, ELEMENTARY ANALYSIS II, SECOND SEMESTER, A.Y. 2023 - 2024

printed, reproduced, shared, or sold without permission from the author.


Copyright protection is governed by R.A. 10372. This work cannot be
Here, D is both type I and type II, but the description of D as a type I region is more
complicated because the lower boundary consists of two parts. Therefore, we prefer to express
D as a type II region:

1 2
D = {(x, y) | − 2 ≤ y ≤ 4, 2y − 3 ≤ x ≤ y + 1}

Then
x Z 4 Z y+1 Z 4  2 x=y+1
x
xy dA = 1 2
xy dx dy = y dy
−2 2 y −3 −2 2 1
x= 2 y2 −3
D
Z 4 
1
y (y + 1)2 − ( 21 y2 − 3)2 dy

= 2
−2
Z 4 5 
1 y 3 2
=2 − + 4y + 2y − 8y dy
−2 4
 6 4
1 y 4 2 3 2
= − + y + y − 4y
2 24 3 −2

= 36.

■ Example 15.3 Find the volume of the solid that lies under the paraboloid z = x2 + y2 and

above the region D in the xy-plane bounded by the line y = 2x and the parabola y = x2 .
15.1 Double Integrals Over General Regions 154

Solution: (D as region of type I)


Sketching the D, we have
Terms of use: This course material is strictly for class use only to students officially
enrolled in MATH 54, ELEMENTARY ANALYSIS II, SECOND SEMESTER, A.Y. 2023 - 2024

printed, reproduced, shared, or sold without permission from the author.


Copyright protection is governed by R.A. 10372. This work cannot be
Here, our D can be described as

D = {(x, y) | 0 ≤ x ≤ 2, x2 ≤ y ≤ 2x}.

Therefore, the volume under z = x2 + y2 and above D is


x Z 2 Z 2x
V= (x2 + y2 ) dA = (x2 + y2 ) dy dx
0 x2
D
y=2x
y3
Z 2
2
= x y+ dx
0 3 y=x2
(2x)3 (x2 )3
Z 2 
2 2 2
= x (2x) + −x x − dx
0 3 3
Z 2 
1 6 4 14 3
= − x − x + x dx
0 3 3
1 7 1 5 7 4 2
 
= − x − x + x
21 5 6 0
216
= .
35
15.1 Double Integrals Over General Regions 155

Solution: (D as region of type II)


Sketching the D, we have
Terms of use: This course material is strictly for class use only to students officially
enrolled in MATH 54, ELEMENTARY ANALYSIS II, SECOND SEMESTER, A.Y. 2023 - 2024

printed, reproduced, shared, or sold without permission from the author.


Copyright protection is governed by R.A. 10372. This work cannot be
Here, our D can be described as

1 √
D = {(x, y) | 0 ≤ y ≤ 4, 2y ≤ x ≤ y}.

Therefore another expression for V is


x Z 4 Z √y
V= (x2 + y2 ) dA = 1
(x2 + y2 ) dx dy
0 2y
D
Z 4 3 x=√y
x
= + y2 x dy
0 3 1
x= 2 y
Z 4 
1 1 1
= y + y − − y3
3/2 5/2
dy
0 3 24 2
2 5/2 2 7/2 13 4 4
 
= y + y − y
15 7 96 0
216
= .
35
15.2 Double Integrals in Polar Coordinates 156

15.2 Double Integrals in Polar Coordinates

s
Suppose that we want to evaluate a double integral R f (x, y) dA over a circular region
or a region between two concentric circles. In either case the description of R in terms of
rectangular coordinates is rather complicated, but R is easily described using polar coordinates.
Recall that the polar coordinates (r, θ ) of a point are related to the rectangular coordinates
Terms of use: This course material is strictly for class use only to students officially

(x, y) by the equations


enrolled in MATH 54, ELEMENTARY ANALYSIS II, SECOND SEMESTER, A.Y. 2023 - 2024

printed, reproduced, shared, or sold without permission from the author.


Copyright protection is governed by R.A. 10372. This work cannot be
r2 = x2 + y2 x = r cos θ x = r sin θ

For instance the circle whose rectangular equation is x2 + y2 = 4 is equivalent to r = 2 in


polar coordinates while the circle centered at (0, 2) with radius 2 has equation x2 + (y − 2)2 = 4
is equivalent to r = 4 sin θ in polar coordinates.
15.2 Double Integrals in Polar Coordinates 157
Terms of use: This course material is strictly for class use only to students officially
enrolled in MATH 54, ELEMENTARY ANALYSIS II, SECOND SEMESTER, A.Y. 2023 - 2024

printed, reproduced, shared, or sold without permission from the author.


Copyright protection is governed by R.A. 10372. This work cannot be
The regions shown above are special cases of a polar rectangle

R = {(r, θ ) | a ≤ r ≤ b, α ≤ θ ≤ β }
s
In order to compute the double integral R f (x, y) dA, where R is a polar rectangle, we
divide the interval [a, b] into m subintervals [ri−1 , ri ] of equal width ∆r = (b − a)/m and we
divide the interval [α, β ] into n subintervals [θi−1 , θi ] of equal width ∆θ = (β − α)/n. Then
the circles r = ri and the rays θ = θ j divide the polar rectangle R into the small polar rectangles
Ri j shown below.
15.2 Double Integrals in Polar Coordinates 158
Terms of use: This course material is strictly for class use only to students officially
enrolled in MATH 54, ELEMENTARY ANALYSIS II, SECOND SEMESTER, A.Y. 2023 - 2024

printed, reproduced, shared, or sold without permission from the author.


Copyright protection is governed by R.A. 10372. This work cannot be
The “center” of the polar subrectangle

Ri j = {(r, θ ) | ri−1 ≤ r ≤ ri , θ j−1 ≤ θ ≤ θ j }

has polar coordinates

ri∗ = 12 (ri−1 + ri ) θ j∗ = 21 (θi−1 + θi )

We compute the area of Ri j using the fact that the area of a sector of a circle with radius r and
central angle θ is 12 r2 θ . Subtracting the areas of two such sectors, each of which has central
angle ∆θ = θ j − θ j−1 , we find that the area of Ri j is

∆Ai = 12 ri2 ∆θ − 12 ri−1


2 2
∆θ = 12 (ri2 − ri−1 ) ∆θ

= 21 (ri + ri−1 )(ri − ri−1 )∆θ = ri∗ ∆r ∆θ .

The rectangular coordinates of the center of of Ri j are (ri∗ cos θ j∗ , ri∗ sin θ j∗ ), so the typical
Riemann sum is
m n m n
∑∑ f (ri∗ cos θ j∗ , ri∗ sin θ j∗ ) ∆Ai = ∑ ∑ f (ri∗ cos θ j∗, ri∗ sin θ j∗) ri∗∆r ∆θ
i=1 j=1 i=1 j=1
15.2 Double Integrals in Polar Coordinates 159

If we write g(r, θ ) = r f (r cos θ r sin θ ), then the above Riemann sum can be written as
m n
∑ ∑ g(ri∗, θ j∗) ∆r ∆θ
i=1 j=1

which is a Riemann sum for the double integral


Z βZ b
g(r, θ ) dr dθ .
α a
Terms of use: This course material is strictly for class use only to students officially

Therefore, we have the following theorem.


enrolled in MATH 54, ELEMENTARY ANALYSIS II, SECOND SEMESTER, A.Y. 2023 - 2024

printed, reproduced, shared, or sold without permission from the author.


Theorem 15.2.1 Change to Polar Coodinates in a Double Integral

Copyright protection is governed by R.A. 10372. This work cannot be


If f is continuous on a polar rectangle R given by 0 ≤ a ≤ r ≤ b, α ≤ θ ≤ β , where
0 ≤ β − α ≤ 2π, then
x Z βZ b
f (x, y) dA = f (r cos θ , r sin θ ) r dr dθ .
α a
R
x
■ Example 15.4 Evaluate (3x + 4y2 ) dA, where R is the is the region in the upper half-plane
R
bounded by the circles x2 + y2 = 1 and x2 + y2 = 4.

Solution: The region R is clearly half-ring and in polar coordinates it is given by 1 ≤ r ≤ 2,


0 ≤ θ ≤ π. Then by Theorem 15.2.1, we have
x Z πZ 2
(3x + 4y2 ) dA = (3r cos θ + 4r2 sin2 θ ) r dr dθ
0 1
R
Z πZ 2
= (3r2 cos θ + 4r3 sin2 θ ) dr dθ
Z0π  1 r=2
= r3 cos θ + r4 sin2 θ r=1

0
Z π
= (7 cos θ + 15 sin2 θ ) dθ
Z0π 
7 cos θ + 15

= 2 (1 − cos 2θ ) dθ
0  π
15 15
= 7 sin θ + θ − sin 2θ
2 4 0
15
= π.
2
What we have done so far can be extended to the more complicated type of region. It’s
similar to the type II rectangular regions that we have considered before. Thus, the next
15.2 Double Integrals in Polar Coordinates 160

theorem will give us the following formula.

Theorem 15.2.2 If f is continuous on a polar region D of the form

D = {(r, θ ) | α ≤ θ ≤ β , h1 (θ ) ≤ r ≤ h2 (θ )}.

Then
x
Terms of use: This course material is strictly for class use only to students officially

Z β Z h2 (θ )
f (x, y) dA = f (r cos θ , r sin θ ) r dr dθ .
enrolled in MATH 54, ELEMENTARY ANALYSIS II, SECOND SEMESTER, A.Y. 2023 - 2024

α h1 (θ )
R

printed, reproduced, shared, or sold without permission from the author.


Copyright protection is governed by R.A. 10372. This work cannot be
In particular, taking f (x, y) = 1, h1 (θ ) = 0, and h2 (θ ) = h(θ ) in this formula, we see that
the area of region D bounded by θ = α, θ = β , and r = h(θ ) is
x Z β Z h(θ )
A(D) = 1 dA = r dr dθ .
α 0
D

■ Example 15.5 Use double integral to find the area of the region enclosed by one loop of the

four-leaved rose r = cos 2θ .

Solution: The graph of r = cos 2θ is shown below.


15.3 Exercises 161

Here, we see that

D = {(r, θ ) | 0 ≤ r ≤ 2 cos 2θ , −π/4 ≤ θ ≤ π/4}.

So the area of one loop is denoted by A(D) is


x Z π/4 Z cos θ
A(D) = dA = r dr dθ
−π/4 0
Terms of use: This course material is strictly for class use only to students officially

D
1 2 cos θ
Z π/4  
enrolled in MATH 54, ELEMENTARY ANALYSIS II, SECOND SEMESTER, A.Y. 2023 - 2024

= r dθ
−π/4 2

printed, reproduced, shared, or sold without permission from the author.


0

Copyright protection is governed by R.A. 10372. This work cannot be


Z π/4
= 1
2 cos2 2θ dθ
−π/4
Z π/4
1
= 4 (1 + cos 4θ ) dθ
−π/4
 π/4
1 1
= θ + sin 4θ
4 4 −π/4
π
= .
8

15.3 Exercises
Exercise 15.1 Do as directed.

x y
1. Evaluate the following double integrals.

(a) 2
dA, D = {(x, y) | 0 ≤ x ≤ 4, 0 ≤ y ≤ x}
x +1
x
D
(b) (x2 + 2y) dA, D is bounded by y = x, y = x3 , x ≥ 0
D

x
2. Evaluate the given integral by changing to polar coordinates.
(a) (x2 y) dA, where D is the top half of the disk with center the at the origin
D

x
and radius 5.
(b) (2x − y) dA, where R is the region in the first quadrant enclosed by the
R
circle x2 + y2 = 4 and the lines x = 0, y = x
Terms of use: This course material is strictly for class use only to students officially
enrolled in MATH 54, ELEMENTARY ANALYSIS II, SECOND SEMESTER, A.Y. 2023 - 2024

2016.
USA, 2014.
Bibliography

[1] Stewart, James, Calculus: Early Transcendentals, Eighth Edition, Cengage Learning,

[2] Larson, Ron and Edwards, Bruce Calculus, Eleventh Edition, Cengage Learning, USA,

Copyright protection is governed by R.A. 10372. This work cannot be


printed, reproduced, shared, or sold without permission from the author.

You might also like